apuntes geom diferencial

180
INTRODUCCI ´ ON A LA GEOMETR ´ IA DIFERENCIAL Dispone de un paquete de Mathematica para calcular m´ etricas, ımbolos de Christoffel, geod´ esicas y curvatura seccional Luis Javier HERN ´ ANDEZ PARICIO

Upload: abel-espartaco-luna

Post on 02-Jan-2016

68 views

Category:

Documents


2 download

TRANSCRIPT

Page 1: Apuntes Geom Diferencial

INTRODUCCION A LA

GEOMETRIA DIFERENCIAL

Dispone de un paquete de Mathematica para calcular metricas,sımbolos de Christoffel, geodesicas y curvatura seccional

Luis Javier HERNANDEZ PARICIO

Page 2: Apuntes Geom Diferencial

ii

Apuntes de Geometrıa Diferencial

Page 3: Apuntes Geom Diferencial

Indice general

INTRODUCCION 11. Breves comentarios historicos 12. Objetivos 53. Requisitos 6

Capıtulo 1. PRELIMINARES 91. Nociones y notaciones asociadas a funciones 92. Topologıa 102.1. Espacios topologicos y funciones continuas 102.2. Base de una topologıa 112.3. Propiedades topologicas 122.4. Construcciones 122.5. Algunos espacios 133. Algebra lineal 144. Analisis matematico 17Problemas 20

Capıtulo 2. VARIEDADES DIFERENCIABLES 231. La nocion de variedad diferenciable 23Problemas 262. La topologıa de una variedad diferenciable 29Problemas 323. Ejemplos de variedades diferenciables 33Problemas 364. El conjunto de ceros de una funcion con valores reales 38Problemas 395. Propiedades basicas de la topologıa de una variedad 43Problemas 456. Miscelanea 46

Capıtulo 3. ESPACIO TANGENTE 491. Notaciones previas 49Problemas 502. Derivadas parciales. Propiedades 51Problemas 513. Vectores tangentes 52Problemas 544. Aplicacion tangente 55

iii

Page 4: Apuntes Geom Diferencial

iv INDICE GENERAL

Problemas 58

Capıtulo 4. SUBVARIEDADES Y VARIEDADES COCIENTE 611. Inmersiones 61Problemas 632. Submersiones 68Problemas 703. Las fibras de una submersion 71Problemas 73

Capıtulo 5. GRUPOS DE TRANSFORMACIONES 831. Grupos discontinuos y variedades recubridoras 83Problemas 862. Sistemas dinamicos 892.1. La resolucion de un sistema lineal holonomo 2x2 89Problemas 913. Grupos de Lie 93Problemas 974. Encajes de la botella de Klein y del plano proyectivo real 98

Capıtulo 6. CAMPOS Y FORMAS 991. Fibrado tangente y cotangente 99Problemas 1002. Definicion y propiedades de campos y formas 101Problemas 1073. Variedades paralelizables 109Problemas 1094. Variedades orientables 110Problemas 1145. Curvas integrales 114Problemas 116

Capıtulo 7. VARIEDADES Y CONEXIONES RIEMANNIANAS1231. Conexiones y derivada covariante 123Problemas 1282. Variedades riemannianas 129Problemas 1313. Longitudes de curvas y volumenes 135Problemas 1364. Conexiones riemannianas 136Problemas 1395. Curvatura 144Problemas 1496. Miscelanea 150

Capıtulo 8. EL PAQUETE RIEMANNIAN GEOMETRY 1551. El paquete RiemannianGeometry 155

Page 5: Apuntes Geom Diferencial

INDICE GENERAL v

1.1. Introduccion 1551.2. Algunas pseudometricas mas frecuentes 1561.3. Pseudo-metrica inducida por una inmersion 1561.4. Sımbolos de Christoffel y ecuaciones de las geodesicas 1571.5. Tensor de curvatura y Curvatura Seccional 1581.6. Coeficientes geometricos inducidos por una inmersion 1591.7. Ejemplos 1612. Integracion numerica de las geodesicas 1643. Algunas aplicaciones informaticas para la geometrıa 1664. Otros enlaces interesantes 167

Indice alfabetico 169

Bibliografıa 173

Page 6: Apuntes Geom Diferencial
Page 7: Apuntes Geom Diferencial

INTRODUCCION

Estos son unos apuntes sobre tecnicas basicas de Geometrıa Dife-rencial. Incluyen las nociones mas importantes acompanadas de algunosejemplos y problemas que creemos que facilitan la comprension de lasmismas y resaltan aquellos aspectos mas interesantes. Se abordan algu-nos resultados elementales, se hace referencia a muchos de los teoremasconsiderados como basicos e importantes y se recomienda bibliografıaadecuada para su estudio. Por un lado, intentamos que el lector asi-mile bien las primeras nociones y sus propiedades y, por otra, se hanseleccionado unos contenidos que puedan presentarse y estudiarse enun periodo breve de tiempo.

La Geometrıa Diferencial es una tecnica que, mediante metodos di-ferenciales, da respuesta a numerosos problemas matematicos y ademasse puede completar con las herramientas necesarias para introducir laGeometrıa Riemannniana que es una teorıa que unifica la geometrıaeuclidiana, la llamada geometrıa analıtica, la geometrıa proyectiva y lageometrıa hiperbolica. Tambien deja el camino abierto para introducirla Geometrıa Pseudo-Riemanniana que da un marco adecuado para elestudio de la Teorıa de la Relatividad.

En esta introduccion hacemos unos breves comentarios historicos,senalamos los objetivos que deseamos que el alumno alcance con laayuda de estos apuntes y comentamos los requisitos que a nuestro juiciodebe reunir este para leer sin dificultad estas notas sobre GeometrıaDiferencial.

1. Breves comentarios historicos

La geometrıa de las culturas griega y egipcia quedo recogida demodo magistral en los Elementos de Euclides. Esta excepcional obra sefecha aproximadamente hacia el 300 a.C. y en ella se establecen fun-damentos de geometrıa y algebra griega que van a tener una influenciadecisiva a lo largo de los dos milenios siguientes.

Los Elementos se construyen a partir de unos postulados basicosque describen las propiedades elementales de los puntos y las rectas delplano. Senalaremos que con este tratado se plantea uno de los proble-mas que mas polemica ha causado y para cuya resolucion completa sehan necesitado mas de dos milenios. Se trata simplemente de averiguarsi los postulados son independientes y si uno de ellos, frecuentemente

1

Page 8: Apuntes Geom Diferencial

2 INTRODUCCION

denominado quinto postulado y tambien postulado de las paralelas, de-pende de los demas. En geometrıa euclidiana se verifica que por puntoexterior a una recta pasa una unica recta paralela y va a ser esta pro-piedad la que hace que la resolucion del problema anterior se conozcatambien como la ciencia de las paralelas.

Por brevedad y para centrarnos rapidamente en los aspectos dife-renciales de la geometrıa vamos a dar un gran salto en el tiempo parasituarnos en el siglo XVII. Con ello no queremos que se desprenda queen el periodo intermedio no se dieran importantes avances geometricos.Empezamos esta nueva etapa mencionando a Rene Descartes (1586-1650), eminente cientıfico frances, que en su obra “Geometrıa”, aso-ciaba a cada punto del plano dos coordenadas x, y que le permitieronformular analıticamente numerosos problemas geometricos. Tambienel matematico frances Pierre de Fermat (1601-1665) utilizaba en estaepoca coordenadas rectangulares en dimension dos en su obra “Intro-duccion a la teorıa de lugares planos y espaciales”.

Las tecnicas de la perspectiva eran conocidas desde epocas remo-tas y especialmente fueron desarrolladas por artistas y arquitectos enla epoca del Renacimiento. Gerard Desargues (1593-1662) utilizaba en1636 coordenadas para la construccion de perpectividades. A el se debesu celebre teorema de los triangulos coaxiales y copolares que juega unpapel importante en la descripciones axiomaticas y algebraicas de lasgeometrıas. Mencionaremos tambien a Blaise Pascal (1623-1662) y suteorema del hexagrama mıstico. El uso de perspectividades necesita-ba de puntos infinitamente alejados que poco a poco dieron lugar a lageometrıa proyectiva y sus transformaciones, denominadas frecuente-mente como proyectividades. Destacaremos que dos siglos mas tarde lageometrıa proyectiva ya se habıa desarrollado como se pone de mani-fiesto en la obra de Jean-Victor Poncelet (1788-1867) “Tratado de laspropiedades proyectivas de las figuras” que fue publicada en Paris en1822, si bien fue planificada siendo este prisionero en Moscu a causa delas guerras napoleonicas.

En el siglo XVII se produce un hito matematico importante: el na-cimiento del calculo diferencial, impulsado principalmente por IsaacNewton (1642-1727) en Inglaterra, e independientemente, por Gott-fried Wilhelm Leibniz (1646-1716) en Alemania. Poco a poco el usode tecnicas diferenciales para la resolucion de problemas geometricosiba determinando la disciplina matematica que hoy denominamos comoGeometrıa Diferencial.

A comienzos del siglo XVII, Isaac Newton en su obra de 1704 “Enu-meracion de las curvas de tercer orden” clasifica curvas segun el gradode su ecuacion y encuentra su interpretacion geometrica como el maxi-mo numero posible de puntos de interseccion de la curva con una recta.

Page 9: Apuntes Geom Diferencial

1. BREVES COMENTARIOS HISTORICOS 3

El uso sistematico de coordenadas en dimension tres puede verse yaen 1731 en el libro de Alexis Claude Clairaut (1713-1765) “Investiga-ciones sobre curvas de doble curvatura”. Clairaut interpretaba correc-tamente una superficie como las soluciones de una ecuacion unica y lacurvas en el espacio las consideraba como interseccion de dos superfi-cies; es decir, mediante dos ecuaciones. Son muy conocidos sus resul-tados sobre las propiedades que tienen las geodesicas de una superficiede revolucion (veanse los problemas del capıtulo 7 de estos apuntes). Afinales del siglo XVII Sylvestre Francois Lacroix (1765-1843) acuno ladenominacion de Geometrıa Analıtica.

Leonard Euler (1707-1783) continuo con la investigacion de geodesi-cas sobre superficies, en particular dio la ecuacion diferencial de unageodesica en una superficie (vease seccion 1 del capıtulo 7 de estosapuntes). Analizo tambien la curvatura de las secciones planas de unasuperficie dando expresiones para el calculo de las curvaturas principa-les. En 1771, en un artıculo sobre cuerpos cuyas superficies se puedensuperponer en un plano, Euler introdujo el concepto de superficie desa-rrollable.

Gaspard Monge (1746-1818) en los anos 80 publico dos obras, enlas que estudio propiedades de curvas en el espacio y de las superficies.Introdujo nociones y terminologıa todavıa utilizadas en la actualidad,como las de superficie desarrollable y rectificable, arista de retroceso,lugar geometrico de los centros de curvatura, etc. La traduccion dehechos geometricos en terminos de ecuaciones en derivadas parcialesy ecuaciones diferenciales ordinarias condujo a la geometrıa diferen-cial a una nueva fase en la que se interrelacionaban algunos aspectosgeometricos con otros de la teorıa de ecuaciones diferenciales.

Una nueva etapa de la Geometrıa Diferencial se puso de manifies-to con las investigaciones de Karl Friedrich Gauss (Grunswick, 1777−Gotinga, 1855) sobre la geometrıa intrınseca de las superficies, es de-cir, aquellas propiedades que son invariantes por transformaciones quepreservan las longitudes y los angulos que forman las curvas contenidasen las superficie. Citaremos el llamado Teorema Egregio de Gauss queasegura que la curvatura de una superficie es intrınseca.

A finales de siglo XVIII y principios del XIX aparecen los tres prin-cipales artıfices en la historia de la geometrıa no euclidiana: Gauss,que ya hemos menciondado, Nikolai Ivanovich Lobachevski (Bajo Nov-gorod, actual Gorki, 1792-1856), Janos Bolyai (Kolozsvar, actual Cluj-Napoca, Rumania, 1802-1860). Aunque las cartas de Gauss prueban losgrandes avances realizados por este en la ciencias de las paralelas, elmerito de su descubrimiento se atribuye de modo independiente a Lo-bachevki y a Bolyai. En sus publicaciones ellos desarrollan una nuevageometrıa suponiendo que, al contrario de lo que sucede en la geo-metrıa euclidiana, por un punto exterior a una recta pasa mas de una

Page 10: Apuntes Geom Diferencial

4 INTRODUCCION

paralela. Esta nueva geometrıa se denomina geometrıa no euclidiana ohiperbolica.

A mediados del siglo XIX aparece un nuevo principio general sobreque es lo que se puede entender por una geometrıa. Esta idea fue ex-puesta por Bernhard Riemann (1826-1886, alumno de Gauss) en el ano1854 en una conferencia titulada “Sobre las hipotesis que yacen en losfundamentos de la Geometrıa”; posteriormente, esta conferencia, quese publica en 1887, ha sido frecuentemente citada. Segun Riemann,para la construccion de una Geometrıa es necesario dar: una variedadde elementos, las coordenadas de estos elementos y la ley que mide ladistancia entre elementos de la variedad infinitamente proximos. Paraello se supone que las partes infinitesimales de la variedad se mideneuclidianamente. Esto significa que hay que expresar en su forma masgeneral el elemento de arco en funcion de las coordenadas. Para ellose da el elemento generico de arco para cada punto a traves de unaforma cuadratica definida positiva de la forma ds2 =

∑i,j gijdxidxj .

Eliminado la condicion de ser definida positiva, este modelo tambiense utiliza para formular la teorıa de la relatividad, tanto en su versionrestringida como en la generalizada mediante el uso de variedades dedimension cuatro, tres coordenadas espaciales y una temporal. En es-tos modelos se toman como transformaciones aquellas aplicaciones quedejan invariante el elemento de arco.

Esta forma de concebir el espacio ha evolucionado con el tratamien-to dado a comienzos del siglo XX en los trabajos de los matematicositalianos M. M. G. Ricci (1853-1925) y T. Levi-Civita (1873-1941) has-ta la nocion que hoy denominamos como variedad riemanniana. Ladefinicion que actualmente utilizamos de variedad diferenciable (o di-ferencial) se atribuye a Hassler Whitney [41] que en 1936 presentabauna variedad como una serie de piezas euclidianas pegadas con funcio-nes diferenciables.

Aquellos lectores que deseen conocer otros aspectos historicos deldesarrollo de la geometrıa pueden consultar los libros [43] , [44]. Paraaquellos aspectos relacionados con geometrıas euclidianas y no eucli-dianas consideramos interesante el libro de Bonola [45] y la pagina web4.1 del capıtulo 8.

En estos apuntes se presenta la nocion de variedad diferenciable demanera parecida a como la introdujo Whitney y se estudian las he-rramientas mas basicas para poder introducir de un modo riguroso lanocion de variedad riemanniana. De un modo rapido diremos que eneste texto se analiza la topologıa inducida por una estructura diferen-ciable para despues abordar el estudio de una funcion diferenciable enun punto a traves de espacios y aplicaciones tangentes. Ello permite in-troducir los conceptos de subvariedad, variedad cociente y mediante latecnica de fibrados (co)tangentes y sus secciones: los campos tangentes

Page 11: Apuntes Geom Diferencial

2. OBJETIVOS 5

y las formas diferenciables. Con todas estas nociones estamos en dispo-sicion de introducir la nocion de variedad riemanniana. Exponemos acontinuacion unas ideas introductorias sobre algunas cuestiones basicasde geometrıa riemanniana.

En una variedad riemanniana se dispone de una forma bilineal en elespacio tangente de cada punto de la variedad que permite medir longi-tudes de vectores tangentes y el angulo determinado por dos vectores.Utilizando las tecnicas usuales de integracion se pueden determinar laslongitudes de las curvas de dicha variedad y se puede calcular el vo-lumen de adecuadas “regiones medibles” de la misma. Por otra parte,dados dos puntos de una componente conexa se pueden considerar to-das las curvas que existan en la variedad entre esos dos puntos. Si losdos puntos estan “suficientemente proximos”, entonces existe una cur-va especial que es la que realiza el recorrido entre ellos con la menorlongitud posible. Estas curvas se denominan geodesicas.

Tambien se introduce la nocion de curvatura riemanniana que aso-cia un escalar a cada plano tangente de la variedad y que para el casode superficies coincide con la nocion de curvatura de Gauss. En loscasos en que la curvatura sea cero la variedad riemanniana se dice quees parabolica, si es constante y mayor que cero se dice que es elıptica ysi es constante y negativa se dice que es una variedad hiperbolica. Estemodelo matematico denominado variedad riemanniana contiene la ma-yor parte de los modelos geometricos estudiados hasta el siglo XX. Poruna parte, los espacios euclıdeos se pueden considerar como variedadesriemannianas con curvatura de Riemann nula, las esferas y espaciosproyectivos tienen estructura de variedades riemannianas elıpticas ylos espacios no euclidianos son casos particulares de variedades rie-mannianas hiperbolicas. Ademas, las superficies e hipersuperficies delos espacios euclidianos admiten tambien de modo natural estructurade variedad riemanniana.

Estas propiedades hacen que la geometrıa riemanniana sea un mar-co adecuado para realizar estudios geometricos. No obstante, existenotras formas de formalizar la geometrıa; por ejemplo, a traves de gru-pos discontinuos de transformaciones tal y como la presento Klein ensu famoso programa de Erlangen. Mencionaremos tambien que existenmodelos que generalizan los considerados en este texto y otros basadosen grupos de transformaciones, aunque desde un punto de vista didacti-co nos parece que un nivel de abstraccion muy elevado puede ocultarla verdadera naturaleza de las nociones y problemas geometricos quequeremos presentar y desarrollar en estas notas.

2. Objetivos

Con esta Introduccion a la de Geometrıa Diferencial se pretendenalcanzar las metas siguientes:

Page 12: Apuntes Geom Diferencial

6 INTRODUCCION

1) Que se comprenda la nocion de variedad diferenciable ası comola importancia de sus aplicaciones en otras ciencias.

2) Que se adquieran las herramientas basicas de trabajo, aprendien-do nociones fundamentales tales como inmersion, submersion, campos,formas, etc.

3) Que se conozcan variedades importantes tales como esferas, es-pacios proyectivos, variedades de Grassmann, grupos de Lie, hipersu-perficies de espacios euclıdeos, etc.

4) Que se adquieran los procedimientos basicos para construir nue-vas variedades: productos, espacios de orbitas de grupos de transfor-maciones discontinuos, cubiertas, fibrados, etc.

5) Que se comprenda que existen otras estructuras analogas o“proximas” a la nocion de variedad diferenciable, y que muchas delas tecnicas desarrolladas se pueden trasladar a otros contextos. Porejemplo, variedades de clase Cr, variedades analıticas, variedades consingularidades, variedades con borde, etc.

6) Que se compruebe que la nocion de variedad riemanniana propor-ciona un buen marco para el estudio de muchos aspectos geometricos.

7) Que se valore positivamente el efecto unificador de la geometrıariemanniana al contener como casos particulares, por un lado, las geo-metrıas elıpticas, parabolicas e hiperbolicas y, por otro, la geometrıade curvas y superficies.

8) Que se conozca la tecnica de conexiones y conexiones riemannia-nas y su utilizacion en el estudio de geodesicas y curvaturas.

9) Que el alumno sepa los hitos historicos mas importantes quehan determinado muchas aportaciones mutuas y enriquecedoras entrela geometrıa y el calculo diferencial dando lugar a una disciplina ma-tematica consolidada, llamada Geometrıa Diferencial.

Con este programa, ademas de intentar dar una formacion muybasica sobre algunos aspectos geometricos, se ha intentado seleccionarlos temas mas motivadores. Se ha preferido renunciar a enfoques muygenerales, escogiendo aquellos metodos que ilustran mejor las propie-dades geometricas de las variedades.

3. Requisitos

Se supone que el alumno ha estudiado cursos de analisis matemati-co en los que se han introducido la derivada lineal de funciones devarias variables, derivadas direccionales, derivadas parciales, teoremade la funcion inversa y teorema de la funcion implıcita. Es tambienconveniente que conozca las tecnicas de integracion de funciones deuna y varias variables tanto es sus aspectos teoricos como en los maspracticos.

Page 13: Apuntes Geom Diferencial

3. REQUISITOS 7

Tambien supondemos que el alumno esta familiarizado con los meto-dos de resolucion de sistemas de ecuaciones diferenciales ordinarias, es-pecialmente de las de primer y segundo orden, y de los sistemas de ecua-ciones diferenciales lineales. En estas notas aplicaremos principalmenteestas tecnicas al estudio de geodesicas en una variedad riemanniana yal calculo de curvas integrales de un campo.

Son fundamentales el conocimiento de las nociones y propiedadesbasicas topologicas asi como los procedimientos mas usuales de cons-truccion de espacios topologicos. Respecto a las nociones, principalmen-te senalaremos, un buen conocimiento de las propiedades mas frecuen-tes: primero y segundo numerable, conexion y conexion por caminosy las correspondientes conexiones locales, y tambien la propiedad decompacidad. En cuanto a metodos de construccion, es conveniente queel alumno maneje bien los subespacios, los productos y las topologıascocientes. Estas notas contienen alguna construccion en la que se utilizala nocion de espacio recubridor y sus propiedades, particularmente lasdel recubridor universal. Si bien no es necesario para la lectura de estetexto, sı consideramos recomendable el estudio previo de las propieda-des de los espacios recubridores y su relacion con el grupo fundamental.

Finalmente, aunque tampoco es estrictamente necesario, es conve-niente tambien adquirir una formacion elemental sobre la nocion degrupo; aquı utilizamos algunos grupos de transformaciones, unas vecesse trata de grupos continuos de Lie y otras, de grupos discontinuosde transformaciones que se utilizan para expresar una variedad comocociente de otra mas conocida. Es tambien de interes el conocimientode la nocion de modulo que utilizamos para el estudio de todos loscampos tangentes y formas globales de una variedad sobre el anillo delas funciones globales reales. Destacaremos que los espacios vectoria-les reales y sus corresponcientes aplicaciones lineales con las nocionesasociadas de dimension y rango que son herramienas muy importantespara el analisis de funciones diferenciables, y es por ello necesario queel alumno haya adquirido la destreza suficiente en su uso.

Page 14: Apuntes Geom Diferencial
Page 15: Apuntes Geom Diferencial

CAPıTULO 1

PRELIMINARES

En este capıtulo hacemos un breve recorrido a traves de nocionestopologicas, algebraicas y de analisis matematico. Se introduce la no-tacion y algunas propiedades basicas que se van a utilizar en el restodel libro.

1. Nociones y notaciones asociadas a funciones

Presentamos a continuacion algunas nociones relacionadas con elconcepto de funcion.

Definicion 1.1. Una funcion ϕ : X → Y con dominio Dom ϕ ⊂ Xes una correspondencia que asocia a cada p ∈ Dom ϕ un unico elementoϕ(p) ∈ Y . El conjunto ϕ(p)|p ∈ Dom ϕ lo llamaremos conjuntoimagen, Im ϕ , tambien se denomina rango de la funcion o codominiode la funcion, Codom ϕ . Diremos que X es el conjunto inicial y queY es el conjunto final de ϕ .

Dada una funcion ϕ : X → Y si A es un subconjunto de X deno-taremos por ϕ|A : X → Y la funcion cuyo dominio es el subconjuntoDom (ϕ|A) = A∩Dom ϕ y para cada p ∈ A ∩Dom ϕ , ϕ|A(p) = ϕ(p) .Notemos que Codom (ϕ|A) = ϕ(p)|p ∈ A ∩ Domϕ . Diremos queϕ|A : X → Y es la restriccion de ϕ a A .

Si ϕ : X → Y es una funcion y A es un subconjunto de X llama-remos imagen de A al subconjunto ϕ(A) = ϕ(p)|p ∈ A ∩ Domϕ .Cuando B es un subconjunto de Y , denotaremos por ϕ−1B = p ∈X|ϕ(p) ∈ B y lo llamaremos imagen inversa de B . En el casoparticular que B = b la imagen inversa de b la llamaremos fi-bra de b , con frecuencia en vez de ϕ−1b utilizaremos la notacionϕ−1(b) . Dadas dos funciones ϕ : X → Y , ψ : Y → Z se define lafuncion composicion ψϕ : X → Z como aquella que tiene como domi-nio Dom (ψϕ) = ϕ−1( Dom ψ) y esta definida por ψϕ(p) = ψ(ϕ(p)) .Notese que Codom(ψϕ) = ψ(Codomϕ) y Dom(ψϕ) = ϕ−1(Domψ) .

Sea ϕ : X → Y una funcion. Se dice que ϕ : X → Y es inyectivacuando se verifica que si ϕ(p) = ϕ(q) , entonces p = q . Dada unafuncion inyectiva ϕ : X → Y , entonces podemos considerar la funcioninversa ϕ−1 : Y → X , que tiene por dominio Dom (ϕ−1) = Codom ϕ ,y que verifica que ϕ−1(y) = x si ϕ(x) = y, x ∈ Domϕ , y ∈ Codomϕ .Una funcion ϕ : X → Y es suprayectiva (exhaustiva o sobreyectiva) si

9

Page 16: Apuntes Geom Diferencial

10 1. PRELIMINARES

Codomϕ = Y . En este caso tambien se dice que ϕ es una funcion de Xsobre Y . Diremos que una funcion ϕ : X → Y es global si Domϕ = X .

Recordemos que una aplicacion f : X → Y es una correspondenciaque asocia a cada punto p del conjunto X un unico punto f(p) delconjunto Y . Es importante observar que una funcion ϕ : X → Y esaplicacion si y solo si ϕ es global.

2. Topologıa

En esta seccion se introducen algunos conceptos basicos de Topo-logıa, no obstante, es conveniente que el lector conozca previamente lasnociones basicas de Topologıa para poder seguir sin dificultades estasnotas.

2.1. Espacios topologicos y funciones continuas.

Definicion 2.1. Sea X un conjunto y sea τ una familia no vaciade subconjuntos de X . Se dice que es una topologıa si τ es cerradapor uniones de subfamilias arbitrarias (la union de la subfamilia vaciaes el subconjunto vacio) y por interseccion de subfamilias finitas (lainterseccion de la subfamilia vacia es el conjunto total X). Llamare-mos espacio topologico a un par (X, τ) donde τ es una topologıa en elconjunto X . Normalmente acortaremos la notacion de modo que X de-notara tanto al par (X, τ) como al conjunto subyacente. Los miembrosde la topologıa τ diremos que son los abiertos del espacio X .

Notemos que dado un espacio topologico el propio X y el subcon-junto vacio ∅ son abiertos.

Ejemplo 2.1. Dado un conjuntoX se puede considerar la topologıatrivial τtri = X, ∅ y la topologıa discreta τdis formada por todos lossubconjuntos de X .

Definicion 2.2. Sean X, Y espacios topologicos. Se dice que unaaplicacion f : X → Y es continua si para todo abierto V de Y setiene que f−1V es abierto en X . Diremos que X, Y son homeomorfossi existen aplicaciones continuas ϕ : X → Y y ψ : X → Y tales queψϕ = idX , ϕψ = idY , donde idX , idY denotan las correspondientesaplicaciones identidad.

Ejemplo 2.2. Sea (X, τ) un espacio topologico y A ⊂ X un sub-conjunto de X . La familia τ/A = A∩U |U ∈ τ es un topologıa sobreel conjunto A que se llama la topologıa relativa de X en A . Es facilcomprobar que la inclusion in : A→ X es una aplicacion continua.

Definicion 2.3. Sean X, Y espacios topologicos. Diremos que unafuncion f : X → Y es continua si la aplicacion f : Dom f → Y , dadapor f(x) = f(x), x ∈ Dom f , es continua, donde en Dom f se considerala topologıa relativa de X en Dom f . Diremos que una funcion f : X →

Page 17: Apuntes Geom Diferencial

2. TOPOLOGIA 11

Y es un homeomorfismo si f es inyectiva y las funciones f y f−1 soncontinuas.

Proposicion 2.1. Sea f : X → Y una funcion entre espacios to-pologicos. Supongamos que ademas tenemos que Dom f es un abiertode X. Entonces la funcion f es continua si y solo si para todo abiertoV de Y se tiene que f−1(V ) es un abierto de X .

Definicion 2.4. Sea X un espacio topologico. Diremos que F esun cerrado si X \ F es un abierto de X . Sea N un subconjunto de Xy p ∈ N . Se dice que N es un entorno de p si existe un abierto U talque p ∈ U ⊂ N . Dado un subconjunto A de X llamaremos interior deA a la reunion de los abiertos contenidos en A . Llamaremos clausurade A que denotaremos por clA a la interseccion de los cerrados quecontienen a A .

2.2. Base de una topologıa.

Definicion 2.5. Sea τ la topologıa de un espacio topologico X .Se dice que una subfamilia B ⊂ τ es una base de la topologıa τ si paracada abierto U existe una subfamilia BU ⊂ B tal que U = ∪B∈BUB .

Proposicion 2.2. Sea X un conjunto y sea B una familia de sub-conjuntos de X . Si la familia de subconjuntos B verifica las siguientespropiedades

(i) X = ∪B∈BB ,(ii) Si p ∈ B ∩ B′ , B,B′ ∈ B , entonces existe B′′ ∈ B tal que

p ∈ B′′ ⊂ B ∩B′

Entonces existe una unica topologıa τ tal que B es una base de τ .

Definicion 2.6. Sea X un espacio topologico y p ∈ X . Una familiade entornos Bp es una base de entornos de p si para cada entorno N dep existe B ∈ Bp tal que p ∈ B ⊂ N .

Diremos que un conjunto es contable si su cardinalidad es finita oes la del conjunto de los numeros naturales.

Definicion 2.7. Sea X un espacio topologico. Diremos que X esprimero contable si para cada punto p ∈ X existe una base de entornoscontable. Se dice que X es segundo contable la topologıa tiene una basecontable.

Ejemplo 2.3. Denotemos por R el conjunto ordenado de los nume-ros reales y consideremos la familia de los intervalos abiertos acotadosB = (a, b)|a, b ∈ R a < b , donde (a, b) = r ∈ R|a < r < b .Entonces B satisface las propiedades (i) y(ii) de la proposicion anteriory determina una unica topologıa en R , que diremos que es la topologıahabitual o usual. Consideraremos tambien con frecuencia el subespa-cio que llamaremos intervalo unidad I = r ∈ R|0 ≤ r ≤ 1 con latopologıa relativa inducida por la topologıa usual de R .

Page 18: Apuntes Geom Diferencial

12 1. PRELIMINARES

Ejemplo 2.4. Denotemos por C el conjunto de los numeros com-plejos. Dado un numero complejo z su conjugado se denota por zy su modulo por |z| = (zz)

12 . Consideremos la familia bolas B =

B(a, r)|a ∈ C , r ∈ R , r > 0 , donde B(a, r) = z ∈ C||z − a| < r .Entonces B satisface las propiedades (i) y(ii) de la proposicion anteriory determina una unica topologıa en C .

2.3. Propiedades topologicas. Ademas de las propiedades deprimero contable y segundo contable es muy frecuente el uso de lassiguientes propiedades topologicas.

Definicion 2.8. Se dice que un espacio topologico X es T0 si paracada pareja de puntos p, q ∈ X , p 6= q existe U entorno de p tal queq 6∈ U o existe V entorno de q tal que p 6∈ V . Diremos que X es T1 sipara cada pareja de puntos p, q ∈ X , p 6= q existe U entorno de p talque q 6∈ U . Se dice que un espacio topologico X es T2 o Hausdorff sipara cada pareja de puntos p, q ∈ X , p 6= q existe U entorno de p yexiste existe V entorno de q tal que U ∩ V = ∅ .

Es facil ver que si X es T2 entonces es T1 y que si X es T1 entonceses T0 .

Definicion 2.9. Se dice que espacio topologico X es conexo sisiempre que X = U ∪ V con U ∩ V = ∅ , entonces U = ∅ o V = ∅ .Diremos que un espacio topologico X es localmente conexo si cadapunto del espacio tiene una base entornos conexos.

Definicion 2.10. Se dice que espacio topologico X es conexo porcaminos si siempre que x, y ∈ X , entonces existe una aplicacion con-tinua f : I → X tal que f(0) = x y f(1) = y . Diremos que un espaciotopologico X es localmente conexo por caminos si cada punto del espa-cio tiene una base entornos conexos por caminos.

Es bien conocido que la conectividad por caminos implica conecti-vidad.

2.4. Construcciones. Dada una familia de espacios Xα , α ∈ Aconsideraremos la suma disjunta (coproducto)

∑α∈AXα al conjunto⋃

α∈AXα × α con la topologıa inducida por la base

B = U × α|U es abierto en Xα , α ∈ AEn el caso que la familia verifique que estos espacios son disjuntos dos ados en vez de tomar la reunion anterior, para tener una mayor facilidaden la notacion, se suele tomar directamente la reunion

⋃α∈AXα y con-

secuentemente la base B = U |U es abierto en Xα para algunα ∈ A .En el caso de familias finitas, por ejemplo dados dos espacios X1 , X2 ,es frecuente usar las notaciones

⊔i∈1,2Xi o tambien X1

⊔X2 .

Otra construccion frecuente asociada a una familia de espacios Xα

α ∈ A es el producto∏

α∈AXα que tiene como soporte el producto

Page 19: Apuntes Geom Diferencial

2. TOPOLOGIA 13

cartesiano y si prα denota las proyecciones canonicas se considera latopologıa producto inducida por la base

B = ⋂

i∈1,··· ,n

pr−1αi

(Uαi)|Uαi es abierto en Xαi , α1, · · · , αn ∈ A

En el caso de familias finitas, por ejemplo dados dos espacios X1 , X2 ,es frecuente usar la notacion X1 ×X2 .

Finalizamos el proceso de construccion de nuevos espacios topologi-cos a partir de otros dados con la topologıa cociente. Sea X un espaciotopologico y sea f : X → Y una aplicacion exhaustiva de X en unconjunto Y . La topologıa cociente en el conjunto Y es la formada poraquellos subconjuntos V de Y tales que f−1V es un abierto de X . Esinteresante recordar que si en Y se considera la topologıa cociente yg : Y → Z es una aplicacion entre espacios topologicos, entonces g escontinua si y solo si gf es continua.

2.5. Algunos espacios. Los siguientes espacios seran utilizadoscon frecuencia en estas notas.

Ejemplo 2.5. Denotemos por Rn el conjunto de n-tuplas de nume-ros reales r = (r1, · · · , rn) . Se considera en Rn la topologıa productoque tiene las siguientes propiedades: Es segundo contable, Hausdorff,conexa por caminos y localmente conexa por caminos.

Ejemplo 2.6. Para n ≥ 0 , la n-esfera unidad Sn se define comoel subespacio

Sn = r ∈ Rn+1|r21 + · · ·+ r2

n+1 = 1

donde hemos considerado la topologıa relativa.

Ejemplo 2.7. Para n ≥ 0 , el n-disco unidad Dn se define como elsubespacio

Dn = r ∈ Rn|r21 + · · ·+ r2

n ≤ 1con la topologıa relativa. La bola unidad la denotaremos por

Bn = r ∈ Rn|r21 + · · ·+ r2

n < 1

Ejemplo 2.8. Para n ≥ 0 , el n-espacio proyectivo real P n(R) sedefine como el cociente obtenido de la n-esfera Sn identificando puntosantıpodas y considerando la topologıa cociente.

Ejemplo 2.9. Una metrica en un conjunto X es una aplicaciond : X ×X → R tal que safisface las siguientes propiedades:

(M1) Para x, y ∈ X, d(x, y) ≥ 0 ; ademas, d(x, y) = 0 si y solo six = y .

(M2) Si x, y ∈ X , se tiene que d(x, y) = d(y, x) .(M3) Sean x, y, z ∈ X . Entonces d(x, z) ≤ d(x, y) + d(y, z) .

Page 20: Apuntes Geom Diferencial

14 1. PRELIMINARES

La bola de centro a ∈ X y radio ε > 0 se define como el subconjunto

Bd(a, ε) = x ∈ X|d(a, x) < ry el disco

Dd(a, ε) = x ∈ X|d(a, x) ≤ rSi en el contexto de trabajo la metrica esta bien determinada o setrata de la metrica usual eliminaremos el subındice en las notacionesde discos y bolas.

Un espacio metrico consiste en un conjuntoX junto con una metricad . En un espacio metrico (X, d) se puede considerar la siguiente familiade subconjuntos:

τ = U ⊂ X|Six ∈ U, existe ε > 0 tal queB(x, ε) ⊂ USe prueba que τ es una topologıa, que diremos que esta inducida

por la metrica d . Un espacio topologico X se dice metrizable si sutopologıa τX esta inducida por una metrica.

Ejemplo 2.10. En Rn se pueden considerar la metrica euclidiana,de modo que si r, s ∈ Rn, r = (r1, · · · , rn), s = (s1, · · · , sn) la aplicaciond viene dada por

d(r, s) =( n∑i=1

(ri − si)2) 1

2

La topologıa inducida por la metrica euclidiana, es precisamente lahemos considerado en el ejemplo 3.1 . Por ser la metrica mas usual enla notacion de bolas y discos suprimiremos el subındice

B(a, ε) = r ∈ Rn|( n∑i=1

(ri − ai)2) 1

2 < ε

y el disco

D(a, ε) = r ∈ Rn|( n∑i=1

(ri − ai)2) 1

2 ≤ ε

Tambien es frecuente la metrica cartesiana, ρ , dada por

ρ(r, s) = max|ri − si|i ∈ 1, · · · , nLa topologıa inducida es tambien la del ejemplo 3.1 . Las bolas y

discos se denotaran por Bρ(a, ε), Dρ(a, ε) .

3. Algebra lineal

Introducimos a continuacion la nocion de grupo abeliano y la deespacio vectorial

Definicion 3.1. Sea V un conjunto y una aplicacion +: V ×V →V , que denotamos por +(a, b) = a+ b . Diremos que la pareja anteriores un grupo abeliano si se verifican las siguientes propiedades:

(i) Asociatividad de la suma: (a+ b) + c = a+ (b+ c) .

Page 21: Apuntes Geom Diferencial

3. ALGEBRA LINEAL 15

(ii) Conmutatividad de la suma: a+ b = b+ a .(iii) Existencia de neutro aditivo: Existe 0 ∈ V tal que a+ 0 = a .iv) Existencia de opuesto aditivo: Para cada a ∈ V existe un a′ ∈

V tal que a+ a′ = 0 .

Definicion 3.2. Un espacio vectorial real es un grupo abeliano Vcon su operacion suma (a, b) 7→ a + b, de V × V en V y que esta pro-visto de una accion (λ, a) 7→ λ · b, de R × V en V , con las siguientespropiedades:

(i) Accion trivial de 1: 1 · a = a .(ii) Asociatividad de la accion: λ · (µ · a) = (λµ) · a .(iii) Distributividad de la accion respecto a la suma:

λ · (a+ b) = λ · a+ λ · b .

(iv) Distributividad de la suma respecto a la accion:

(λ+ µ) · a = λ · a+ µ · a .

Ejemplo 3.1. El espacio vectorial mas importante para el desarro-llo de estos apuntes es Rm, donde la suma y la accion que se definenpara cada coordenada estan inducidas por la suma y el producto delanillo de division de los numeros reales. Otro ejemplo basico lo consti-tuyen el espacio de las matrices reales de n filas y m columnas (isomorfoa Rn×m) con la suma y la accion definidas tambien coordenada a coor-denada.

Una sucesion ordenada B = (v1, . . . , vn) de vectores ; es decir, deelementos en un espacio vectorial V , es una base si cada vector v de Vpuede expresarse de una unica manera como

v =n∑i=1

λivi .

Si este es el caso, decimos que los escalares λ1, . . . , λn son las coordena-das de v en la base B . Se puede probar que dos bases tienen el mismocardinal. Llamaremos dimension, dim(V ), de un espacio vectorial V alcardinal de una de sus bases.

Definicion 3.3. Una aplicacion lineal de un espacio vectorial Ven otro W es una aplicacion L : V → W que verifica:

L(λa+ µb) = λL(a) + µL(b) .

Teniendo en cuenta que hay una correspondencia biunıvoca entreRn y las matrices reales de n filas y 1 columna. Podemos denotar unvector de Rn mediante un vector columna r.

Si A es una matriz de n filas y m columnas (una matriz n×m), laaplicacion

LA : Rm → Rn , LA(r) = Ar

Page 22: Apuntes Geom Diferencial

16 1. PRELIMINARES

es una aplicacion lineal, donde r es el vector columna (matriz de unacolumna) que representa un vector de Rn . Recıprocamente, si L : Rm →Rn es una aplicacion lineal, podemos formar la matriz AL cuya i-esimacolumna es el vector columna L(ei); es decir, la imagen por L del i-esimo vector canonico de Rn colocado en forma de columna.

En la definicion de LA estamos usando el producto de matrices enel caso particular de una matriz de n×m por un vector columna quees una matriz m× 1. En general si A es una matriz de n×m y B otrade m× l, se define el producto C = AB como la matriz de n× l cuyacoordenada en la fila i columna j es:

cij =m∑h=1

aihbhj.

Ası que el producto Ar es un vector columna cuya coordenda i-esimaes

m∑h=1

aihrh,

es decir, coincide con el producto de la i-esima fila de A con r.

Definicion 3.4. Sea una aplicacion lineal L de un espacio vectorialreal V en otro W . Llamaremos nucleo de L al subespacio Ker = v ∈V |L(v) = 0 . Como en el caso de funciones Im(L) = L(v)|v ∈ V denotara el conjunto imagen.

Proposicion 3.1. Sea una aplicacion lineal L de un espacio vec-torial real V en otro W

(i) Existe un isomorfismo lineal canonico V/Ker(L) ∼= Im(L)(ii) Si V y W son de dimension finita se tiene que

dim(V ) = dim(Ker(L)) + dim(Im(L)).

(iii) Si V y W son de dimension finita se tiene que

dim(Im(L)) ≤ maxdim(V ), dim(W ).

Definicion 3.5. Sea una aplicacion lineal L de un espacio vectorialreal V en otro W . Llamaremos rango de L a dim(Im(L)) .

Definicion 3.6. Sea una aplicacion lineal L de un espacio vectorialreal V en otro W . Diremos que L es un monomorfismo si L es inyectivay L es un epimorfismo si L es suprayectiva.

Proposicion 3.2. Sea una aplicacion lineal L de un espacio vec-torial real V en otro W , ambos de dimension finita.

(i) Si el rango de L es igual a dim(V ), entonces L es un mono-morfismo.

(ii) Si el rango de L es igual a dim(W ), entonces L es un epimor-fismo.

Page 23: Apuntes Geom Diferencial

4. ANALISIS MATEMaTICO 17

4. Analisis matematico

En esta seccion recordamos algunas de las nociones basicas de anali-sis matematico que se van a utilizar en estos apuntes. En primer lugaranalizamos aquellas funciones que se pueden aproximar en un punto desu dominio por una aplicacion lineal.

En estas notas hemos utlizado la palabra diferenciable para denomi-nar a la funciones de clase C∞ , sin embargo es tambien muy frecuenteel uso de la misma palabra “diferenciable” para designar a las funcio-nes que se pueden aproximar en cada punto de su dominio por unaaplicacion lineal. Para evitar confusiones en este texto se utilizara eltermino derivable para designar el ultimo concepto; es decir, para de-nominar a las funciones que se pueden aproximar en cada punto poruna aplicacion lineal.

Definicion 4.1. Una funcion f de Rm en Rn es derivable en unpunto r0 si existe un entorno abierto U de r0 tal que U ⊂ Dom f yexiste una transformacion lineal L : Rm → Rn tal que

lıms→0

(f(r0 + s)− f(r0)− L(s)

‖s‖

)= 0

Ademas, cuando este sea el caso, la transformacion lineal L se llamala derivada de la funcion en el punto r0 y la denotarermos por Dfr0 .Si una funcion f es derivable en todos los puntos de su dominio diremosque es derivable.

Las siguientes propiedades de la derivacion son bien conocidas, re-ferimos al lector a textos tales como [1, 14] .

Proposicion 4.1. Supongamos que f es derivable en r0, entoncesf es continua en r0 .

Observacion 4.1. Toda transformacion lineal L : Rm → Rn esderivable en cada punto r0 ∈ Rm y ademas DLr0 = L.

Teorema 4.1. (Regla de la cadena) Supongamos que la funcion fde Rl en Rm es derivable en un punto r0 ∈ Rl y que la funcion g deRm en Rn es derivable en s0 = f(r0) ∈ Rm . Entonces h = g f esderivable en r0 y ademas Dhr0 = Dgs0 Dfr0

Definicion 4.2. Dado el espacio Rn la proyecciones canonicas sedefinen como las aplicaciones ri : Rn → R , 1 ≥ i ≥ n ,

ri(a1, · · · , ai, · · · , an) = ai .

Dada una funcion f : Rm → Rn sus componentes son las funcionesf1 = r1f , . . . ,fn = rnf .

Si f : Rm → Rn es una funcion, entonces f es de la forma (f1, . . . , fn),donde la funcion fi : Rm → R es la i-esima componente de f , es decir,fi = ri f , donde ri : Rm → R es la i-esima proyeccion canonica. Conestas notaciones podemos enunciar la siguiente

Page 24: Apuntes Geom Diferencial

18 1. PRELIMINARES

Proposicion 4.2. Para que una funcion f : Rm → Rn sea derivableen un punto r0 ∈ Rm es necesario y suficiente que cada una de suscomponentes fi sea derivable en el mismo punto. Ademas, si este es elcaso, se tiene que (Df)r0 = ((Df1)r0 , . . . , (Dfn)r0) .

Dadas las funciones f : Rk → Rm y g : Rl → Rn , la funcion pro-ducto f × g se define del siguiente modo (f × g)(r, s) = (f(r), g(s) .

Proposicion 4.3. Si dos funciones f y g son derivables respecti-vamente en r0 y s0, entonces su producto f × g resulta derivable en(r0, s0) y ademas

D(f × g)(r0,s0) = Dfr0 ×Dgs0 .

Es decir, la derivada del producto es el producto de las derivadas.

La derivada Dfr0 da una aproximacion lineal de la funcion f , demodo que Dfr0(r− r0) aproxima el valor f(r)−f(r0) . Esta aproxima-cion lineal solo depende de los valores de f en las proximidades de r0 .En el caso de dimension superior a 1, puede aproximarse a un puntopor caminos totalmente diferentes. Por ejemplo, en el caso de dos va-riables, podemos aproximarnos al punto (a, b) por las rectas paralelasa los ejes coordenados, es decir, por puntos de la forma (a + ξ, b) conξ → 0 o de la forma (a, b+ ξ) con ξ → 0. Pero tambien puede acercar-se siguiendo la direccion de otro vector (u, v) por puntos de la forma(a+ ξu, b+ ξv) donde ξ → 0 . Incluso estas formas de acercarse a (a, b)no agotan todas las posiblidades porque es posible seguir la trayectoriade una red, y en particular una curva, que termine en (a, b) . Estasobservaciones nos van a conducir a la nocion de derivadas parciales yderivadas direccionales y a estudiar sus relaciones con la derivada.

Definicion 4.3. Supongamos que f : Rn → R es una funcion defi-nida en un entorno de r0 ∈ Rm . Dado un vector u , decimos que f esderivable en r0 en la direccion de u si existe el lımite

lımξ→0

f(r0 + ξu)− f(r0)

ξ

En tal caso escribiremos (Duf)r0 = lımξ→0f(r0+ξu)−f(r0)

ξy diremos que

es la derivada direccional de f en la direccion u en el punto r0 .

Proposicion 4.4. Si f : Rn → R es derivable en r0, entonces esderivable en cuaquier direccion u y ademas (Duf)r0 = Dfr0(u)

Sea ei = (0, . . . , 0, 1, 0, . . . , 0) es i-esimo vector de la base canonicade Rm , definimos la derivada parcial de una funcion en r0 del siguientemodo:

Definicion 4.4. Si f : Rm → R es una funcion definida en unentorno de un punto r0 ∈ Rm, decimos que f es derivable en r0 con

Page 25: Apuntes Geom Diferencial

4. ANALISIS MATEMaTICO 19

respecto a la i-esima variable si es derivable en r0 en la direccion deli-esimo vector canonico ei. En ese caso escribimos(

∂f

∂ri

)r0

= (Deif)r0

y la llamaremos la i-esima derivada parcial de f en r0. Diremos queexiste la i-esima derivada parcial de f si existe en cada punto de sudominio.

Notemos que la i-esima derivada parcial de f en r0 se puede expresartambien como(

∂f

∂ri

)r0

= lımξ→0

f(r01, · · · , r0

i + ξ, · · · , r0m)− f(r0

1, · · · , r0i , · · · , r0

m)

ξ

Observemos que cuando existe la i-esima derivada parcial de f ve-

rifica que Dom(∂f∂ri

)= Dom f .

En la proposicion siguiente se analiza como estan relacionadas lasderivadas parciales con la derivada de una funcion.

Proposicion 4.5. Sea f : Rm → Rn es derivable en r0 , y conside-remos (f1, . . . , fn) sus componentes fi = rif . Si tomamos las derivadasparciales (

∂fi∂rj

)r0

= (Dejfi)r0 = (Dfi)r0(ej)

se tiene

ri(Df)r0(u) =m∑

1=1

(∂fi∂rj

)r0uj

donde u = (u1, . . . , um) =∑m

i=1 uiei .

El siguiente resultado permite determinar la derivabilidad de unafuncion a partir de la existencia y continuidad de las derivadas parciales.

Teorema 4.2. Supongamos que f : Rm → R es una funcion defini-da en un conjunto abierto A ⊆ Rm. Una condicion suficiente para quef sea derivable en cada punto de A es que todas las derivadas parciales∂f/∂ri esten definidas en A y que al menos n−1 de ellas sean continuasen A .

Definicion 4.5. Se dice que una funcion f : Rm → R es de claseCk en un punto r0 ∈ Dom f si existe un entorno abierto V de r0 talque V ⊂Dom f y f tiene en V y en todos los ordenes menores o igualesque k todas las derivadas parciales

∂i1+···+imf

(∂r1)i1· · ·(∂rm)im(ik ≥ 0, i1 + · · ·+ im ≤ k)

Page 26: Apuntes Geom Diferencial

20 1. PRELIMINARES

y estas son continuas en V . Se dice que f es de clase Ck si f es de claseCk en todos los puntos r0 de Dom f . Notese que f es de clase C0 si ysolo si es continua y tiene dominio abierto.

Definicion 4.6. Se dice que una funcion f : Rm → R es de claseC∞ en un punto r0 ∈ Dom f si existe un entorno abierto V de r0 talque V ⊂Dom f y f tiene en V y en todos los ordenes posibles todaslas derivadas parciales y estas son continuas en V . Se dice que f es declase C∞ si f es de clase C∞ en todos los puntos r0 de Dom f .

Definicion 4.7. Una funcion f : Rm → R de clase C∞ se dice quees analıtica real en un punto r0 ∈ Dom f si existe un entorno abiertoV de r0 tal que V ⊂ Dom f y f |V coincide con su desarrollo de Taylorque converge en V . Se dice que f es analıtica real si lo es en cada puntode su dominio.

Definicion 4.8. Se dice que una funcion f : Rm → Rn es de claseC∞ en un punto r0 ∈ Dom f si lo son cada una de sus componentesf1, . . . , fm : Rm → R . Se dice que f es de clase C∞ si f es de clase C∞

en todos los puntos r0 de Dom f . Similarmente se definen las funcionesde Rm en Rn de clase Ck y las analıticas reales.

Observacion 4.2. Una funcion analıtica real es de clase C∞ y estasultimas son de clase Ck . Las funciones de clase Ck son continuas.

Utilizaremos con frecuencia las siguientes propiedades de las fun-ciones C∞ .

Proposicion 4.6. (i) Si f : Rm → Rn es una funcion C∞ enun punto r0 ∈Dom f , entonces f es continua en r0 .

(ii) Sea f : Rl → Rm una funcion C∞ en un punto r0 ∈Dom f ysea g : Rm → Rn otra funcion C∞ en el punto f(r0) . Entoncesgf es C∞ en el punto r0 .

(iii) Sean f, g : Rm → Rn funciones tales que Dom f ⊂ Dom g yg|Dom f = f . Si r0 ∈ Dom f y f es C∞ en r0 , entonces g esC∞ en r0 .

(iv) Sea f : Rm → Rn una funcion C∞ y supongamos que un abier-to U ⊂ Dom f . Entonces f |U es C∞ .

(v) Si f : Rm → Rn es una funcion C∞ , entonces Dom f es unabierto de Rn .

Problemas

4.1. Sean f, g : Rm → R funciones. Demostrar que si f y g sonC∞ , entonces f + g , f · g son C∞ . La funciones f + g, f · g : Rn → Rtienen como dominio Dom f ∩Dom g y vienen dadas por (f + g)(r) =f(r) + g(r) , (f · g)(r) = f(r)g(r) , para r ∈ Dom f ∩ Dom g . Si elcontexto no da lugar a equıvocos el punto que denota el producto defunciones se suele suprimir.

Page 27: Apuntes Geom Diferencial

NOCIONES PREVIAS Y NOTACIONES 21

4.2. Sea g : Rm → R funcion tal que g(r) 6= 0 para r ∈ Dom g.Demostrar que si g es C∞ , entonces 1

ges C∞ . Probar que si ademas

f : Rn → R es C∞ , entonces fg

es una funcion C∞ (Nota: La funcionfg

tiene por dominio Dom f ∩ Dom g y viene dada por (fg)(r) = f(r)

g(r),

para r ∈ Dom f ∩Dom g).

4.3. Demostrar que una funcion polinomica

P =n∑k=0

∑i1+···+im=k

ai1···imri11 · · · rimm

es una funcion C∞ . Sean P,Q aplicaciones polinomicas, probar que lafuncion racional P

Q: Rm −→ R es una funcion C∞.

Solucion: Es conocido que las proyecciones ri : Rm → R , parai ∈ 1, · · · ,m , y las aplicaciones constantes a : Rm → R son C∞ .Aplicando el problema 4.1 se obtiene que las aplicaciones polinomicasson C∞ . Como consecuencia del problema 4.2 se sigue que las funcionesracionales son C∞ .

4.4. Demostrar que la funcion f : R −→ R tal que f(r) = r12 no es

C∞ en el 0 . Probar que f |(0,∞) es una funcion C∞.

4.5. Comprobar que las funciones f, g : R −→ R dadas por f(r) =

r13 y g(r) = r3 sirven de ejemplo para demostrar la falsedad de la

siguiente afirmacion: “Dadas dos funciones f y g, tales que gf y g sonde clase C∞ en r y f(r) respectivamente, entonces f es de clase C∞ enr”.

Solucion: Notemos que f no es de clase C1 en el punto r = 0 .En efecto df

dr= 1

3r−23 ni esta definida ni admite una extension continua

para r = 0 . En este caso se tiene que gf = idR y g es una aplicacionpolinomica, por lo que ambas son de clase C∞ . Luego gf es de claseC∞ en 0 y g es de clase C∞ en f(0) = 0 . Sin embargo f no es de claseC∞ en 0 .

Page 28: Apuntes Geom Diferencial
Page 29: Apuntes Geom Diferencial

CAPıTULO 2

VARIEDADES DIFERENCIABLES

En este capıtulo, introducimos la nocion de variedad diferenciable yfuncion diferenciable, tambien vemos que una variedad determina unatopologıa en el correspondiente conjunto subyacente y estudiamos laspropiedades basicas de las estructuras diferenciable y topologica de unavariedad.

1. La nocion de variedad diferenciable

Segun Riemann para dar una geometrıa necesitamos un conjuntode puntos y una correspondencia que asocie a cada punto unas coor-denadas. Esta idea se puede modelar a traves de la nocion de carta:

Definicion 1.1. Sea M un conjunto cualquiera, una funcionx : M → Rm inyectiva con rango abierto se llama carta m-dimensional.Si tomamos las proyecciones canonicas, ri : Rm → R , a la composicionxi = rix : M → R se le llama la i-esima coordenada de la carta.

Observese que en el estudio de curvas y superficies en R3 se utilizanfunciones con valores vectoriales cuyos dominios son abiertos de R paralas curvas y abiertos de R2 para las superficies. Estas funciones se suelenllamar parametrizaciones y corresponden a las funciones inversas de lascartas consideradas en la definicion anterior.

Definicion 1.2. Sea M un conjunto cualquiera. Una coleccionA = xα|xα carta m-dimensional sobre el conjunto M es denominadaatlas diferenciable m-dimensional de M si se satisfacen las siguientescondiciones:

(i) La reunion de los dominios de las cartas es M .(ii) Para cada pareja de cartas xα, xβ de A la funcion xβx

−1α es de

clase C∞ .

Definicion 1.3. Sea M un conjunto cualquiera. Dos atlas A , Bdiferenciables y m-dimensionales de M son compatibles si A∪ B es unatlas diferenciable m-dimensional de M .

Proposicion 1.1. La relacion de compatibilidad en la familia de losaltas diferenciables m-dimensionales de un conjunto M es una relacionde equivalencia.

Demostracion. La propiedades reflexiva y simetrica se verificanfacilmente. Para comprobar la transitividad supongamos que A,B, C

23

Page 30: Apuntes Geom Diferencial

24 2. VARIEDADES DIFERENCIABLES

son atlas diferenciables de M de modo que A,B son compatibles y B, Ctambien lo son. Puesto que tanto A como C son atlas se verifica quela reunion de los dominios de A ∪ C es M . Para ver que se verifica lapropiedad (ii), sean xα carta de A y xγ carta de C . Veamos que xγx

−1α

es C∞ en cada uno de los puntos de su dominio. Sea r0 = xα(p0) conp0 ∈ Domxγ y tomemos una carta xβ en el punto p0 . Notemos que(xγx

−1β )(xβx

−1α ) es C∞ y su dominio U es un abierto tal que r0 ∈ U ⊂

Dom(xγx−1α ) . Entonces (xγx

−1α )|U = (xγx

−1β )(xβx

−1α ) que es C∞ por

ser composicion de funciones C∞ ; vease (ii) de la proposicion 4.6 delcapıtulo 1 . Aplicando ahora (iii) de esta misma proposicion 4.6 se tieneque xγx

−1α es C∞ en r0 , esto sucede para cada r0 de su dominio, por lo

tanto xγx−1α es C∞ . De modo analogo se ve que un cambio de la forma

xαx−1γ es C∞ . En consecuencia, se tiene que A es compatible con C y

la relacion es de equivalencia.

Definicion 1.4. Una variedad diferenciable m-dimensional1 (enalgunos textos tambien se denomina variedad diferencial) consiste enun conjunto M y en una clase de equivalencia [A] del conjunto de atlasdiferenciables m-dimensionales modulo la relacion de compatibilidad.Llamaremos a M el conjunto subyacente de la variedad y la clase [A]diremos que es la estructura diferenciable dada sobre el conjunto M .

Notemos que una variedad queda determinada por una pareja (M,A)donde A es una atlas diferenciable m-dimensional de M . Dos parejas(M,A) , (M,B) determinan las misma variedad si A es compatible conB . Cuando el contexto no de lugar a confusion, diremos que A es laestructura diferenciable de la variedad en vez de su clase de equivalen-cia. Tambien sera frecuente que denotemos directamente por M a unavariedad diferenciable de modo que unas veces M denota el conjuntosubyacente y otras la pareja formada por el conjunto subyacente y laestructura diferenciable.

En la familia de los atlas diferenciables m-dimensionales de un con-junto M , se puede considerar la relacion de contenido . Los elementosmaximales de este conjunto ordenado verifican la siguiente propiedad:

Proposicion 1.2. Las estructuras diferenciables m-dimensionalessobre el conjunto M estan en correspondencia biunıvoca con los ele-mentos maximales del conjunto ordenado de los atlas diferenciablesm-dimensionales de M .

Demostracion. Sea c una clase de equivalencia, y consideremosMc = ∪C∈cC . En primer lugar, comprobaremos que Mc es un atlas.Supongamos que xα ∈ A ∈ c y xβ ∈ B ∈ c . Puesto que A ∪ B es unatlas se tiene que x−1

β xα es C∞ . Ademas es obvio que la reunion delos miembros de Mc es M . Por lo tanto Mc es un atlas y tambien se

1La definicion moderna de variedad diferenciable se atribuye a Hassler Whitney[41]

Page 31: Apuntes Geom Diferencial

1. LA NOCION DE VARIEDAD DIFERENCIABLE 25

tiene que Mc ∈ c . En segundo lugar veremos que Mc es maximal. SiMc ⊂ D , entonces Mc ∪ D = D es un atlas, luego D ∈ c , y por lotanto D ⊂Mc. Por lo que Mc es maximal.

Ahora a cada atlas maximal M le podemos asociar la clase cM =[M] . Sea c = [A] , entonces A ⊂Mc y en consecuencia [Mc] = [A] =c . Por otra parte, si C es maximal, se tiene que C ⊂ M[C] . EntoncesC =M[C] .

Ejemplo 1.1. Consideremos el conjunto de los numeros reales R ,tomemos la carta identidad idR : R → R . Entonces el atlas A = iddetermina una estructura diferenciable 1-dimensional en el conjunto R .En algunos de los ejemplos posteriores la variable que representa a unnumero real se puede interpretar como la variable tiempo, es por elloque, en este caso, tambien denotaremos la carta identidad por t = idR .

Ejemplo 1.2. Sea U un abierto de Rn , consideremos la inclusionx = in: U → Rn que es inyectiva y tiene codominio abierto. Entonces elatlas A = x da una estructura diferenciable n-dimensional al abiertoU . En particular la identidad de Rn induce una estructura canonicade variedad en Rn .

Proposicion 1.3. Sean M y N variedades diferenciables y seaf : M → N una funcion. Sean x , x cartas de M en p ∈ Dom f , e y, y de N en f(p) . Entonces yfx−1 es de clase C∞ en x(p) si y solo siyf x−1 es de clase C∞ en x(p) .

Demostracion. Supongamos que yfx−1 es de clase C∞ en x(p) .Como xx−1 es C∞ en x(p) y yy−1 es C∞ en yf(p) si aplicamos (ii) de laproposicion 4.6 del capıtulo 1, se tiene que (yy−1)(yfx−1)(xx−1) es C∞

en x(p) . Ademas Dom(yy−1yfx−1xx−1) ⊂ Dom(yf x−1) . Por lo tantocomo consecuencia del apartado (iii) de la proposicion 4.6 obtenemosque yf x−1 es de clase C∞ en x(p) .

Definicion 1.5. Sean M y N variedades diferenciables y sea-f : M → N una funcion. Se dice que f es diferenciable en un puntop ∈M si existen cartas x de M en p e y de N en f(p) tal que yfx−1 esde clase C∞ en x(p) . Se dice que f es diferenciable si f es diferenciableen cada punto de su dominio. Diremos que la funcion f es un difeo-morfismo si f es inyectiva y f , f−1 son diferenciables. Se dice que unavariedad M es difeomorfa a una variedad N si existe un difeomorfismoglobal de M sobre N .

El conjunto de funciones diferenciables de M en N se denotara porC∞(M,N) . Si p y q son puntos deM yN respectivamente, utilizaremoslas siguientes notaciones:

C∞((M, p), N) = f ∈ C∞(M,N)|p ∈ Dom f ,

C∞((M, p), (N, q)) = f ∈ C∞(M,N)|p ∈ Dom f, f(p) = q .

Page 32: Apuntes Geom Diferencial

26 2. VARIEDADES DIFERENCIABLES

Para U abierto de M usaremos las notaciones:

C∞U (M,N) = f ∈ C∞(M,N)|Dom f = U .

En el caso mas frecuente que N = R se reducen del modo siguiente:

C∞(M,R) = C∞(M) ,

C∞((M, p),R) = C∞(M, p) ,

C∞U (M,R) = C∞U (M) .

Proposicion 1.4. Una funcion F : (Rm, [idRm,])→ (Rn, [idRn,])es diferenciable si y solo si F : Rm → Rn es de clase C∞ .

Demostracion. Sea x = idRm e y = idRn , entonces yFx−1 = F .De la definicion de funcion diferenciable se tiene que F es diferenciablesi y solo si F es C∞ .

Notemos que para dar la definicion de variedad diferenciable yfuncion diferenciable hemos utilizado los espacios Rm y las funcio-nes C∞ entre ellos. Si en vez de usar estas funciones tomamos, porejemplo, funciones de clase Ck , se obtiene la nocion de variedad declase Ck m-dimensional. En particular, para k = 0 tenemos la no-cion de variedad topologica. Otra familia interesante es la de las va-riedades analıticas reales; para definir esta nocion se consideran fun-ciones analıticas reales. Es tambien frecuente utilizar el semiespacioRm

+ = (r1, · · · , rm)|rm ≥ 0 para introducir la nocion de variedadcon borde. Para definir las variedades complejas se usa el espacio Cm .Finalmente hacemos notar que se pueden utilizar espacios de Banachpara introducir variedades de dimension infinita.

Problemas

1.1. Demostrar que la funcion β : R −→ R definida por β(x) = x3

es una carta que determina una estructura diferenciable en R diferentede la estructura estandar.

Solucion: Supondremos conocido que la funcion β(x) = x3 es globaly biyectiva. Entonces β es un atlas para el conjunto M . Notemosque id−1 β = β es de clase C∞ . Sin embargo id β−1 = β−1 no esde clase C1 ya que no es derivable en el punto x = 0 . Por lo tanto(R, [id]) 6= (R, [β]) .

1.2. Consideramos en R las dos siguientes estructuras de varie-dad diferencial: a) la estructura estandar, b) la que le dota la funcionβ : R −→ R definida por β(x) = x3 . Demostrar que estas dos varieda-des diferenciales son difeomorfas.

Solucion: Consideremos la funcion β−1 : (R, [id]) → (R, [β]) .Puesto que β es una biyeccion solo es necesario comprobar que β, β−1

Page 33: Apuntes Geom Diferencial

LA NOCION DE VARIEDAD DIFERENCIABLE 27

son diferenciables. Notemos que ββ−1 id−1 = id es de clase C∞ . Tam-bien id ββ−1 = id es de clase C∞ . Esto implica que β es un difeomorfis-mo global y suprayectivo. Luego las variedades (R, [id]) , (R, [β]) ,que ya como ya hemos visto son distintas, son sin embargo difeomorfas.

1.3. Considerar el conjunto de puntos O = (sen 2s, sen s)|s ∈R , vease figura 2 . Probar que la funcion x : O → R definida porx(sen 2s, sen s) = s si 0 < s < 2π , es una funcion inyectiva con co-dominio abierto y dominio O . Notese que esta carta determina unaestructura diferenciable enO . Probar tambien que la funcion y : O → Rdefinida por y(sen 2t, sen t) = t si −π < s < π , es una funcion inyecti-va con codominio abierto y dominio O . Probar que estas estructurasdiferenciables aunque son distintas son difeomorfas.

Solucion: Estudiemos el cambio de cartas xy−1 . Se tiene queDom(xy−1) = (−π, π) y su codiminio es Codom(xy−1) = (0, 2π) . Elcambio viene dado por la funcion

xy−1(t) =

t+ 2π si −π < t < 0,

π si t = 0,

t si 0 < t < π

que no es continua. Luego las estructuras [x] , [y] son distintas.Sin embargo, si consideramos el difeomorfismo, φ : (−π, π) → (0, 2π)definido por φ(t) = t + π , se tiene que x−1φy es un difeomorfismoglobal y suprayectivo de (O, [y]) en (O, [x]) .

1.4. Sea f : A −→ M una biyeccion definida en un conjunto A yque toma valores en una variedad diferenciable M . Demostrar que:

a) Si α es una carta de M , αf es una carta para A.b) La coleccion de todas las cartas definidas en el anterior apartado

es un atlas diferenciable para A.c)f es un difeomorfismo entre las variedades A y M .

Solucion: a) Puesto que la composicion de funciones inyectivas eninyectiva, se tiene que αf es inyectiva. Por ser f una biyeccion se ob-tiene que Codom(αf) = Codom(α) es un abierto. Entonces cada αfes una carta diferenciable de la misma dimension que la de M . b) Lareunion de los dominios es la siguiente: ∪Dom(αf) = ∪f−1(Domα) =f−1(∪Domα) = f−1M = A . Para α, β cartas de M , el cambio decartas viene dado por (αf)(βf)−1 = αff−1β−1 = αβ−1 que es unafuncion de clase C∞ . c) Para cada carta α se tiene que αf(αf)−1 = idy (αf)f−1(α)−1 = id son de clase C∞ . Entonces f es un difeomorfismoglobal y sobre de A en M .

1.5. Sea A un conjunto con cardinalidad del continuo y n un enterono negativo. Probar que A admite al menos una estructura de variedaddiferenciable n-dimensional.

Page 34: Apuntes Geom Diferencial

28 2. VARIEDADES DIFERENCIABLES

Solucion: Para n = 0 , para cada elemento a ∈ A se puede consideraruna carta a : A→ R0 tal que Dom a = a y definida por a(a) = 0 . Elatlas a|a ∈ A dota a A de una estructura de variedad diferenciable0-dimensional. Para cada n > 0 , puesto que cada Rn tambien tiene lacardinalidad del continuo, existen una biyeccion x : A → Rn de modoque (A, [x]) es una variedad diferenciable de dimension n .

1.6. Sea M una variedad diferenciable de dimension m. Demostrarque:

a) Toda carta de M es un difeomorfismo.b) Todo difeomorfismo f : M −→ Rm es una carta de M .

Solucion: a) Hemos de probar que si x : M → Rm es una carta deM , entonces x, x−1 son diferenciables en cada punto de su dominio.Para cada p ∈ Domx tomemos las cartas x en p e idRm en x(p) .Notemos que idRm xx

−1 = id |Codomx y xx−1 id−1Rm = id |Codomx son de

clase C∞ . Por lo tanto x, x−1 son diferenciables.Para probar b) supongamos que f es un difeomorfismo. Para probar

que f es una carta hemos de ver que es compatible con todas las cartasde un atlas de M . Si x es una carta de M , entonces aplicando a)se tiene que fx−1 y xf−1 son diferenciables. Puesto que ademas sonfunciones de Rm en Rm se tiene que fx−1 y xf−1 son de clase C∞ .Por lo tanto f es compatible con todas las cartas de un atlas, luego fes una carta de M .

1.7. Sea f : M −→ N una funcion entre variedades diferenciables.Demostrar que las siguientes condiciones son equivalentes:

a) f es diferenciable en un punto p de su dominio.b) ψf es diferenciable en p para toda funcion ψ : N −→ R que es

diferenciable en f(p).c) Existe una carta y de N en f(p) tal que yif es diferenciable

en p para i ∈ 1, · · · , n.Solucion: Si f es diferenciable en p , aplicando las propiedades de

composicion de funciones diferenciables, se obtiene que ψf es diferen-ciable en p . Si se verifica b), en particular se obtiene que para una cartay de N en f(p), yif es diferenciable en p para i ∈ 1, · · · , n . Finalmen-te, suponiendo que se verifica c), si x es una carta en p y sea y una cartaen f(p) con componentes yi : N → R tales que y1f, · · · , ynf son diferen-ciables en p . Entonces se tiene que yfx−1 = (y1fx

−1, · · · , ynfx−1) esC∞ en p ; es decir , f es diferenciable en p .

1.8. Sea ∅ el conjunto vacio y n un entero no negativo. Probarque ∅ admite exactamente una estructura de variedad diferenciable n-dimensional para cada n .

1.9. Probar que el atlas determinado por las proyeciones del ejemplo3.2 y el atlas determimado al tomar argumentos en el ejemplo 3.3 paraS1 son equivalentes.

Page 35: Apuntes Geom Diferencial

2. LA TOPOLOGIA DE UNA VARIEDAD DIFERENCIABLE 29

1.10. Probar que el atlas determinado por las proyeciones del ejem-plo 3.2 y el atlas estereografico del ejemplo 3.4 para Sn−1 son equiva-lentes.

1.11. Dar una estructura de variedad diferenciable 2-dimensional alespacio de las rectas de un plano euclidiano.

1.12. Sean M y M ′ dos variedades de la misma dimension sobreel mismo conjunto soporte. Demostrar que si la aplicacion identidadid : M −→ M ′ es un difeomorfismo, M y M ′ tienen el mismo atlasmaximal y por lo tanto M = M ′.

1.13. Probar que la n-bola Bn es difeormorfa a Rn .

1.14. Probar que una variedad diferenciable m-dimensional tieneun atlas de modo que cada una de sus cartas tiene como codominioRn .

2. La topologıa de una variedad diferenciable

En esta seccion veremos que la estructura diferenciable de una va-riedad siempre induce una topologıa en el conjunto subyacente de esta.

Proposicion 2.1. Sea A una atlas diferenciable m-dimensionalsobre un conjunto M . Sea x una carta de A y supongamos que W ⊂Domx verifica que x(W ) es un abierto de Rm , entonces A∪ x|W esun atlas diferenciable m-dimensional compatible con A .

Demostracion. Sea y una carta de A . Entonces teniendo encuenta que el dominio de una composicion de funciones viene dado porDom(ϕψ) = ψ−1(Dom(ϕ)) . En particular se tiene que Dom y(x|W )−1) =x|W (Dom y) = x(W ∩ Domx ∩ Dom y) = x(W ∩ Dom y) = x(W ) ∩x(Dom y) = x(W ) ∩ Dom(yx−1) . De modo analogo Dom(x|Wy−1) =y(Domx|W ) = y((W ∩ Domx ∩ Dom y) = y(W ∩ Dom y) = y(W ) =(xy−1)−1x(W ) . Por lo tanto se verifica que

y(x|W )−1 = yx−1|x(W )∩Dom(yx−1)

(x|W )y−1 = (xy−1)|(xy−1)−1xW .

Aplicando la proposicion 4.6 del capıtulo 1 se obtiene que estas funcio-nes son C∞ . El primer cambio es la restriccion de una funcion C∞ a unabierto . El segundo cambio tiene como dominio y(W ) = (xy−1)−1xWque es la imagen inversa por una funcion continua de un abierto, y porlo tanto es abierto, ası que tambien es la restriccion de una funcion C∞

a un abierto. Ademas la reunion de los dominios de A∪x|W contienea la reunion de los dominios de A que es M .

Una forma de topologizar un conjunto consiste en probar que unafamilia de subconjuntos verifica las propiedades de la proposicion 2.2del capıtulo 1, como vemos a continuacion:

Page 36: Apuntes Geom Diferencial

30 2. VARIEDADES DIFERENCIABLES

Proposicion 2.2. Los dominios de las cartas del atlas maximal deuna variedad diferenciable M verifican las condiciones necesarias paraser la base de una topologıa en el conjunto M .

Demostracion. En primer lugar, notese que la reunion de losdominios de las cartas de un atlas maximal M es el conjunto M . Ensegundo lugar, si x, y son cartas de M entonces x(Domx ∩ Dom y) =Dom(yx−1) que es un abierto por ser dominio de una funcion C∞ .Aplicando la proposicion 2.1 se obtiene que x|Domx∩Dom y es tambien unacarta del atlas maximal. Consecuentemente Domx∩Dom y es tambienel dominio de una carta del atlas maximal.

Definition 2.1. A la topologıa inducida por los dominios de lascartas del atlas maximal de una variedad diferenciable M la llamaremostopologıa inducida por la estructrura diferenciable (a veces abreviamosdiciendo topologıa inducida).

Definition 2.2. Si (M, τ) un espacio topologico (vease la defi-nicion 2.1), diremos que [A] es una estructura diferenciable sobre elespacio topologico (M, τ) si la topologıa inducida por la estructura esτ .

Observacion 2.3. Se pueden dar ejemplos de estructuras diferen-ciables C∞ que no son difeomorfas y que tienen el mismo espacio to-pologico subyacente; vease el problema 4.10 del capıtulo 3 . El problemade encontrar estructuras diferenciables no difeomorfas es mas difıcil enel caso que el espacio topologico subyacente sea Hausdorff. Sin embargoJ. W. Milnor [24] en 1956 probo que la esfera S7 tiene varias estructu-ras que no son difeomorfas. Es conocido que el numero de estructurasno difeomorfas de una esfera es finito. Se sabe que la esfera S7 tiene 28estructuras; la esfera S11, 992; la esfera S15, 16256 y la esfera S31 masde dos millones. Para mas informacion ver el trabajo de Milnor [24] .

Proposicion 2.3. Sea una variedad diferenciable M con su topo-logıa inducida. Si x es una carta de M , entonces la funcion x es unhomeomorfismo (para la definicion de homeomorfismo para funciones,vease la definicion 2.3).

Demostracion. Puesto que Domx es un abierto para ver que xes una funcion abierta es suficiente ver que las imagenes de los abiertosbasicos son abiertos euclıdeos. Sea W ⊂ Domx un abierto basico quesera el dominio de una carta y . Teniendo en cuenta que el dominiode yx−1 es abierto y que Dom(yx−1) = x(W ) se deduce que x(W ) esabierto. Para ver que x es continua, sea U abierto de Rm, si W =x−1(U) se tiene que x(W ) = Codomx ∩ U es un abierto. Aplicando laproposicion 2.1 obtenemos que x|W es tambien una carta y su dominioW es un abierto basico y por lo tanto abierto. Luego la funcion x escontinua y, en consecuencia, la funcion x es un homeomorfismo.

Page 37: Apuntes Geom Diferencial

2. LA TOPOLOGIA DE UNA VARIEDAD DIFERENCIABLE 31

Proposicion 2.4. SeaM una variedad diferenciablem-dimensional,x una carta en un punto p deM y seaBε(x(p)) una bola de centro x(p) yradio ε contenida en Codomx . Entonces la familia x−1Bδ(x(p))|δ < εes una base de entornos de p en la topologıa inducida por la variedadM .

Demostracion. Por la proposicion anterior sabemos que la cartax : M → Rm es un homeormorfismo. Sea G un entorno de p en M ,entonces existe un abierto U tal que p ∈ U ⊂ G . Notemos que V = U∩Domx es tambien un entorno abierto de p en Domx . Aplicando que xes un homeomorfismo se tiene que x(V ) es un entorno abierto de x(p) enCodomx . Puesto que Bδ(x(p))|δ < ε es una base de entornos de x(p)en Codomx , tenemos que existe δ tal que x(p) ∈ Bδ(x(p)) ⊂ x(V ) .Entonces p ∈ x−1Bδ(x(p)) ⊂ V ⊂ U ⊂ G . Por lo tanto se deduceque x−1Bδ(x(p))|δ < ε es una base de entornos de p en la topologıainducida por la variedad M .

Proposicion 2.5. Sean M y N variedades diferenciables y seaf : M → N una funcion. Si f es diferenciable en un punto p de M ,entonces la funcion f es continua en p .

Demostracion. Sean x e y cartas de M y N en los puntos py f(p) , respectivamente. Entonces yfx−1 es C∞ en el punto x(p) .Por lo tanto existe un abierto U de Rm tal que U ⊂ Dom(yfx−1) yF = yfx−1|U es C∞ . Ademas se tiene que U ⊂ Codomx . Aplicandola proposicion 4.6 del capıtulo 1 tenemos que F es continua en U . Porser la funcion x continua se tiene que x−1U es un abierto. Teniendo encuenta que y es un homeomorfismo se concluye que y−1Fx es continuaen el abierto x−1U ⊂ Dom f . Por otra parte f |x−1U = y−1Fx . Por lotanto f es continua en x−1U y en consecuencia en el punto p .

Proposicion 2.6. Sean M y N variedades diferenciables, f : M →N una funcion y U un abierto de M . Si f es diferenciable, entoncesla funcion f |U es diferenciable. Si f es un difeomorfismo, entonces lafuncion f |U es un difeomorfismo.

Demostracion. Sea p ∈ Dom(f |U) = Dom f ∩ U . Sean x e ycartas de M y N en los puntos p y f(p) , respectivamente. Notemosque y(f |U)x−1 = yfx−1|x(f−1(Dom y)∩U) . Por ser f diferenciable, apli-cando la proposicion 2.5, obtenemos que f es continua y por lo tantof−1(Dom y) es un abierto de M . Por la proposicion 2.3 se tiene quex es un homeomorfismo y en consecuencia x(f−1(Dom y) ∩ U) es unabierto. Por ser f diferenciable en el punto p , entonces yfx−1 es C∞ enel punto x(p) ∈ x(f−1(Dom y)∩U) ⊂ x(f−1(Dom y)) = Dom(yfx−1) .Aplicando la proposicion 4.6 del capıtulo 1 se obtiene que y(f |U)x−1 esC∞ en el punto x(p) . Luego f |U es diferenciable en cada punto de sudominio, y por lo tanto f |U es diferenciable.

Page 38: Apuntes Geom Diferencial

32 2. VARIEDADES DIFERENCIABLES

Supongamos ahora que f es un difeomorfismo. Entonces f |U estambien inyectiva y diferenciable por el argumento anterior. Por serf−1 continua se tiene que f(U) = (f−1)−1(U) es abierto. Notemos que(f |U)−1 = f−1|f(U) tambien es diferenciable. Entonces tenemos que f |Ues un difeomorfismo.

Proposicion 2.7. Sean P ,Q ,R variedades diferenciables, f : P →Q , g : Q→ R funciones diferenciables. Si f , g son diferenciables, en-tonces gf es diferenciable.

Demostracion. Sea a ∈ Dom(gf) y tomemos cartas x, y, z en a ,f(a) y gf(a) , respectivamente. Notemos que Dom((zgy−1)(yfx−1)) ⊂Dom(zgfx−1) . Ademas zgfx−1|Dom((zgy−1)(yfx−1)) = (zgy−1)(yfx−1) .Sabemos que yfx−1 es C∞ en x(a) y que zgy−1 es C∞ en yf(a) . Apli-cando la proposicion 4.6 del capıtulo 1 se obtiene que (zgy−1)(yfx−1) esC∞ en x(a) . Por lo tanto zgfx−1 es C∞ en x(a) . De aquı se obtieneque gf es diferenciable en cada a de su dominio. Luego gf es diferen-ciable.

Problemas

2.1. Sea M una variedad diferenciable m-dimensional y sea A unatlas de la variedad. Probar que si U ⊂ M , entonces U es un abiertode la topologıa inducida si y solo si U ∩ Domx es abierto en Dom xpara todo x ∈ A .

Solucion: Si U es un abierto de M , Domx es un subconjunto de My tenemos en cuenta la definicion de la topologıa relativa (ver el ejemplo2.2 ) se sigue que U ∩ Domx es abierto en Domx . Recıprocamente,si U ∩ Domx es abierto en Domx para todo x ∈ A se tiene que U ∩Domx es abierto en M ya que los dominios de cartas de la estructuradiferenciable son siempre abiertos. Ademas U =

⋃x∈A(U ∩Domx) por

ser A un atlas de M , puesto que la reunion de abiertos es abierto, seobiene que U es abierto.

2.2. Sea M una variedad diferenciable m-dimensional y sea A unatlas de la variedad. Probar que si Bα es una base del espacio topologicoDomxα, para cada xα ∈ A , entonces B =

⋃Bα es una base de la

topologıa inducida por la estructura diferenciable m-dimensional.

Solucion: Si U es un abierto de M , por el ejercicio anterior se tieneque U =

⋃x∈A(U∩Domx) y cada U∩Domx es abierto en Domx . Cada

U ∩Domx es entonces reunion de abiertos de Bα y en consecuencia Ues una reunion de abiertos de

⋃Bα . Luego B =

⋃Bα es una base de

la topologıa inducida.

2.3. Sea M una variedad diferenciable m-dimensional no vacıa. Pro-bar que M es un espacio topologico discreto si y solo si m = 0 .

Page 39: Apuntes Geom Diferencial

3. EJEMPLOS DE VARIEDADES DIFERENCIABLES 33

2.4. Sea M una variedad diferenciable m-dimensional y sea A unatlas de la variedad. Probar que la aplicacion φ :

⊔x∈ACodomx→M ,

tal que para r ∈Codom x, φ(r) = x−1(r) es diferenciable, continua yabierta.

2.5. Sea M una variedad diferenciable m-dimensional. Probar queM es un cociente de una reunion disjunta de bolas m-dimensionales.

2.6. Encontrar un espacio topologico que no admita estructura devariedad. Es decir que las estructuras diferenciables que admita el con-junto subyacente induzcan topologıas distinta de la dada.

2.7. Probar que la reunion de los ejes coordenados de Rn para n ≥ 2provisto con la topologıa usual no admite estructura de variedad. Esdecir que las estructuras diferenciables que admita el conjunto subya-cente inducen topologıas distinta de la dada.

2.8. Intentar probar la certeza o falsedad de la siguiente afirmacion:Existen espacios topologicos que admiten estructuras diferenciables dis-tintas.

2.9. Intentar probar la certeza o falsedad de la siguiente afirmacion:Existen espacios topologicos que admiten estructuras diferenciables nodifeomorfas.

2.10. Encontrar una funcion continua entre variedades que no seadiferenciable.

3. Ejemplos de variedades diferenciables

En primer lugar veamos que todo espacio vectorial real de dimen-sion n admite una estructrua estandar de variedad diferenciable n-dimensional.

Ejemplo 3.1. Espacios vectoriales reales de dimension finita. Seae1, e2, · · · , en una base de un espacio vectorial real E. Notemos que elisomorfismo f : E −→ Rn que asocia a cada vector de E sus coordena-das en la base anterior, es una carta que define una estructura diferen-ciable en el conjunto E. Tenemos que probar que la estructura diferen-ciqable es independiente de la base elegida. Sea e′1, e

′2, · · · , e′n otra base

de E y sea f ′ : E → R es la correspondiente aplicacion coordenada. Siel cambio de base viene dado por ei =

∑aije

′j , i, j ∈ 1, · · · , n , en-

tonces el cambio de cartas estara determinado por f ′f−1(r1, · · · , rn) =(∑riai1, · · · ,

∑riain) . Puesto que cada componente de f ′f−1 es de

clase C∞ , se tiene que los atlas f y f ′ son compatibles. (Es-ta estructura se llama estructura diferenciable estandar de un espaciovectorial real.)

La siguiente notacion facilitara la descripcion de algunos ejem-plos de esta seccion, dada una n-tupla (r1, · · · , ri, · · · , rn) y un i ∈

Page 40: Apuntes Geom Diferencial

34 2. VARIEDADES DIFERENCIABLES

1, · · · , n , la (n−1)-tupla obtenida al suprimir la componente i-esima,(r1, · · · , ri−1, ri+i · · · , rn) la denotaremos a veces por (r1, · · · , ri, · · · , rn) .

Ejemplo 3.2. Sea Sn−1 = r ∈ Rn|r21 + · · · + r2

n − 1 = 0 . Paracada i ∈ 1, · · · , n sean xi+ , xi− funciones de Sn−1 en Rn−1 condominios Domxi+ = r ∈ Sn−1|ri > 0 , Domxi− = r ∈ Sn−1|ri < 0y definidas por

xi+(r1, · · · , ri, · · · , rn) = (r1, · · · , ri, · · · , rn),

xi−(r1, · · · , ri, · · · , rn) = (r1, · · · , ri, · · · , rn) .

Entonces el atlas x1+, x1−, x2+, x2−, · · · , xn+, xn− dota a Sn−1 de unaestructura de variedad diferenciable (n− 1)-dimensional.

Ejemplo 3.3. Sea S1 = (cos r, sen r) ∈ R2|r ∈ R . ConsideremosU = (cos s, sen s) ∈ R2| − π < s < π y V = (cos t, sen t) ∈ R2|0 <t < 2π . Sean x, y dos funciones de S1 en R con dominios Domx = U ,Dom y = V y definidas por x(cos s, sen s) = s , y(cos t, sen t) = t . Elcambio de cartas viene dado por

yx−1(s) =

2π + s si −π < s < 0,

s si 0 < s < π,

que es una funcion C∞ . Entonces el atlas x, y dota a S1 de unaestructura de variedad diferenciable 1-dimensional.

Ejemplo 3.4. El atlas estereografico de la esfera Sn−1 = r ∈Rn|r2

1 + · · · + r2n − 1 = 0 . Sean x , y funciones de Sn−1 en Rn−1

con dominios Domx = Sn−1 \ N (donde N = (0, · · · , 0, 1) es el“polo Norte”) y Dom y = Sn−1 \ −N definidas por x(r1, · · · , rn)= ( r1

1−rn , · · · ,rn−1

1−rn ) , y(r1, · · · , rn) = ( r11+rn

, · · · , rn−1

1+rn) . Notemos que

tenemosxi =

ri1− rn

, yi =ri

1 + rnLa sumas de cuadrados verifican

y21 + · · ·+ y2

n−1 =r2

1 + · · ·+ r2n−1

(1 + rn)2=

1− r2n

(1 + rn)2=

1− rn(1 + rn)

x21 + · · ·+ x2

n−1 =r2

1 + · · ·+ r2n−1

(1− rn)2=

1− r2n

(1− rn)2=

1− rn(1 + rn)

De donde se sigue que el cambio de cartas viene dado por

xi =ri

1− rn=

ri(1 + rn)

(1 + rn)(1− rn)=

yiy2

1 + · · ·+ y2n−1

yi =ri

1 + rn=

ri(1− rn)

(1− rn)(1 + rn)=

xix2

1 + · · ·+ x2n−1

que son funciones C∞ .Entonces el atlas x, y , que llamaremos estereografico, dota a Sn−1

de una estructura de variedad diferenciable (n− 1)-dimensional.

Page 41: Apuntes Geom Diferencial

3. EJEMPLOS DE VARIEDADES DIFERENCIABLES 35

Ejemplo 3.5. En Rn+1\0 definimos la siguiente relacion de equi-valencia (r0, · · · , rn) ∼ (s0, · · · , sn) si existe t 6= 0 tal que (r0, · · · , rn) =(ts0, · · · , tsn) . Denotemos la clase de equivalencia de (r0, · · · , rn) por[r0, · · · , rn] y al conjunto cociente por P n(R) y lo llamaremos espacioproyectivo real de dimension n . Para cada i ∈ 0, · · · , n sea xi la fun-cion de P n(R) en Rn con dominio Domxi = [r0, · · · , rn] ∈ P n(R)|ri 6=0 , y definida por xi[r0, · · · , ri, · · · , rn] = (r0/ri, · · · , ri, · · · , rn/ri) .Entonces el atlas x0, · · · , xn dota a P n(R) de una estructura de va-riedad diferenciable n-dimensional.

Ejemplo 3.6. Sea U un abierto de una variedad diferenciable m-dimensional M . Entonces U admite una estructura de variedad di-ferenciable m-dimensional de tal modo que la inclusion in : U → Mes un difeomorfismo. En efecto supongamos que A es un atlas paraM , entonces consideremos la familia AU = xU |x ∈ A , donde lafuncion xU : U → Rm tiene como dominio U ∩ Domx y se define porxU(p) = x(p) para p ∈ U ∩Domx . Veamos que AU es un atlas diferen-ciable m-dimensional que da estructura de m-variedad a U . Es claroque la reunion de los dominios es U . Los cambios de cartas vienendados por (yU)(xU)−1 = yx−1|x(U) que son funciones C∞ . Por otraparte la inclusion in : U → M es inyectiva y tanto ella como su inver-sa son diferenciables. En efecto para cada carta x de M se tiene quex in(xU)−1 = id |x(U) = (xU) in−1 x−1 .

Ejemplo 3.7. M(n × p,R) matrices reales de orden n × p . Tie-ne estructura de variedad diferenciable np-dimensional ya que es unespacio vectorial real de dimension np , vease el ejemplo 3.1. Denote-mos por Eij la matrız que es nula salvo en el lugar (i, j) que tiene ununo. Se puede tomar como atlas el formado por la funcion coordenadaasociada a la base E11, · · · , E1p, · · · , En1, · · · , Enp que en esencia aso-cia a una matrız la tupla obtenida al colocar cada fila a continuacionde anterior. Un caso particular es la variedad de matrices cuadradasM(n,R) = M(n× n,R) que tiene dimension n2 .

Ejemplo 3.8. GL(n,R) matrices cuadradas reales y no singulares,tambien se llama el grupo lineal real n-dimensional. Tiene estructu-ra de variedal diferenciable n2-dimensional ya que es un abierto deM(n,R) que tiene dimension n2 . Para ver que es un abierto se pue-de considerar la aplicacion determinante det : M(n,R) → R , que porser una funcion polinomica es continua, vease el problema 4.3 de estecapıtulo.

Ejemplo 3.9. Variedades de Grassmann 2 . Sea G(n, p,R) el es-pacio de todos los p-planos de Rn . Una base de un p-plano se puede

2Las variedades de Grassmann juegan un papel importante en la categorıade las variedades diferenciables. Se ulilizan para el estudio de transversalidad avariedades inmersas en variedades riemannianas. Ademas sus grupos de homotopıay de homotopıa estable determinan las clases de cobordismo

Page 42: Apuntes Geom Diferencial

36 2. VARIEDADES DIFERENCIABLES

representar por una matriz A de dimension n × p que tenga rango p .Notemos que dos matrices A y B representan el mismo p-plano si y solosi existe una matriz T no singular p×p tal que B = AT . De este modorepresentaremos un p-plano como la clase de equivalencia [A] inducidapor la relacion de equivalencia anterior.

Supongamos que α : 1, · · · , p → 1, · · · , n es una aplicacion cre-ciente e inyectiva. Para cada matriz A denotemos por Aα la submatrizformada por las filas α(1), · · · , α(p) y por Aα la submatriz formada porel resto de las filas de A . Notemos que si B = AT con T no singular, setiene que las filas α de B tienen rango maximo si y solo si se verifica lomismo para las de A . Definamos una carta xα : G(n, p,R)→ R(n−p)p detal modo que Domxα = [A]|Aα es no singular y que viene dada porla formula xα([A]) = (AA−1

α )α . En primer lugar veamos que esta biendefinida. Si B = AT con T matriz regular, se tiene que (BB−1

α )α =(AT (AT )−1

α )α = (AT (AαT )−1)α = (ATT−1A−1α )α = (AA−1

α )α .Las funciones xα son inyectivas. En efecto si xα([A]) = (AA−1

α )α =(BB−1

α )α = xα([B]) Entonces AA−1α = BB−1

α . Por lo tanto, B =AA−1

α Bα . En consecuencia, [A] = [B] . Por otra parte dado Z ∈R(n−p)p , podemos suponer que Z es una matriz (n−p)×p para despuescontruir una unica matriz αZ de dimensidones n× p tal que al extraerlas filas α-esimas se obtenga la matriz identidad, (αZ)α = I y el restode las filas sean precisamente sea la matriz Z , (αZ)α = Z . Entoncesxα([αZ]) = Z . Es decir que las funciones xα son suprayectivas, o dichode otro modo Codomxα = R(n−p)p .

Puesto que la reunion de los dominios es precisamente G(n, p,R) ylos cambios de cartas son funciones racionales, que son C∞ , entoncesA = xα|α es inyectiva y creciente es un atlas diferenciable (n− p)p-dimensional.

Problemas

3.1. Probar que C y cualquier espacio vectorial complejo de dimen-sion finita n tiene una estructura de variedad diferenciable (real) dedimension 2n .

3.2. Probar que H , vease el problema 6.3 , y cualquier espaciovectorial cuaternionico de dimension finita n tiene una estructura devariedad diferenciable (real) de dimension 4n.

3.3. Definir el espacio proyectivo complejo P n(C) y demostrar quetiene una estructura diferenciable de dimension 2n determinada por unatlas de n+ 1 cartas.

3.4. Demostrar que la recta proyectiva compleja P 1(C) es difeomor-fa a la esfera S2.

Page 43: Apuntes Geom Diferencial

EJEMPLOS DE VARIEDADES DIFERENCIABLES 37

3.5. Sea H el algebra de division de los cuaterniones de Hamilton.Definir el espacio proyectivo P n(H) y demostrar que tiene una estruc-tura diferenciable de dimension 4n determinada por un atlas de n + 1cartas.

3.6. Demostrar que la recta proyectiva cuaternionica P 1(H) es di-feomorfa a la esfera S4.

3.7. El conjunto M(n,C) de las matrices complejas n×n, tiene unaestructura C∞ estandar de dimension 2n2. Tal estructura esta definidapor una carta global:

α : M(n,C) −→ R2n2

dada por α(a+ bi) = (u(a), u(b)), siendo (a) y (b) las correspondientesmatrices reales (parte real y parte imaginaria) y u la carta que definela estructura C∞ de M(n,R). Demostrar que el conjunto GL(n,C) delas matrices regulares complejas es un abierto de M(n,C) que puedeser dotado de estructura de variedad.

3.8. El conjunto M(n,H) de las matrices cuaternionicas n×n, tieneuna estructura C∞ estandar de dimension 4n2. Demostrar que el con-junto GL(n,H) de las matrices regulares cuatenionicas es un abiertode M(n,H) que puede ser dotado de estructura de variedad.

3.9. Demostrar que el conjunto Mp(n× p,R) de las matrices n× preales de rango p es un subconjunto abierto de la variedad M(n×p,R)y tiene por lo tanto una estructura estandar de variedad. Probar losresultados analogos para C y para H .

3.10. Dar una estructura de variedad diferenciable a los conjuntosde p-planos (variedades de Grassmann) complejos y cuaternionicos.

3.11. Consideremos un pendulo, que podemos representar por unsegmento de longitud fija situado en un plano y en el que uno de susextremos pasa por un punto fijo. Dar estructura de 1-variedad a lavariedad de todas las posiciones posibles de dicho pendulo.

3.12. Encontrar una variedad que modele todas posiciones posiblesde un solido rıgido.

3.13. ¿ Es adecuada la nocion de variedad (sin borde), para mode-lizar matematicamente todas las posiciones posibles de un oscilador?

3.14. Consideremos un brazo articulado de un robot formado pordos segmentos articulados en el que uno de los extremos esta fijo en unpunto del espacio. ¿Que variedad podemos utilizar para realizar unaprogramacion de todos los movimientos posibles de este brazo?

Page 44: Apuntes Geom Diferencial

38 2. VARIEDADES DIFERENCIABLES

4. El conjunto de ceros de una funcion con valores reales

Recordamos a continuacion el siguiente resultado que se suele deno-minar como el teorema de la funcion implıcita. Aquı utilizaremos unaversion enunciada en terminos de funciones C∞ .

Teorema 4.1. (Funcion implıcita) Sea (a, b) un punto del do-minio de una funcion C∞ f : Rp × Rq → Rp tal que f(a, b) = 0 y

det

((∂fi∂rj

)(a,b)

)6= 0 , i, j = 1, · · · , p . Entonces existen entornos abier-

tos V de a en Rp y W de b en Rq tal que para todo w ∈ W existe un uni-co ϕ(w) ∈ V tal que f(ϕ(w), w) = 0 . Ademas la funcion ϕ : Rq → Rp

es de clase C∞ .

Este resultado facilita la demostracion de que, bajo ciertas condicio-nes, el conjunto de ceros de una funcion de varias variables con valoresreales tiene una estructura canonica de variedad.

Teorema 4.2. Sea f : Rn → R una funcion C∞ y sea S = f−10 .Supongamos que para cada z ∈ S existe un i ∈ 1, . . . , n tal que(∂f∂ri

)z6= 0 . Entonces f induce una estructura de variedad diferenciable

(n− 1)-dimensional tal que la topologıa de la variedad S coincide conla topologıa de S como subespacio de Rn .

Demostracion. Sea s ∈ S y sea i = i(s) un entero tal que(∂f∂ri

)s6= 0 . Como consecuencia del teorema de la funcion implıci-

ta, existen entornos abiertos W de (s1, · · · , si, · · · , sn) y V de si talque para cada w = (z1, · · · , zi−1, zi+1, · · · , zn) ∈ W existe un unicoϕi(w) ∈ V tal que

f(z1, · · · , zi−1, ϕi(z1, · · · , zi−1, zi+1, · · · , zn), zi+1, · · · , zn) = 0

Ademas la funcion ϕi : W → V es C∞ . Sea ahora

Pi(W,V ) = (z1, · · · , zi, · · · , zn)|(z1, · · · , zi−1, zi+1, · · · , zn) ∈ W , zi ∈ V que es un abierto de Rn . Sea θi la funcion de Rn en Rn−1 que tiene comodominio Pi(W,V ) y que aplica (z1, · · · , zi, · · · , zn) en (z1, · · · , zi, · · · , zn) ,es decir es la restriccion de una proyeccion a un abierto. Considere-mos la funcion x : S → Rn−1 definida por la composicion x = θiin ,siendo in : S → Rn la inclusion canonica. Notese que Codomx =θi(S ∩ Pi(W,V )) ⊂ W . Por el teorema de la funcion implıcita da-do w = (z1, · · · , zi−1, zi+1, · · · , zn) ∈ W existe un unico ϕi(w) ∈ Vtal que u = (z1, · · · , zi−1, ϕi(w), zi+1, · · · , zn) ∈ f−1(0) = S . Luegou ∈ (S ∩ Pi(W,V )) = Domx es tal que x(u) = w . De donde seobtiene que Codomx = W es un abierto de Rn−1 . Sean ahora u =(z1, · · · , zi, · · · , zn) y u′ = (z′1, · · · , z′i, · · · , z′n) puntos de Domx tales

que x(u) = x(u′) . Entonces (z1, · · · , zi, · · · , zn) = (z′1, · · · , z′i, · · · , z′n)aplicando la unicidad del teorema de la funcion implıcita se obtiene

Page 45: Apuntes Geom Diferencial

EL CONJUNTO DE CEROS DE UNA FUNCIoN CON VALORES REALES 39

que zi = z′i . Por lo tanto u = u′ y se tiene que x es inyectiva. Luego xes una carta.

Sea ahora otra carta de la forma y = θjin . El dominio de yx−1 esθi(S ∩ Dom θi ∩ Dom θj) = θi(S ∩ Dom θi) ∩ θi(Dom θj) = Codomx ∩θi(Dom θj) que es un abierto por ser x una carta y θi una aplicacionabierta. Ademas si j > i , se tiene que

yx−1(z1, · · · , zi, · · · , zn) = y(z1, · · · , zi−1, ϕi(z1, · · · , zi, · · · , zn), · · · , zn)

= (z1, · · · , zi−1, ϕi(z1, · · · , zi, · · · , zn), zi+1, · · · , zj, · · · , zn) es una fun-cion C∞ . Analogamente se procede para j < i y en el caso i = j ,yx−1 es la identidad de un abierto. Por lo tanto los cambios de cartasson C∞ . Notemos que para cada s ∈ S hemos construido una carta ens , ası que facilmente se deduce que la reunion de los dominios de lascartas es el propio S .

Para ver que la topologıa de la variedad S es precisamente la to-pologıa traza, veamos por una parte que la inclusion in : S → Rn esdiferenciable. En efecto sea x una carta de S, entonces

idRn inx−1(z1, · · · , zi, · · · , zn) = (z1, · · · , ϕi(z1, · · · , zi, · · · , zn), · · · , zn)

que es una funcion C∞ . Por otra parte si U es un entorno abierto de scontenido en el dominio de una carta x se tiene que U = S∩Pi(x(U), V )y por lo tanto tambien es un entorno abierto de s en la topologıarelativa.

Problemas

4.1. Demostrar que la funcion f : R3 −→ R dada por

f(r1, r2, r3) = (r21 + r2

2 + r23 + 3)2 − 16(r2

2 + r23)

determina en f−1(0) la estructura de una variedad diferenciable. Veasela figura 1.

Figura 1. f(r1, r2, r3) = (r21 + r2

2 + r23 + 3)2 − 16(r2

2 + r23)

Page 46: Apuntes Geom Diferencial

40 2. VARIEDADES DIFERENCIABLES

4.2. Considerar la funcion f : R2 −→ R dada por

f(r1, r2) = r42 − r2

2 +1

4r2

1

Comprobar que f y sus derivadas parciales se anulan simultaneamenteen algun punto. Demostrar que f−1(0) es la figura Ocho estudiada enel problema 1.3 . Vease figura 2 .

Solucion: Notemos que ∂f∂r1

= r12

y ∂f∂r2

= 4r32 − 2r2 . Entonces si

0 = r12

y 0 = 4r32−2r2 . Tiene como soluciones (0, 0) , (0, 1√

2) , (0,− 1√

2) .

La primera es tambien solucion de f pero las dos ultimas no son zerosde f .

Es facil comprobar que r1 = sen 2s y r2 = sen s satisface la ecuacionf(r1, r2) = 0 . Reciprocamente, sea (r1, r2) una pareja tal que r2

2(r22 −

1) = −(r12

)2. Entonces r2

2 ≤ 1 , por lo tanto existe t ∈ R tal quer2 = sen t = sen(π− t) . En consecuencia r2

2(r22 − 1) = −(sen t cos t)2 =

−(

sen 2t2

)2= −

(r12

)2. De aqui se tiene que r1 = sen 2t o r1 = − sen 2t =

sen 2(π−t) . En cualquier de los casos se tiene que (r1, r2) es de la forma(sen 2s, sen s) para algun s .

Figura 2. Ocho: r42 − r2

2 + 14r2

1 = 0

4.3. Sea F : Rn −→ R un funcion de clase C∞ homogenea de gradodistinto de cero y con al menos un valor positivo. Demostrar que lafuncion f : Rn −→ R dada por f(r) = F (r) − 1 determina en f−1(0)una estructura de variedad diferenciable.

Solucion: Si F es homogenea de grado k significa que para cada n-tupla r = (r1, · · · , rn) y cada escalar λ se verifica que F (λr) = λkF (r) .Esta funciones verifican que r1

∂F∂r1

+· · ·+rn ∂F∂rn = kF . En efecto, denote-

mos por s = (s1, · · · , sn) , derivando la expresion F (λs) = λkF (s) res-

pecto λ se tiene que s1

(∂F∂r1

)λs

+· · ·+sn(∂F∂rn

)λs

= kλk−1F (s) . Tenien-

do en cuenta que r = λs , obtenemos que r1λ

(∂F∂r1

)r+ · · ·+ rn

λ

(∂F∂rn

)r

=

Page 47: Apuntes Geom Diferencial

EL CONJUNTO DE CEROS DE UNA FUNCIoN CON VALORES REALES 41

kλk−1F ( rλ) = k

λF (s) . Multiplicando por λ se obitene la expresion

deseeada. Sea ahora un r tal que f(r) = F (r) − 1 = 0 . Entonces∂f∂ri

= ∂F∂ri

. Si suponemos que en el punto r es tal que ∂f∂ri

= 0 para

i ∈ 1, · · · , n . Entonces kF (r) = r1

(∂F∂r1

)r

+ · · · + rn

(∂F∂rn

)r

= 0 .

Pero como F (r) = 1 se tendrıa que k = 0 . Esta contradiccion vienede suponer que todas las derivadas parciales se anulan en el punto r .Entonces siempre existe alguna derivada parcial que no anula en cadapunto r con f(r) = 0 . En consecuencia r|f(r) = 0 es un variedaddiferenciable (n − 1)-dimensional. Es interesante observar si para un

r0 , F (r0) 6= 0 , F

(r0

(F (r0)1k

)= 1 . Por lo tanto la fibra r|f(r) = 0 es

no vacia.

4.4. Demostrar que las funciones f, g : R3 −→ R dadas por

f(r1, r2, r3) = r21 + r2

2 − r23 − 1

g(r1, r2, r3) = r21 − r2

2 − r23 − 1

determinan en f−1(0) y en g−1(0) una estructura de variedad diferen-ciable (notar que en el primer caso se trata del hiperboloide de unahoja y en el segundo del de dos hojas,vease figura 3). Dar en cada casoun atlas finito que defina su estructura diferenciable.

Figura 3. Hiperboloides de una, r21 + r2

2 − r23 − 1 = 0 ,

y de dos hojas, r21 − r2

2 − r23 − 1 = 0 .

4.5. Sea F : Rn−1 −→ R una funcion cualquiera que es diferenciableen Rn−1.

a) Demostrar que la funcion f : Rn −→ R dada por

f(r1, · · · , rn) = F (r1, · · · , rn−1)− rndetermina en f−1(0) , que es el grafo de F , una estructura de variedaddiferenciable.

Page 48: Apuntes Geom Diferencial

42 2. VARIEDADES DIFERENCIABLES

b) Demostrar que esta variedad de difeomorfa a Rn−1.c) Considerar como ejemplo las variedades diferenciables en R3 de-

terminadas por las funciones f, g : R3 −→ R , vease 4 , dadas por:

(i) f(r1, r2, r3) = r21 + r2

2 − r3

(ii) g(r1, r2, r3) = r21 − r2

2 − r3.

Figura 4. Paraboloide r21 + r2

2 − r3 = 0 , y silla demontar, r2

1 − r22 − r3 = 0 .

4.6. Sea f : Rn −→ R una funcion C∞ y sea S = f−1(0) de modo

que para cada p ∈ S la matrız jacobiana

((∂f∂r1

)p, · · · ,

(∂f∂rn

)p

)es no

nula, por lo que sabemos que S es una hipersuperficie ((n−1)−variedaddiferenciable) . Definimos la funcion g : Rn × R −→ R por la formulag(r1, . . . , rn+1) = f(r1, . . . , rn) . Demostrar que g−1(0) es una hipersu-perficie en Rn+1 (Esta nueva variedad se llama cilindro sobre S . Veasefigura 5)

4.7. Sea f : R2 −→ R una funcion C∞ y sea C = f−1(0) de modo

que para cada p ∈ C la matrız jacobiana

((∂f∂r1

)p,(∂f∂r2

)p

)es no nula,

por lo sabemos C es una hipersuperficie (curva en el plano) que ademassupondremos que esta situada en el semiplano r2 > 0. Sea S = g−1(0)donde g : R3 −→ R es la funcion definida de la siguiente manera:

g(r1, r2, r3) = f(r1, (r22 + r2

3)1/2)

Demostrar que S = g−1(0) es una 2-variedad (superficie de revolucionobtenida por rotacion de la curva C alrededor del eje r1 .)

Page 49: Apuntes Geom Diferencial

5. PROPIEDADES BASICAS DE LA TOPOLOGIA DE UNA VARIEDAD 43

Figura 5. Cilindro

Solucion: Sea la curva de ecuacion f(s1, s2) = 0 de tal modo ques2 > 0 y ademas una de las derivadas paerciales ∂f

∂s1, ∂f∂s2

es no nula.Entonces, se tiene que

∂g

∂r1

=∂f

∂s1

∂g

∂r2

=∂f

∂s2

r2

(r22 + r2

3)12

∂g

∂r3

=∂f

∂s2

r3

(r22 + r2

3)12

De donde se obtiene que(∂g

∂r2

)2

+

(∂g

∂r3

)2

=

(∂f

∂s2

)2

De las igualdades anteriores se sigue que no se pueden anular si-multaneamente las tres derivadas parciales de la g .

4.8. Aplicar el ejercicio anterior para encontrar la ecuacion en implıci-tas del toro generado al girar alrededor del primer eje una circunferenciade radio uno.

5. Propiedades basicas de la topologıa de una variedad

En esta seccion estudiamos algunas propiedades topologicas quetienen las topologıas inducidas por las estructuras diferenciales. Con-cretamente vemos que son T1, vease la definicion 2.8; primero contable,definicion 2.7; y localmente conexa, definicion 2.9 ; donde todas lasdefiniciones son del del capıtulo 1.

Page 50: Apuntes Geom Diferencial

44 2. VARIEDADES DIFERENCIABLES

Proposicion 5.1. Sea M una variedad diferenciable. Entonces elespacio topologico subyacente verifica:

(i) M es un espacio T1 ,(ii) M es primero contable,

(iii) M es localmente conexa.

Demostracion. Sea p un punto de una variedad diferenciable M ,consideremos una carta x de M en p . Si q es otro punto de M distintode p pueden ocurrir dos casos, que q 6∈ Domx o que q ∈ Domx . En elprimero, Domx es un entorno abierto de p al cual no pertenece q . Enel segundo, por ser Domx homeomorfo a un abierto euclıdeo, se tieneque Domx es T1 , luego existe U abierto de Domx tal que p ∈ U yq 6∈ U , ademas puesto que Dom x es abierto en M se tiene que U esabierto en M luego U es un entorno abierto de p en M .

Para probar (ii) y (iii), tomemos p ∈ M y una carta x en el puntop . Puesto que Domx es homeomorfo a un abierto euclıdeo se tieneque existe una base contable de entornos conexos de p en Domx .Ademas por ser Domx abierto en M se concluye que tambien es unabase de entornos de p en M . Luego M es primero contable y localmenteconexa.

Un espacio topologico X se dice que es localmente compacto sipara p ∈ X y cada entorno N , entonces existe un entorno V tal quep ∈ V ⊂ V ⊂ N tal que V es compacto.

Proposicion 5.2. SeaM una variedad diferenciable. SiM es Haus-dorff, entonces M es locamente compacta.

Demostracion. Sea N un entorno de un punto p de una variedadM . Tomemos una carta x de M en p tal que Dom x ⊂ N . Sea B unentorno de x(p) en Codomx tal que clB sea compacto, donde cl denotael operador clausura, y clB ⊂ Codomx . Por ser x−1 continua se tieneque x−1B ⊂ x−1(clB) ⊂clDomx(x

−1B) ⊂clM(x−1B) . Por otra partex−1(clB) es un compacto en el espacio Hausdorff M luego es cerradoen M . Entonces clM(x−1B) ⊂ x−1(clB) . En consecuencia x−1B esun entorno abierto de p en M cuya clausura x−1(clB) es un entornocompacto de p contenido en N .

Para recordar la nocion de segundo contable, vease la definicion2.7 del capıtulo 1. En general las variedades diferenciables no son se-gundo contables como podemos ver en el ejemplo 5.1 .

Proposicion 5.3. Sea M una variedad diferenciable. Entonces Mtiene un atlas contable si y solo si M es segundo contable.

Demostracion. SeaA un atlas contable, para cada carta x ∈ A setiene que Domx es homeomorfo a un abierto euclıdeo que es segundocontable. Si Bx

i |i ∈ N es una base contable de Dom x , entoncesBx

i |x ∈ A , i ∈ N es una base contable de M . Recıprocamente, si

Page 51: Apuntes Geom Diferencial

PROPIEDADES DE LA TOPOLOGIA DE UNA VARIEDAD 45

M es segundo contable y B es una base contable y A un atlas, entoncesla familia B′ = B ∈ B| existe una carta xB ∈ A tal que B ⊂ DomxBes contable y xB|B ∈ B′ es un atlas de M .

Observacion 5.1. Si M es una variedad diferenciable, entoncespor ser localmente conexa, cada componente conexa C de M es cerraday abierta en variedad. Por ser abierta C hereda de modo natural unaestructrura diferenciable de la variedadM . Si Ci, i ∈ I es el conjunto decomponentes de M se tiene que M =

⊔i∈I Ci . Ademas cada inclusion

ini : Ci → M es una aplicacion diferenciable. Tambien es facil de verque f : M → N es una funcion entonces f es diferenciable si y solo sicada f ini es diferenciable.

Es tambien de interes la suma disjunta de variedades. Sea Mi ,i ∈ I una familia de variedades m-dimensionales. Si consideramos lareunion disjunta M =

⊔i∈IMi = ∪i∈IMi × i , entonces M admite el

siguiente atlas si x es una carta de Mi entonces xi : M → Rm tiene comodominio Domx×i y esta definida por xi(p, i) = x(p) . Se compruebacon facilidad que es un atlas diferenciable para M de manera que las“inclusiones” ini : Mi → M , ini(p) = (p, i), p ∈ Mi, son diferenciables.Notemos que las funciones fi : Mi → N son diferenciables si y solo sila unica funcion f : M → N tal que f ini = fi es diferenciable.

Ejemplo 5.1. Sea M una variedad diferenciable m-dimensional ysea I in conjunto de ındices de cardinalidad no contable y tomemosMi = M . Entonces

⊔i∈IMi es una variedad que no es segundo conta-

ble.

Problemas

5.1. Sea L el subconjunto de R2 que es union de los conjuntosU = (r, 0) | r ∈ R y V = (r, 1) | r ∈ R, r ≥ 0. Sea U1 el subcon-junto obtenido de U reemplazando los puntos (r, 0) por (r, 1) ∀r ≥ 0.Definimos dos cartas α y α1 con dominios U y U1 respectivamentedadas por

α(r, 0) = r

α1(r, 0) = r (r < 0); α1(r, 1) = r (r ≥ 0)

a) Demostrar que estas cartas forman un atlas diferenciable de Len R. Demostrar que la topologıa inducida no es Hausdorff.

b) Definimos otra carta α2 con dominio U2 = U1 dada por

α2(r, 0) = r3 (r < 0); α2(r, 1) = r3 (r ≥ 0)

Demostrar que las cartas α y α2 forman un atlas diferenciable de L enR.

c) Demostrar que los dos atlas anteriores determinan dos estructu-ras diferenciables diferentes sobre el mismo espacio topologico.

Page 52: Apuntes Geom Diferencial

46 2. VARIEDADES DIFERENCIABLES

5.2. Sea F : Rn −→ R, n > 1, una funcion homogenea de gradok ≥ 1 que toma al menos un valor positivo. Sabemos que la funcionf : Rn −→ R definida por f(r) = F (r)−1 determina en f−1(0) una es-tructura de variedad diferenciable. Demostrar que f−1(0) es compactasi y solo si F (r) > 0 ∀r 6= 0.

Solucion: Sea r1 6= 0 un punto tal que F (r1) > 0 . Supongamos queexiste un punto r0 6= 0 tal que F (r0) ≤ 0 . Puesto que Rn \ 0 esconexo por caminos, ya que n > 1 , se tiene que existe un camino β enRn\0 tal que β(0) = r0 y β(1) = r1 . La funcion fβ verifica que existeun t0 tal que 0 ≤ t0 < 1 de modo que F (β(t0)) = 0 y para t tal que

t0 < t ≤ 1 se tiene que Fβ(t) > 0 . Entonces F

(β(t)

(Fβ(t))1k

)= 1 . Pero sin

embargo lımt−>t0 ‖β(t)

(Fβ(t))1k‖ = ∞ . Esto implica que la variedad no es

acotada. Puesto que su topologıa es la de subespacio de Rn se tiene quesi fuera compacta tambien serıa acotada. Por lo tanto no es compacta.Reciprocamente, si F (r) = 1 , entonces r 6= 0 . Si Z = F−1(1) , entoncesla aplicacion φ : Z → Sn−1 dada por φ(r) = r

‖r‖ esta bien definida y es

continua. Por otro lado, sea r tal que ‖r‖ = 1 , entonces F (r) > 0 yse tiene que r

(F (r))1k∈ Z, luego ψ : Sn−1 → Z dada por ψ(r) = r

(F (r))1k

esta bien definida y es continua. Por lo tanto el espacio subyancentede Z es homoeomorfo a un espacio compacto, luego Z es una variedadcompacta.

5.3. Demostrar que el hiperboloide de dos hojas no es conexo.

5.4. Demostrar que P n(R) es conexo y compacto.

5.5. Demostrar que P n(C) y P n(H) son conexas y compactas.

5.6. Sean a, b y c tres nuneros reales no todos nulos. Demostrar quela funcion f : R3 −→ R dada por

f(r1, r2, r3) = ar2r3 + br3r1 + cr1r2 − 1

determina en f−1(0) una estructura de variedad diferenciable no com-pacta.

5.7. Demostrar que GL(n,R) no es conexo.

5.8. ¿Admite la 2-esfera un atlas con una sola carta? ¿Y el espacioproyectivo?

6. Miscelanea

Los cuaterniones y los octoniones juegan un interesante papel engeometrıa y topologıa y, por supuesto, en algebra.

Willian Rowan Hamilton sabıa que para extender los complejos co-mo conjunto de numeros habıa que renunciar a la conmutatividad del

Page 53: Apuntes Geom Diferencial

6. MISCELANEA 47

producto. Ası que intento la construccion de un algebra real asociati-va 3-dimensional de division, pero aquella tarea era imposible ya quecomo hoy sabemos no existe esa algebra. No obstante, el exito acom-pano a Hamilton en 1843 cuando su hoy famosa algebra real asociativade division 4-dimensional aparecio en su mente mientras paseaba placi-damente con su senora por Dublın. Se conoce esta anecdota, ası comootras circunstancias mas o menos interesantes acerca del nacimiento delos cuaterniones (aunque con menos frecuencia tambien son llamadoscuaternios), gracias a un escrito del propio Hamilton con ocasion deldecimo-quinto aniversario de su descubrimiento.

Mencionaremos tambien que los octoniones parece ser fueron des-cubiertos por John T. Graves en Diciembre de 1843, solo dos mesesdespues del nacimiento de los cuaterniones, comunico su resultado aHamilton en carta en 1844 pero su publicacion se pospuso hasta 1848.Mientras tanto Arthur Cayley redescubrio los octoniones en 1845 ypublico su descubrimiento ese mismo ano.

6.1. Probar que el espacio vectorial real H = a+bi+cj+dk|a, b, c, d ∈R admite una unica aplicacion bilineal P : H × H → H tal que pro-ductos P (i, j) = k = −P (j, i) , P (j, k) = i = −P (k, j) ,P (k, i) = j =−P (i, k) . Probar que es una aplicacion C∞ .

Sean p, q ∈ H y denotemos P (p, q) = pq el producto de cuaternionescon el que H tiene la siguiente estructura:

6.2. Probar que H = a+ bi+ cj + dk|a, b, c, d ∈ R es un algebrade division, donde el producto esta determinado por los productos ij =k = −ji , jk = i = −kj , ki = j = −ik .

6.3. Probar que la aplicacion ι : H \ 0 → H, ι(q) = 1q

es una

aplicacion C∞ .

Utilizando los resultados anteriores, se puede probar con facilidadque los espacios proyectivos cuaternionicos y los correspondients gruposlineales y grassmannianas tienen estructura de variedad.

Damos a continuacion unos ejemplos de difeomorfismos:

6.4. Sea f : R3 −→ R3 dadas por:

f(r1, r2, r3) = (r1 cos r3 − r2 sen r3, r1 sen r3 + r2 cos r3, r3)

Demostrar que f |S2 toma valores en S2. Demostrar que la aplicacioninducida de S2 en S2 es un difeomorfismo.

6.5. Consideramos G(3, 1,R) y G(3, 2,R). A cada 1-plano en R3

con matrız A = (a1, a2, a3)t le hacemos corresponder el 2-plano en R3

que es solucion del sistema homogeneo Atr = 0. Demostrar que estabiyeccion es un difeomorfismo.

El estudio de las conicas en el plano euclıdeo se puede abordar comoel estudio del conjunto de ceros de algunas funciones.

Page 54: Apuntes Geom Diferencial

48 2. VARIEDADES DIFERENCIABLES

6.6. Estudiar el conjunto de ceros de las siguientes funciones de R2

en R . Para cada caso analizar si el conjunto de ceros admite de modonatural una estructura de variedad, dar su dimension y averiguar elnumero de componentes conexas y si cada una de estas componenteses o no compacta.

a) f33(x, y) = x2 + y2 + 1 .b) f31a(x, y) = x2 − y2 + 1 .c) f31b(x, y) = x2 + y2 − 1 .d) f22a(x, y) = x2 + y2 .e) f22b(x, y) = x2 + 1 .f) f21a(x, y) = x2 − y2 .g) f21b(x, y) = x2 − 1 .h) f11a(x, y) = x2 .i) f11b(x, y) = 1 .j) f00(x, y) = 0 .

De modo similar se puede abordar el estudio de cuadricas en elespacio euclıdeo:

6.7. Analogo al anterior para las funciones de R3 en R .

a) f44(x, y, z) = x2 + y2 + z2 + 1 .b) f43a(x, y, z) = x2 + y2 − z2 + 1 .c) f43b(x, y.z) = x2 + y2 + z2 − 1 .d) f43a(x, y, z) = x2 − y2 − z2 + 1 .e) f43b(x, y.z) = x2 + y2 − z2 − 1 .f) f33a(x, y, z) = x2 + y2 + z2 .g) f33b(x, y, z) = x2 + y2 + 1 .

6.8. Probar que la familia de las conicas de un plano proyectivoreal tiene una estructura natural de variedad diferenciable. ¿De quevariedad se trata?. Probar que la familia de conicas no degeneradastambien tiene estructura de variedad.

Page 55: Apuntes Geom Diferencial

CAPıTULO 3

ESPACIO TANGENTE

1. Notaciones previas

Recordemos que (vease la seccion 4 del capıtulo 1) para una funcionC∞, f : Rm → Rn , podemos asociar a cada punto r ∈Dom f la deriva-da (diferencial) de la funcion en r , que denotamos por Dfr : Rm → Rn .La matriz de esta aplicacion lineal calculada respecto las bases canoni-cas se denotara por Jf (r) y se denomina matriz jacobiana de f en elpunto r . Si g : Rm → R es una funcion C∞ , denotamos sus deriva-das parciales por ∂g

∂ri: Rm → R y su valor en un punto r ∈Dom g por(

∂g∂ri

)r

y algunas veces por ∂g∂ri

(r) .

Supongamos que las componentes de f : Rm → Rn, funcion C∞ ,son f1, · · · , fn : Rm → R, entonces la matriz jacobiana en el punto r sepuede expresar en funcion de las derivadas parciales de sus componen-tes:

Jf (r) =

(∂f1∂r1

)r. . .

(∂f1∂rm

)r

.... . .

...(∂fn∂r1

)r. . .

(∂fn∂rm

)r

La estructura afın de Rm permite interpretar un vector u ∈ Rm

como un “vector tangente” en cada punto r ∈ Dom f . Recordemos quela derivada direccional de f segun u verifica que (Duf)r = Dfr(u) , parauna funcion f que tenga derivada en el punto r . Ello permite asociara cada “vector tangente” u una aplicacion u que asocia a cada funcionreal g de clase C∞ en el punto r, el valor u(g) = Dgr(u) .

Si g, g′ funciones reales de clase C∞ definidas en r y α, α′ ∈ R ,entonces la aplicacion u verifica las siguientes propiedades:

Linealidad:

u(αg + α′g′) = D(αg + α′g′)r(u) = αDgr(u) + α′Dg′r(u)= αu(g) + α′u(g′)

Regla de Leibniz:

u(g · g′) = D(g · g′)r(u) = Dgr(u) · g′(r) + g(r) ·Dg′r(u)= u(g) · g′(r) + u(g′) · g(r)

De este modo podemos interpretar cada vector tangente como unaaplicacion que asocia a cada funcion real de clase C∞ definida en r unvalor real y que ademas verifica las propiedades anteriores. Notemos

49

Page 56: Apuntes Geom Diferencial

50 3. ESPACIO TANGENTE

que hemos utilizado la suma, el producto de funciones y tambien elproducto de un escalar por una funcion. Estas operaciones son un casoparticular de la siguiente situacion mas general.

Sea X un conjunto, en la familia de funciones de X en R podemosconsiderar la suma de funciones, el producto de funciones y el productode una funcion por un escalar definidos del modo siguiente: Dadasf, g : X → R definimos f + g como la funcion cuyo dominio Dom(f +g) = Dom f ∩ Dom g y esta definida por (f + g)p = fp + gp , p ∈Dom(f + g) . Similarmente, Dom(f · g) = Dom f ∩Dom g y (f · g)p =(fp) · (gp) . Para α ∈ R se define αf como la funcion cuyo dominioes Dom(αf) = Dom f y (αf)p = α(fp) , p ∈ Dom(αf) . El conjuntode todas las funciones de X en R dispone para cada subconjunto Sde X de un operador restriccion de modo que si f es una funcioncon valores reales, entonces f |S es una funcion que tiene por dominioDom (f |S) = S ∩Dom f y viene dada por f |S(s) = f(s) .

Es interesante observar que el subconjunto de funciones con dominioS tiene estructura natural de anillo conmutativo con unidad. Si S esel vacıo (el uno coincide con el cero). Tambien se verifica que si T ⊂ Sla restriccion a T induce un homomorfismo de anillos con unidad. Esdecir, que el conjunto de las funciones reales puede ser expresado comola reunion disjunta de una familia de anillos conmutativos con unidad,este tipo se estructura se suele denominar “un haz de anillos”.

Problemas

1.1. Sea X un conjunto no vacio. Probar que el conjunto de aplica-ciones de X en R tiene una estructura natural de anillo conmutativocon unidad. Probar que este anillo contiene a un subanillo isomorfo alanillo de los numeros reales.

1.2. Sea X un conjunto. Probar que el conjunto de funciones de Xen R con dominio S ⊂ X , S 6= ∅ , tiene una estructura natural de anilloconmutativo con unidad. Probar que este anillo contiene a un subanilloisomorfo al anillo de los numeros reales. Probar que si ∅ 6= T ⊂ S , larestriccion induce un homomorfismo de anillos con unidad que preservalos elementos del subanillo de los reales. Analizar el caso T = ∅ .

1.3. Sea M una variedad diferenciable. Probar que el conjunto defunciones diferenciables C∞V (M) de X en R con dominio un abiertoV ⊂ X , V 6= ∅ , tiene una estructura natural de anillo conmutativo conunidad. Probar que este anillo contiene a un subanillo isomorfo al anillode los numeros reales. Probar que si ∅ 6= U ⊂ V , la restriccion induceun homomorfismo de anillos con unidad que preserva los elementos delsubanillo de los reales. ¿Que sucede en el caso U = ∅ ?

Page 57: Apuntes Geom Diferencial

DERIVADAS PARCIALES. PROPIEDADES 51

2. Derivadas parciales. Propiedades

Veamos como se traslada la nocion de derivada parcial para lasfunciones diferenciables de una variedad diferenciable. En este casocada carta determina un sistema de derivadas parciales asociado a suscoordenadas.

Definicion 2.1. Sea x : M → Rm una carta y f : M → R unafuncion C∞ . Entonces la derivada parcial de f con respecto la i-esima

coordenada xi se define como la composicion ∂f∂xi

= ∂(fx−1)∂ri

x . Notemos

que su dominio es Dom(∂f∂xi

)= Dom f∩Domx . El valor de la derivada

parcial en un punto p ∈ Dom(∂f∂xi

)se denotara por

(∂f∂xi

)p

y tambien

por(∂f∂xi

)(p).

Proposicion 2.1. Supongamos que f, g : M → R son funcionesC∞ y α, β ∈ R , entonces

(i) ∂(αf+βg)∂xi

= α ∂f∂xi

+ β ∂g∂xi

,

(ii) ∂(f ·g)∂xi

= ∂f∂xi· g + f · ∂g

∂xi.

Demostracion. Para probar (i), consideremos las igualdades:

∂(αf+βg)∂xi

= ∂((αf+βg)x−1)∂ri

x = α∂(fx−1)∂ri

x+ β ∂(gx−1)∂ri

x = α ∂f∂xi

+ β ∂g∂xi

.

Para (ii), tenemos las siguientes:

∂(f ·g)∂xi

= ∂((f ·g)x−1)∂ri

x = ∂(fx−1)∂ri

x · (gx−1)x+ (fx−1)x · ∂(gx−1)∂ri

x

= ∂f∂xi· g + f · ∂g

∂xi

Problemas

2.1. Si x es una carta en una variedad diferenciable demostrar que∂xi∂xj

es una funcion constante de valor δij (delta de Kroneker).

Solucion: Aplicando la definicion de derivada parcial se tiene

∂xi∂xj

=∂xix

−1

∂rjx =

∂ri∂rj

x = δji |Domx

2.2. Si x es una carta en una varidad M y f : M −→ R es unafuncion definida por f = senx1 + cosx2, demostrar que ∂f

∂x1= cosx1 y

∂f∂x2

= − senx2.

Solucion: Utilizaremos la notacion ri para denotar la i-esima coorde-nada de una m-tupla (r1, · · · , rm) y tambien para la i-esima proyecciondel producto ri : Rm → R . Entonces tenemos

f = senx1 + cosx2 = sen r1x+ cos r2x = (sen r1 + cos r2)x

Page 58: Apuntes Geom Diferencial

52 3. ESPACIO TANGENTE

Por lo tanto∂f∂x1

= ∂fx−1

∂r1x = ∂(sen r1+cos r2)xx−1

∂r1x

= ∂(sen r1+cos r2)∂r1

x = (cos r1)x = cosx1

∂f∂x2

= ∂fx−1

∂r2x = ∂(sen r1+cos r2)xx−1

∂r2x

= ∂(sen r1+cos r2)∂r2

x = (− sen r2)x = − senx2

2.3. Sean x = (x1, · · · , xm) : M → Rm una carta en una varie-dad diferenciable M , F : Rm −→ R una funcion C∞ y f = Fx =F (x1, · · · , xm) . Probar que ∂f

∂xi= ∂F

∂rix = ∂F

∂ri(x1, · · · , xm) .

3. Vectores tangentes

Sea M una variedad diferenciable y p ∈M . Recordemos que esta-mos utilizando la notacion C∞(M, p) = f : M → R|f es diferenciabley p ∈ Dom f . Notemos que si α ∈ R , f, g ∈ C∞(M, p) , entoncesαf ∈ C∞(M, p) , f + g ∈ C∞(M, p) y f · g ∈ C∞(M, p) .

Definicion 3.1. Una aplicacion Λ: C∞(M, p) → R se dice quees lineal si verifica que Λ(αf + βg) = αΛf + βΛg para α, β ∈ R yf, g ∈ C∞(M, p) .

Proposicion 3.1. Sea Λ: C∞(M, p)→ R una aplicacion lineal. Sif, g ∈ C∞(M, p) coinciden en un entorno de p , entonces Λf = Λg .

Demostracion. Supongamos que existe un entorno abierto U dep en M tal que U ⊂ Dom f ∩Dom g y f |U = g|U . Notese que Dom(f−f |U) = U y que f − f |U = 0|U = 0(0|U) . Entonces Λ(f − f |U) =Λ(0|U) = Λ(0(0|U)) = 0Λ(0|U) = 0 . Por lo tanto Λ(f) = Λ(f |U) .Analogamente se obtiene que Λ(g) = Λ(g|U) . Entonces Λ(f) = Λ(g) .

En C∞(M, p) podemos definir la siguiente relacion. Dadas f, g ∈C∞(M, p) , diremos que f tiene el mismo germen que g en p si existe unentorno abierto U de p en M tal que U ⊂ Dom f ∩Dom g y f |U = g|U .El cociente con las operaciones inducidas tiene una estructura naturalde R-algebra y sera denotado por C∞p (M) , donde llamamos R-algebraA a un homomorfismo de anillos conmutativos con unidad R → A .Notese que un operador lineal Λ: C∞(M, p) → R factoriza de modonatural del modo siguiente: Λ: C∞(M, p)→ C∞p (M)→ R .

Definicion 3.2. Sea M una variedad. Una derivacion o vector tan-gente en un punto p ∈ M es una aplicacion lineal Λ: C∞(M, p) →R que ademas verifica la regla de Leibniz; es decir, que si f, g ∈C∞(M, p) , entonces Λ(fg) = f(p)Λ(g) + Λ(f)g(p) .

Ejemplo 3.1. Sea x una carta de una variedad M y p ∈ Domx un

punto. Entonces podemos definir(

∂∂xi

)p

: C∞(M, p)→ R mediante la

Page 59: Apuntes Geom Diferencial

3. VECTORES TANGENTES 53

formula(

∂∂xi

)pf =

(∂f∂xi

)p

. Es inmediato comprobar que(

∂∂xi

)p

es un

vector tangente en el punto p de M .

Proposicion 3.2. Sea Λ: C∞(M, p)→ R un vector tangente a Men el punto p . Si f ∈ C∞(M, p) es constante en un entorno de p ,entonces Λf = 0 .

Demostracion. Supongamos que existe un entorno abierto U dep en M tal que U ⊂ Dom f y f |U = c|U , donde c denota la funcionconstante con valor c . Aplicando la proposicion 3.1 se tiene que Λ(f) =Λ(c) = Λ(c · 1) = c · Λ(1) . Por ser Λ una derivacion tenemos lasigualdades Λ(1) = Λ(1 · 1) = 1 · Λ(1) + Λ(1) · 1 = 2Λ(1) . Esto implicaque Λ(1) = 0 . Entonces Λ(f) = c · Λ(1) = 0 .

Si Λ y Λ′ son vectores tangentes en un punto p de una variedadM , entonces la suma Λ + Λ′ definida por la formula (Λ + Λ′)(f) =Λ(f) + Λ′(f) es tambien un vector tangente en el punto p . Tambiense tiene que si α ∈ R podemos definir αΛ por (αΛ)(f) = α(Λ(f)) . Demodo rutinario de comprueba que αΛ es de nuevo un vector tangenteen el punto p . El conjunto de los vectores tangentes en un punto pde una variedad M con las operaciones anteriores tiene estructura deespacio vectorial real.

Definicion 3.3. Denotaremos por TpM al espacio vectorial real delos vectores tangentes en el punto p de una variedad M . Diremos queTpM es el espacio tangente de la variedad en el punto p de M .

Para estudiar la dimension de este espacio tangente utilizaremos elresultado siguiente:

Lema 3.1. Sea x una carta de una variedad M diferenciable m-dimensional y p ∈ Domx un punto tal que x(p) = a . Si f ∈ C∞(M, p)entonces existen h1, . . . , hm ∈ C∞(M, p) tal que f tiene el mismo ger-men que la funcion

f(p) +m∑i

(xi − ai)hi

Demostracion. Tomemos la nueva carta y = x − a que verificaque y(p) = 0 . Sea B una bola contenida en Dom(fy−1) y de centroy(p) = 0 y sea F = (fy−1)|B . Para r = (r1, · · · , rm) ∈ B aplicando laregla de Barrow se tiene

F (r1, · · · , rm)− F (0, · · · , 0) =∫ 1

0dF (sr1,··· ,srm)

dsds =∫ 1

0

∑mi ri

(∂F∂ri

)(sr1,··· ,srm)

ds =∑m

i riHi(r1, · · · , rm)

donde Hi(r1, · · · , rm) =∫ 1

0

(∂F∂ri

)(sr1,··· ,srm)

ds para (r1, · · · , rm) ∈

B . Tomemos hi = Hiy . Para cada q ∈ y−1B se tiene que

Page 60: Apuntes Geom Diferencial

54 3. ESPACIO TANGENTE

f(q)− f(p) = fy−1(r)− fy−1(0) = F (r)− F (0) =∑riHi(r)

=∑m

i yi(q)Hiy(q) = (∑m

i yihi)(q) = (∑m

i (xi − ai)hi)(q) .De donde se obtiene el resultado deseado.

Proposicion 3.3. Sea x una carta de una variedad M y p ∈Domx . Entonces los vectores

(∂∂xi

)p

para i = 1, . . .m forman una

base de TpM . Un vector tangente v ∈ TpM admite en dicha base laexpresion

v =∑i

v(xi)

(∂

∂xi

)p

.

Demostracion. Sea Λ una derivacion en el punto p de M . Supon-gamos que f es una funcion real definida en el punto p . Por el lema 3.1 ylas proposiciones 3.1 , 3.2 se tiene que Λ(f) = Λ(f(p)+

∑i(xi−ai)hi) =∑

i Λ(xi)hi(p) . En particular para el vector tangente(

∂∂xi

)p

la formula

anterior determina que(∂f∂xi

)p

=∑(

∂xj∂xi

)phj(p) = hi(p) . Sustituyen-

do en la primera formula obtenemos que Λ(f) =∑

i Λ(xi)(∂f∂xi

)p

=∑i Λ(xi)

(∂∂xi

)p

(f) . Por lo tanto Λ =∑

i Λ(xi)(

∂∂xi

)p

. Es decir, las

derivadas parciales en el punto p constituyen un sistema generador.

Para ver que son independientes supongamos que 0 =∑λi

(∂∂xi

)p

.

Entonces 0 = 0(xj) =∑λi

(∂xj∂xi

)p

= λj para cada j . Luego el sistema

de vectores es independiente.

Notemos que si x e y son dos cartas en el punto p , entonces elcambio de bases viene dado por(

∂∂xi

)p

=∑

j

(∂yj∂xi

)p

(∂∂yj

)p.

Puesto que (∂yj∂xi

)p

=(∂rjyx

−1

∂ri

)x(p)

la matriz del cambio de base es precisamente la matriz jacobiana Jyx−1(x(p)) .

Problemas

3.1. Consideramos el subconjunto C∞s (M, p) de C∞(M, p) dado porlas funciones f : M −→ R que en algun entorno de p sean de la forma

f = c+∑α

fαgα

donde c es una funcion constante y el segundo termino es una suma fini-ta de funciones fα, gα de C∞(M, p) que verifican que fα(p) = gα(p) = 0.

Page 61: Apuntes Geom Diferencial

4. APLICACION TANGENTE 55

Demostrar que la aplicacion lineal Λ: C∞(M, p) −→ R es una deriva-cion si y solo si es cero sobre C∞s (M, p).

Solucion: Supongamos que Λ: C∞(M, p) −→ R se anula sobre lasfunciones de la forma indicada. En particular se tiene que la funcion−f(p)g(p)+(f−f(p))(g−g(p)) = fg−f(p)g−g(p)f esta en C∞s (M, p) .Por lo tanto se tiene que Λ(fg−f(p)g−g(p)f) = 0 . EquivalentementeΛ(fg) = −f(p)Λ(g) + Λ(f)g(p) .

4. Aplicacion tangente

Sea p un punto de una variedad diferenciable M que esta en eldominio de una funcion diferenciable φ : M → N . Entonces φ induceuna aplicacion lineal Tpφ : TpM → Tφ(p)N del modo siguiente: Si v esun vector tangente en p a M y f : N → R una funcion diferenciabledefinida en p , entonces Tpφ(v)(f) = v(fφ) .

Definicion 4.1. Sea p un punto de una variedad diferenciableM que esta en el dominio de una funcion diferenciable φ : M → N .Llamaremos a Tpφ : TpM → Tφ(p)N la aplicacion tangente a φ en elpunto p y a veces diremos que es la derivada lineal de φ en el punto p .Ademas de la notacion anterior tambien se utilizara Tpφ = φ∗p .

Notese que si x es una carta en p e y es un carta en φ(p) , entoncesTpφ(v) =

∑j Tpφ (v)(yj)(

∂∂yj

)φ(p)

=∑

j v(yjφ)(∂∂yj

)φ(p)

. En particular

para v =(∂∂xi

)p

se obtiene

Tpφ(

∂∂xi

)p

=∑

j

(∂(yjφ)

∂xi

)p

(∂∂yj

)φ(p)

Teniendo en cuenta que (∂(yjφ)

∂xi

)p

=(∂(yjφx

−1)

∂ri

)x(p)

se tiene que la matriz

lineal de Tpφ respecto a las bases asociadas a las cartas x e y , quellamaremos matriz jacobiana de φ en el punto p y denotaremos porJy,xφ (p) , es la siguiente

Jy,xφ (p) =

((∂(yjφ)

∂xi

)p

)=

((∂(yjφx

−1)

∂ri

)x(p)

)= Jyφx−1(x(p))

que coincide con la matriz jacobiana de yφx−1 en el punto x(p) .Recordemos que si F : Rn → Rm es diferenciable y r esta en su

dominio, entonces la matriz jacobiana de F en el punto r respecto lascoordenadas canonicas es precisamente la matriz de la aplicacion linealDFr .

Proposicion 4.1. La aplicacion tangente verifica las siguientespropiedades:

(i) Si φ : M → N y ψ : N → Q son funciones diferenciables y pesta en el dominio de ψφ , entonces Tp(ψφ) = Tφ(p)ψ Tpφ .

(ii) Sea U un abierto de una variedad M y sea p ∈ U . EntoncesTp(id |U) = idTpM .

Page 62: Apuntes Geom Diferencial

56 3. ESPACIO TANGENTE

Demostracion. Sea v una derivacion en el punto p y f una fun-cion diferenciable real definida en el punto p . Entonces Tp(ψφ) (v)(f) =v(fψφ) = Tpφ (v)(fψ) = (Tφ(p)ψ Tpφ) (v)(f) . De aquı se concluye queTp(ψφ) = Tφ(p)ψ Tpφ . Para la segunda parte se tiene que Tp(id |U)(v)(f)= v(f id |U) = v(f |U) = v(f) = idTpM(v)(f) .

Ejemplo 4.1. Sea σ : R → M una funcion diferenciable. Si s ∈Domσ , se llama vector velocidad de la curva en el punto s al vectortangente en σ(s) a M dado por Tsσ

(ddt

)s

.Si A es un espacio vectorial y a ∈ A , se puede definir un isomorfismo

canonico θa : A→ TaA del modo siguiente para cada v ∈ A considere-mos la curva σv(t) = a+ tv , entonces se toma θa(v) = (T0σ

v)(ddt

)0

.

Recordemos que si Tpφ : TpM → TφpN es una aplicacion lineal,llamamos rango de Tpφ a la dimension de la imagen de la aplicacionlineal (Vease definicion 3.5 del capıtulo 1).

Definicion 4.2. Sea φ : M → N una aplicacion diferenciable y pun punto de su dominio. Llamaremos rango de φ en p precisamente alrango de Tpφ .

Es conveniente tener en cuenta que para la aplicacion linealTpφ : TpM → TφpN la suma de su rango y la dimension de su nucleo esigual a la dimension del espacio vectorial TpM (Vease poposicion 3.1del capıtulo 1).

Utilizaremos el siguiente teorema de la funcion inversa para probaruna version similar para variedades.

Teorema 4.1. Sea r un punto del dominio de una funcion C∞ ,h : Rm → Rm . Entonces la diferencial Dhr de h en r es un isomor-fismo si y solo si existe un entorno abierto V de r tal que h|V es undifeomorfismo.

Para variedades diferenciables se tiene la siguiente version:

Teorema 4.2. (Funcion inversa) Sea p un punto del dominio deuna funcion diferenciable φ : M → N . La aplicacion tangente Tpφ esun isomorfismo si y solo si existe un entorno abierto U de p tal que φ|Ues un difeomorfismo.

Demostracion. Supongamos que φ|U es un difeomorfismo. En-tonces Tp(φ|U) es un isomorfismo. Ahora bien en facil comprobar queTp(φ|U) = Tpφ , por lo que Tpφ es un isomorfismo.

Recıprocamente si Tpφ es un isomorfismo, entonces m =dimM =dimN = n . Sean x e y cartas en p y en φ(p) , respectivamente. Lamatriz jacobiana de φ en el punto p , que es la matriz jacobiana deyφx−1 en x(p) , es inversible. Entonces existe un entorno abierto Vde x(p) contenido en el dominio de yφx−1 tal que (yφx−1)|V es undifeomorfismo. Sea U = x−1V y consideremos φ|U . Notese que φ|U =y−1((yφx−1)|V )x , por lo que φ|U es un difeomorfismo.

Page 63: Apuntes Geom Diferencial

4. APLICACION TANGENTE 57

Definicion 4.3. Sea φ : M → N una aplicacion diferenciable yp un punto de su dominio. Se dice que φ es un difeomorfismo localen el punto p si existe un entorno abierto U de p tal que φ|U es undifeomorfismo. Si φ es un difeomorfismo local en todos los puntos desu dominio diremos que φ es un difeomorfismo local.

Dadas dos variedades M y N de dimensiones m y n , en el productocartesiano M × N se puede dar una estructura de variedad (m + n)-dimensional tomando como atlas la familia de cartas x × y| donde xes carta de M e y es carta de N . Notese que el cambio de cartas vienedado por (x′ × y′)(x × y)−1 = x′x−1 × y′y−1 , que claramente es unafuncion C∞ .

Proposicion 4.2. Sean M y N variedades, φ : M × N → M yψ : M × N → N las proyecciones canonicas. Sea (p, q) un punto deM × N y denotemos por φ∗ = T(p,q)φ y ψ∗ = T(p,q)ψ . Entonces la apli-cacion lineal (φ∗, ψ∗) : T(p,q)(M ×N)→ TpM ⊕TqN es un isomorfismo.

Demostracion. Sean iq : M → M × N , ip : N → M × N lasinclusiones definidas por iq(p

′) = (p′, q) , ip(q′) = (p, q′) . Sea la apli-

cacion ξ : TpM ⊕ TqN → T(p,q)(M × N) dada por ξ(u, v) = (iq)∗u +(ip)∗v , donde (iq)∗ = Tpiq , (ip)∗ = Tqip. Notese que (φ∗, ψ∗)ξ(u, v) =(φ∗, ψ∗)((iq)∗u + (ip)∗v) = (φ∗((iq)∗u + (ip)∗v), ψ∗((iq)∗u + (ip)∗v)) =(φ∗(iq)∗u+φ∗(ip)∗v, ψ∗(iq)∗u+ψ∗(ip)∗v) = ((φiq)∗u+(φip)∗v, (ψiq)∗u+(ψip)∗v) = (u, v) , donde hemos utilizado la proposicion 4.1 y el hechode que la aplicacion tangente en un punto de una aplicacion constantees nula. Consecuentemente, la aplicacion lineal (φ∗, ψ∗) entre espaciosvectoriales de la misma dimension es suprayectiva, lo que implica quees un isomorfismo.

Proposicion 4.3. Sea (p, q) un punto del producto M ×N y seanφ : M × N → M y ψ : M × N → N las proyecciones canonicas eiq : M → M × N , ip : N → M × N las inclusiones definidas poriq(p

′) = (p′, q) , ip(q′) = (p, q′) . Si (p, q) esta en el dominio de una

funcion diferenciable f : M×N → L , entonces para w ∈ T(p,q)(M×N)se tiene que

T(p,q)f(w) = Tp(fiq)T(p,q)φ(w) + Tq(fip)T(p,q)ψ(w)

Demostracion. Aplicando la proposicion anterior se tiene quew = (iq)∗φ∗(w) + (ip)∗ψ∗(w) . Entonces f∗(p,q)(w) = (fiq)∗φ∗(w) +(fip)∗ψ∗(w) .

Page 64: Apuntes Geom Diferencial

58 3. ESPACIO TANGENTE

Problemas

4.1. Sean φ, ψ : M −→ N funciones diferenciables que coinciden enalgun entorno de p . Demostrar que Tp(φ) = Tp(ψ).

Solucion: Sabemos que existe un entorno abierto U del punto ptal que f |U = g|U . Entonces, Tp(f |U) = Tp(f id |U) = Tp(f)Tp(id |U) =Tp(f) idTpM = Tp(f) y analogamente Tp(g|U) = Tp(g) . Entonces Tp(f) =Tp(g) .

4.2. Si φ : M −→ N es una aplicacion constante en algun entornode un punto p ∈M , demostrar que Tp(φ) es la aplicacion nula.

4.3. Sean A y B dos espacios vectoriales reales de dimension finitay φ : A −→ B una aplicacion lineal. Dado un vector a ∈ A, demostarque para todo v ∈ A se verifica

φ∗(θav) = θφ(a)(φ(v))

Es decir, que salvo isomorfismos canonicos la aplicacion tangente deuna aplicacion lineal es ella misma.

Solucion: Sea σ : R→M una funcion diferenciable. Si s ∈ Domσ ,se llama vector velocidad de la curva en el punto s al vector tangenteen σ(s) a M dado por σ∗s

(ddt

)s

.Si A es un espacio vectorial y a ∈ A , se puede definir un isomorfismo

canonico θa : A→ TaA del modo siguiente para cada v ∈ A considere-mos la curva σv(t) = a + tv , entonces se toma θa(v) = (σv)∗0

(ddt

)0

.Notemos que

φσv(t) = φ(a) + tφ(v) = σφ(v)(t)φ∗(θav) = φ∗(σ

v)∗0(ddt

)0

= (φσv)∗0(ddt

)0

= (σφ(v))∗0(ddt

)0

= θφ(a)(φ(v))

Por lo tanto φ∗θa = θφ(a)φ .

4.4. Si M es el hiperboloide de dos hojas, demostrar que la inyeccionnatural j : M −→ R3 tiene rango dos en todo punto.

Solucion: El hiperboloide de dos hojas tiene por ecuacion. r21 −

r22 − r2

3 − 1 = 0 . Consideremos el atlas dado por x+, x− de modo queDomx+ = (r1, r2, r3) ∈ M |r1 > 0 , Domx− = (r1, r2, r3) ∈ M |r1 <0 y estan definidas por x+(r1, r2, r3) = (r2, r3) y x−(r1, r2, r3) =(r2, r3) .

Entonces idR2 jx−1+ (a, b) = ((1 +a2 + b2)

12 , a, b) que tiene por matrız

jacobiana

J(a, b) =

a

(1+a2+b2)12

b

(1+a2+b2)12

1 00 1

Page 65: Apuntes Geom Diferencial

APLICACION TANGENTE 59

que evidentemente tiene rango dos. Para la otra carta se obtiene lamisma matriz salvo que hay que considerar signo negativo en las raicescuadradas.

Nota. Admitiendo probados los resultados del capıtulo 4, la soluciones inmediata. La fibras de una submersion son subvariedades regulares.

4.5. Si f ∈ C∞(M, p) y v ∈ Tp(M), demostrar que Tp(f)(v) =a(∂∂t

)f(p)

, siendo a = v(f).

4.6. Demostrar que dos variedades difeomorfas tienen la misma di-mension.

4.7. Demostrar que la funcion f : S2 −→ P 2(R) que asocia a cadapunto z ∈ S2 el 1-plano (la recta) de R3 determinada por z, es undifeomorfismo local.

Solucion: Utilizando resultados del capıtulo 4 se pueden dar so-lucines mas elaboradas utilizando tecnicas de secciones locales o degrupos de transformaciones. Sin embargo aquı vamos a calcular di-rectamene la matriz jacobiana y su rango. Lo haremos respecto unapareja de cartas y de modo analogo se procede en el resto de los ca-sos. Sea p = (r1, r2, r3) ∈ S2 con r1 > 0 y tomemos la carta x en ptal que x(r1, r2, r3) = (r2, r3) y la carta y en [(r1, r2, r3)] definida pory[(r1, r2, r3)] = ( r2

r1, r3r1

) . Entonces si a2 + b2 < 1 se tiene

yfx−1(a, b) = yf((1− a2 − b2)12 , a, b) = y[((1− a2 − b2)

12 , a, b)] =

(a

(1− a2 − b2)12

,b

(1− a2 − b2)12

)

Por tanto la matriz jacobiana sera

J(a, b) =

1−b2

(1−a2−b2)32

ab

(1−a2−b2)32

ab

(1−a2−b2)32

1−a2

(1−a2−b2)32

cuyo determinante es no nulo:

det(J(a, b)) =1

(1− a2 − b2)2

Por lo que f es un difeomorfismo local.

4.8. Demostrar que la funcion f : R −→ S1 definida por f(s) =(sen 2πs, cos 2πs) es un difeomorfismo local.

4.9. Sea a un numero real mayor que cero. Consideramos el difeo-morfismo

λa : Rn+1 \ 0 −→ Rn+1 \ 0definida por λa(z) = az, que satisface la condicion σλa = σ, siendo

σ : Rn+1 \ 0 −→ Sn

Page 66: Apuntes Geom Diferencial

60 3. ESPACIO TANGENTE

la aplicacion dada por σ(z) = z|z| .

Deducir que σ tiene en todos los puntos rango n.

4.10. Sabemos que las variedades M1 y M2 definidas sobre el subes-pacio H de R2 que consiste de los puntos (x, 0) (x ∈ R), junto con elpunto (0, 1) que vienen definidas por los siguientes atlas: Sean U =(x, 0)|x ∈ R y U1 = (x, 0)|x 6= 0

⋃(0, 1) . El atlas de M1

esta formado por las cartas, α : U −→ R definida de la siguiente mane-ra, α(x, 0) = x y α1 : U1 −→ R dada por α1((x, 0)) = x , α1(0, 1) = 0 .El atlas de M2 por las cartas: α : U −→ R definida por α(x, 0) = xy α2 : U1 −→ R dada por α2((x, 0)) = x3 , α2(0, 1) = 0 . Demostrarque estas dos variedades no son difeomorfas pero inducen la mismatopologıa en el conjunto H .

Solucion: Considerar la funcion f : M1 → R dada por f(x, 0) = x ,F (0, 1) = 0 . Es facil ver que f es diferenciable y tiene rango 1.

Veamos que si g : M2 → R es una funcion global y diferenciable,entonces g tiene rango 0 en el punto (0, 0) .

Sea G = gα−1 , G2 = gα−12 . En R \ 0 se tiene que

G = gα−1 = Ggα−12 α2α

−1 .

Entonces se verifica que G(x) = G2(x3) . Las derivadas satisfacen larelacion G′(x) = 3x2G′2(x3) para x 6= 0 . Puesto que son continuas seobtiene que G′(0) = lımx→0G

′(x) = 0 . De aquı se sigue que g tienerango cero en (0, 0) .

Supongamos que existe un difeomorfismo global φ : M2 → M1 . Setiene que φ tiene rango 1, por lo que la composicion fφ : M2 → R tienerango 1. Esto contradice el hecho de que toda g : M2 → R tiene rangocero en (0, 0) . Por lo tanto M1 y M2 no son difeomorfas.

Page 67: Apuntes Geom Diferencial

CAPıTULO 4

SUBVARIEDADES Y VARIEDADESCOCIENTE

El rango de una funcion diferenciable tiene especial interes si coin-cide con la dimension de la variedad inicial o de la variedad final. Enel caso que el rango coincida con ambas se trata de un difeomorfismolocal.

1. Inmersiones

Definicion 1.1. Una funcion f : M → N se dice que es una in-mersion en un punto p de su dominio, si el rango de f en p coincidecon la dimension de M . Si f es inmersion en todos los puntos de sudominio diremos que f es una inmersion. Cuando el dominio de f esM se dice que f es una inmersion global. Si f es una inmersion glo-bal inyectiva se dice que f es un encaje. Si f : M → f(M) es tambienun homeomorfismo, diremos que f es un encaje regular. En este ulti-mo caso si ademas f es una inclusion se dice que M es subvariedad yrespectivamente subvariedad regular de N .

Ejemplo 1.1. Para cada n ≥ m tenemos la descomposicion canoni-ca Rn = Rm × Rn−m y la inclusion inducida in : Rm → Rm × Rn−m

definida por in(r) = (r, 0) . La matriz jacobiana de la inclusion es dela forma

Jid(r) =

(id0

)que tiene rango m . Por lo que in es una inmersion.

Lema 1.1. Sea a ∈ Dom g donde g : Rm → Rn es diferenciable. Si gtiene rango m en a , entonces existe un difeomorfismo h : Rn → Rn demodo que la funcion hg coincide con r → (r, 0) en un entorno abiertode a .

Demostracion. Si g tiene rango m en a , entonces existen 1 ≤σ1 < σ2 < · · · < σm ≤ n tal que det

((∂gσi∂rj

)a

)6= 0 , i ∈ 1, · · · ,m .

Podemos extender σ : 1, · · · ,m → 1, · · · , n a una permutacionσ : 1, · · · , n → 1, · · · , n y definir θ : Rn → Rn mediante la formula

θ(r1, · · · , rn) = (rσ1 , · · · , rσn) . Notemos que det((∂(θg)i

∂rj

)a

)=

det((∂gσi

∂rj

)a

)6= 0 .

61

Page 68: Apuntes Geom Diferencial

62 4. SUBVARIEDADES Y VARIEDADES COCIENTE

Consideremos la funcion ϕ : Rm × Rn−m → Rn dada por ϕ(r, s) =

θg(r) + (0, s) . Entonces se tiene que det((

∂ϕi∂rj

)(a,0)

)6= 0 . Aplican-

do el teorema de la funcion inversa podemos asegurar que existe unaf : Rn → Rm × Rn−m tal que Codom f = U × V con U entorno abier-to de a contenido en Dom(θg) y V entorno abierto de 0 y ademasϕ|U×V = f−1 . Tomemos h = fθ . Para r ∈ U tenemos hg(r) =fθg(r) = f(θg(r) + (0, 0)) = fϕ(r, 0) = (r, 0) .

Proposicion 1.1. Si f : M → N es una inmersion en un punto p ,entonces existen cartas x, y de M y N en p y f(p) , respectivamente,tal que yfx−1(r) = (r, 0) para r ∈ Dom(yfx−1) .

Demostracion. Sea x una carta de M en p y sea y una cartade N en f(p) , entonces yf x−1 es una inmersion en el punto x(p) .El lema 1.1 asegura la existencia de un difeomorfismo h de Rn talque para cada r ∈ Dom(hyfx−1) se tiene que hyfx−1(r) = (r, 0) .Tomemos y = hy y x = x . Entonces si r ∈ Dom(yfx−1) se obtieneque yfx−1(r) = (r, 0) .

Corolario 1.1. Supongamos que p esta en el dominio de unafuncion diferenciable f : M → N . Entonces f es una inmersion en p siy solo si existe un entorno abierto U de p y una funcion diferenciableπ : N →M tal que π(f |U) =id|U .

Demostracion. Supongamos que f es una inmersion en p . En-tonces por la proposicion anterior, existen cartas x en p e y en f(p)tal que yfx−1(r) = (r, 0) . Consideremos la descomposicion Rn =Rm × Rn−m y sea pr : Rn → Rm la proyeccion natural. Sea U =Domx ∩ f−1(Dom y) y π = x−1 pr y . Entonces π(f |U) = x−1 pr y(f |U)= x−1 pr y(f |U)x−1x = x−1 id |x(f−1(Dom y))x = id |U .

Recıprocamente, si π(f |U) =id|U , entonces (Tf(p)π)Tp(f |U) =idTpM .Luego Tp(f |U) = Tp(f) es un monomorfismo. Por lo tanto f es una in-mersion en p .

Proposicion 1.2. Sea f : M → N una inmersion global y seag : L → M una funcion continua. Entonces si fg es diferenciable setiene que g es diferenciable.

Demostracion. Sea p ∈ Dom g . Por ser f una inmersion eng(p) , aplicando corolario 1.1, existe V entorno abierto de g(p) y existeπ : M → N tal que π(f |V ) =id|V . Por ser g continua U = g−1V esun entorno abierto de p . Entonces g|U =(id|V )(g|U) = π(f |V )(g|U) =π(fg|U) . Por ser (fg|U) diferenciable se obtiene que g|U es diferencia-ble. Luego g es diferenciable en cada punto de su dominio. Por lo tantog es diferenciable.

Proposicion 1.3. Sea f : M → N un encaje regular y sea g : L→M una funcion. Entonces si fg es diferenciable se tiene que g es dife-renciable.

Page 69: Apuntes Geom Diferencial

INMERSIONES 63

Demostracion. Notemos que g = f−1(fg) . Entonces fg es con-tinua por ser diferenciable y puesto que f es un encaje se tiene quef−1 es continua considerandola como una aplicacion de f(M) , con latopologıa traza, en M con la topologıa inducida por su estructura devariedad. Puesto que la composicion de aplicaciones continuas es con-tinua, se tiene que g es continua. Aplicando la proposicion anterior seconcluye que g es diferenciable.

Observese que un encaje de una variedad compacta en una Haus-dorff es un encaje regular.

Observacion 1.1. Existe un notable teorema que asegura que da-da una variedad compacta Hausdorff de dimension m se puede encajarregularmente en R2m . Una version del teorema anterior puede verseen la seccion 6 del capıtulo 5 del [5] . Whitney[41] probo una versionmas general para variedades Hausdorff y segundo contables probandoque existıa un encaje regular en R2m+1 . Despues en [42] demostro quese podıa cambiar R2m+1 por R2m .

Problemas

1.1. Sea W un subespacio de un espacio vectorial V de dimensionfinita. Cuando se dan en ambos sus estructuras C∞ estandar, demostrarque W es una subvariedad regular de V .

1.2. Considerar las funciones globales c1, c2 : R −→ R2 definidaspor c1(s) = (sen 2s, sen s) y c2(s) = (cos 2s, cos s).

a) Determinar si son inmersiones.b) Representar graficamente los conjuntos imagen de las aplica-

ciones anteriores.c) Probar que c1|(0,2π) : (0, 2π) → R2 es un encaje que no es re-

gular.

1.3. Determinar los puntos en los que la funcion ψ : R2 −→ R3 defi-nida por: ψ(r1, r2) = (cos r2 cos r1, cos r2 sen r1, sen r2) es una inmersion.Representar graficamente el conjunto imagen de la aplicacion anterior.Vease figura 1

Los parametos (r1, r2) se denominan coordenadas esfericas, normal-mente a r2 ∈ [−π

2, π

2] se le llama latitud y a r1 ∈ [−π, π] longitud.

1.4. Determinar los puntos en los que la funcion ψ : R2 −→ R3

definida por: ψ(r1, r2) = (r2 cos r1, r2 sen r1, r2) es una inmersion. Re-presentar graficamente el conjunto imagen de la aplicacion anterior.Vease figura 2

Solucion: La matriz jacobiana −r2 sen r1 cos r1

r2 cos r1 sen r1

0 1

Page 70: Apuntes Geom Diferencial

64 4. SUBVARIEDADES Y VARIEDADES COCIENTE

Figura 1. Esfera de dimension dos

Figura 2. Cono

tiene rango dos para r2 6= 0 y rango uno para r2 = 0 . Por lo tanto laaplicacion es una inmersion en (r1, r2)|r2 6= 0 .

1.5. Determinar los puntos en los que la funcion ψ : R2 −→ R3 defi-nida por: ψ(r1, r2) = (0, cos r1, sen r1)+r2(cos r1

2, sen r1

2cos r1, sen r1

2sen r1)

es una inmersion. Probar que ψ no es una aplicacion inyectiva. Repre-sentar graficamente el conjunto imagen de la aplicacion anterior. Veasefigura 3

1.6. Demostrar que la funcion f : R −→ R3 dada por

f(t) = (cos 2πt, sen 2πt, t)

es una inmersion. Representar graficamente el conjunto imagen de laaplicacion anterior.Vease figura 4

Solucion: La matriz jacobiana −2π sen 2πt2π cos 2πt

1

Page 71: Apuntes Geom Diferencial

INMERSIONES 65

Figura 3. Banda de Mobious

-1-0.5 0 0.5 1

-1-0.5

00.5

1

-2

-1

0

1

2-1-0.5

00.5

1

Figura 4. Helice

tiene rango uno en todos los puntos . Por lo tanto la aplicacion es unainmersion. Tambien es global e inyectiva; es decir, es un encaje.

1.7. Demostrar que las funciones f, g : (1,∞) −→ R2 definidas por:f(t) = ((1

t) cos 2πt, (1

t) sen 2πt) y g(t) = ( t+1

2tcos 2πt, ( t+1

2t) sen 2πt)

son inmersiones. Representar graficamente los conjuntos imagen de lasaplicaciones anteriores.

Solucion: La matriz jacobiana de f es la siguiente

1

t2

(− cos 2πt− 2πt sen 2πt− sen 2πt+ 2πt cos 2πt

)tiene rango uno en todos los puntos . Notemos que la norma del vector

anterior es 1+(2πt)2

t2, que es mayor que cero. Por lo tanto la aplicacion

es una inmersion global e inyectiva; es decir, es un encaje.Denotemos por A(t) = t+1

2tLa matriz jacobiana de g es la siguiente

dA(t)

dt

(cos 2πtsen 2πt

)+ 2πA(t)

(− sen 2πtcos 2πt

)

Page 72: Apuntes Geom Diferencial

66 4. SUBVARIEDADES Y VARIEDADES COCIENTE

El rango sera uno salvo en los puntos tales que A(t) = 0 y dA(t)dt

= 0 .Puesto que t 6= 1 el sistema anterior no tiene solucion se obtiene que laaplicacion g es una inmersion global e inyectiva; es decir, es un encaje.

1.8. Si ψ : M −→ N es una inmersion y ψ′ : M −→ N ′ es unafuncion diferenciable, demostrar que (ψ, ψ′) : M −→ N × N ′ es unainmersion.

1.9. Si ψ : M −→ N y φ : M ′ −→ N ′ son inmersiones, demostrarque ψ × φ : M ×M ′ −→ N ×N ′ es tambien una inmersion.

1.10. Sea M el siguiente subconjunto de R2: M = (R×0)⋃

(R×1). Consideramos el atlas sobre M formado por las siguientes cartas:x : M −→ R dada por x(s, 0) = s y x1 : M −→ R dada por x1(s, 1) = s.Sean ahora φ : R −→ M definida por φ(s) = (s, 1) si s 6= 0 y φ(0) =(0, 0) y ψ : M −→ R definida por ψ(s, 1) = ψ(s, 0) = s.

Demostrar que ψ es una inmersion, ψφ es diferenciable pero φ noes diferenciable.

1.11. Definir una estructura C∞ sobre el conjunto de las matricessimetricas de tal forma que sea una subvariedad regular de M(n,R).¿Forman las matrices simetricas un conjunto abierto de M(n,R)?

Solucion: Denotemos por S(n,R) el conjunto de matrices simetricasque es un subconjunto deM(n,R) . Con el objetivo de dar una estructu-ra de variedad diferenciable a S(n,R) notemos que para cada entero s ≥1 existen un unico i ≥ 1 y j ≥ 1 tales que s = 1+2+3+· · ·+(i−1)+j .De modo analogo, para cada entero t ≥ 1 existen unicos enteros k ≥ 1y l ≥ 1 tales que t = (k − 1)n + l . Recordemos que la biyeccion

x : M(n,R)→ Rn2definida por xs((aαβ)) = akl .

Definamos ahora la biyeccion y : S(n,R) → Rn(n+1)

2 definida porys((aαβ)) = aij . Denotemos por in : S(n,R) → M(n,R) la inclusioncanonica, entonces se tiene que para t = (k − 1)n+ l

xt in =

y1+···+(l−1)+k si k ≤ l

y1+···+(k−1)+l si l ≤ k

Por lo tanto x in es diferenciable y tambien lo sera in = x−1x in .De modo analogo se ve que la aplicacion τ : M(n,R) → S(n,R) ,

definida por τ(A) = A+At

2, es diferenciable, donde At denota la matriz

transpuesta. De la igualdad τ in = id se desprende que S(n,R) es unasubvariedad regular. Excepto para n = 1 la dimension de S(n,R) esestrictamente menor que la de M(n,R) , entonces para n > 1 no puedeser un abierto.

1.12. Demostrar que con cualquiera de las dos estructuras de varie-dad diferenciable de que se ha dotado a la figura Ocho, ver problema1.3 del capıtulo 2 , es una subvariedad de R2

Page 73: Apuntes Geom Diferencial

INMERSIONES 67

1.13. Sea M ′′ una subvariedad de M ′ y M ′ una subvariedad de M .Demostrar que M ′′ es subvariedad de M .

1.14. Recordemos que una inyeccion φ : M −→ N induce una es-tructura C∞ en su rango φ(M). Si φ es una inmersion, demostrar queφ(M) es una subvariedad de N difeomorfa a M .

1.15. Encontrar un encaje regular del toro S1 × S1 en R3 . Veasefigura 5

Figura 5. Toro

Solucion: Definamos la aplicacion E : R2 → R3 definida porE(φ, ψ) = (2 cos 2πφ, 2 sen 2πφ, 0)

+ (cos 2πψ cos 2πφ, cos 2πψ sen 2πφ, sen 2πψ) == (2 cos 2πφ+cos 2πψ cos 2πφ, 2 sen 2πφ+cos 2πψ sen 2πφ, sen 2πψ)

Esta es diferenciable y su matriz jacobiana se comprueba que tienerango dos. Por lo tanto la aplicacion es una inmersion. Si consideramosel difeomorfismo local pr : R2 → (S1)2 se tiene que E factoriza a travesde dicho cociente para determinar el encaje buscado. Notemos que elencaje es regular ya que la aplicacion inducida es una inyeccion continuade una variedad compacta en una variedad Hausdorff.

1.16. Consideremos el toro T encajado regularmente en R3 tal comohemos visto en el problema anterior. Probar que la aplicacion que acada punto del toro le hace correspondes su vector ortogonal normal(exterior) es una aplicacion diferenciable de T en S2 .

1.17. La funcion f : R3 −→ R definida por: f(r1, r2, r3) = (r21 +

r22 − 1) determina en f−1(0) una estructura de subvariedad regular S

(cilindro circular).Si t es la funcion identidad en R y si j : S1 −→ R2 es la inclusion

natural, demostrar que j×t es una inmersion cuya imagen es S. Deducirque S1 × R es difeomorfa al cilindro circular.

Page 74: Apuntes Geom Diferencial

68 4. SUBVARIEDADES Y VARIEDADES COCIENTE

1.18. Sea M1 una subvariedad de M que esta contenida en unasubvariedad regular M ′ de M . Demostar que M1 es una subvariedadde M ′.

1.19. Demostrar que una subvariedad conexa de M es necesaria-mente un subconjunto conexo de M .

Encontrar una subvariedad de R2 que no sea conexa y que sin em-bargo sea un subconjunto conexo de R2.

1.20. Demostrar que una subvariedad compacta de M es necesaria-mente un subconjunto compacto de M .

Usar la figura Ocho, ver problema 1.3 del capıtulo 2 , para dar unejemplo de una subvariedad de R2 que no es compacto aunque es unsubconjunto compacto de R2.

1.21. Demostrar que una componente de una variedad M es unasubvariedad conexa de M .

1.22. Sea f : R2 −→ R3 la aplicacion definida por

f(θ, φ) = (cosφ cos θ, cosφ sen θ, φ)

a) Encontrar los puntos en los que f es una inmersion.b) Probar que Imf no admite una estructura de variedad de modo

que la inclusion Imf ⊂ R3 sea un encaje regular.

1.23. Considerar la aplicacion g : S1tR→ R2 de la reunion disjuntade una circunferencia y una recta en el plano, definida por g|S1 =inclusiony g(t) = (1 + et)(cos t, sen t). Contestar razonadamente si g es o no esun encaje regular.

2. Submersiones

Definicion 2.1. Una funcion f : M → N se dice que es una sub-mersion en un punto p de su dominio, si el rango de f en p coincidecon la dimension de N . Si f es una submersion en todos los puntos desu dominio diremos que f es una submersion. Cuando el dominio de fes M diremos que f es una submersion global. Si f es una submersionglobal suprayectiva diremos que N es un f -cociente de M . Diremosque N es un cociente de M si existe una submersion global suprayectivade M en N .

Ejemplo 2.1. Para cada m ≥ n tenemos la descomposicion canoni-ca Rm = Rn × Rm−n y la proyeccion canonica inducida pr1 : Rn ×Rm−n → Rn definida por pr1(r1, r2) = r1 . La matriz jacobiana de laproyeccion es de la forma

Jpr1(r1, r2) =(id 0

)que tiene rango n . Por lo que pr1 es una submersion.

Page 75: Apuntes Geom Diferencial

2. SUBMERSIONES 69

Lema 2.1. Sea a ∈ Dom g donde g : Rm → Rn es diferenciable. Sig tiene rango n en a , entonces existe un difeomorfismo h : Rm → Rm

de modo que la funcion gh coincide con la proyeccion (r1, r2) → r1 enun entorno abierto de h−1(a) .

Demostracion. Si g tiene rango n en a , entonces existen 1 ≤σ1 < σ2 < · · · < σn ≤ m tal que det

((∂gi∂rσj

)a

)6= 0 . Podemos exten-

der σ : 1, · · · , n → 1, · · · ,m a una permutacion σ : 1, · · · ,m →1, · · · ,m y definir θ : Rm → Rm mediante la formula θ(r1, · · · , rm) =(rσ1 , · · · , rσm) .

Notese que det((∂(gθ)i

∂rj

)θ−1a

)= det

((∂gi∂rσj

)a

)6= 0 , i, j ∈ 1, · · · , n .

Consideremos la funcion ϕ : Rm → Rn × Rm−n dada por ϕ(r) =

(gθ(r), pr2(r)) . Entonces se tiene que det((

∂ϕi∂rj

)θ−1a

)6= 0 , i, j ∈

1, · · · ,m . Aplicando el teorema de la funcion inversa podemos ase-gurar que existe una f : Rn × Rm−n → Rm tal que Codom f es unentorno abierto de θ−1a contenido en Dom(gθ) y ademas ϕ|Codom f =f−1 . Tomemos h = θf . Para (r1, r2) ∈ Dom f tenemos gh(r1, r2) =gθf(r1, r2) = pr1 ϕf(r1, r2) = r1 .

Proposicion 2.1. Si f : M → N es una submersion en un puntop , entonces existen cartas x, y de M y N en p y f(p) , respectivamente,tal que yfx−1(r1, r2) = r1 para (r1, r2) ∈ Dom(yfx−1) ⊂ Rn × Rm−n .

Demostracion. Sea x una carta de M en p y sea y una carta deN en f(p) . Entonces yf x−1 es una submersion en el punto x(p) . Ellema 2.1 asegura la existencia de un difeomorfismo h de Rm tal quepara cada (r1, r2) ∈ Dom(yf x−1h) se tiene que yf x−1h(r1, r2) = r1 .Tomemos y = y y x = h−1x . Entonces si (r1, r2) ∈ Dom(yfx−1) seobtiene que yfx−1(r1, r2) = yf x−1h(r1, r2) = r1 .

Corolario 2.1. Supongamos que p esta en el dominio de unafuncion diferenciable f : M → N . Entonces f es una submersion en psi y solo si existe una funcion diferenciable s : N →M definida en f(p)tal que fs =id|Dom s y sf(p) = p .

Demostracion. Si f es una submersion en p , entonces exis-ten cartas x, y de M y N en p y f(p) , respectivamente, tal queyfx−1(r1, r2) = r1 para (r1, r2) ∈ Dom(yfx−1) ⊂ Rn × Rm−n . Supon-gamos que x(p) = (a, b) y sea pr : Rn × Rm−n → Rn la proyeccioncanonica, denotemos por ib : Rn → Rm la aplicacion ib(r1) = (r1, b) .Sea el entorno abierto de yf(p) = a dado por W = i−1

b (x(Dom f)) ∩Codom y , sea S = ib|W y tomemos como s = x−1Sy , ahora esfacil ver que sf(p) = p . Notese tambien que Codom(fx−1Sy) ⊂Dom y . Entonces fs = fx−1Sy = y−1yfx−1Sy = y−1 pr(ib|W )y =y−1id|Wy =id|y−1W = id|Dom s .

Recıprocamente, supongamos que existe dicha seccion local s . En-tonces Tpf Tf(p)s = Tf(p)(id |Dom s) = Tf(p) id = idTf(p)N . De la formula

Page 76: Apuntes Geom Diferencial

70 4. SUBVARIEDADES Y VARIEDADES COCIENTE

anterior se deduce que Tpf es un epimorfismo. Luego f es una submer-sion en p .

Proposicion 2.2. Si f : M → N es una submersion, entonces f esuna funcion abierta.

Demostracion. Si U un abierto de M , se tiene que f |U es tam-bien una submersion. Ademas Im(f |U) = f(U) . Entonces sera sufi-ciente que veamos que la imagen de una submersion es un abierto.Supongamos que f(p) ∈Imf . Por ser f una submersion en el puntop , sabemos por el corolario 2.1 que existe una funcion diferenciables : N →M tal que sf(p) = p y fs =id|Dom s . De aquı se concluye queDom s ⊂Imf . Ademas Dom s es un abierto por ser el dominio de unafuncion diferenciable. Luego Imf es entorno de cada uno de sus puntos.Esto implica que Imf es abierto.

Proposicion 2.3. Sea f : M → N una submersion suprayectiva ysea g : N → L una funcion. Entonces si gf es diferenciable se tiene queg es diferenciable.

Demostracion. Sea q ∈ N . Puesto que f es suprayectiva existep ∈ M tal que f(p) = q . Por el corolario 2.1 existe s diferenciable talque fs =id|Dom s . Entonces g|Dom s = (gf)s que es diferenciable por sercomposicion de diferenciables. Por lo tanto g es diferenciable en q paracada q ∈ N . Luego g es diferenciable.

Problemas

2.1. Demostrar que la funcion ξ : C2 −→ P 1(C) definida en C2 \(0, 0) por ξ(z) = 1 - plano de C2 que contiene a z es una submersion.

2.2. Sea M la variedad que hemos definido en el problema 1.10 sobreel conjunto (R × 0)

⋃(R × 1). Sea M1 la variedad definida en el

problema 4.10 del capıtulo 3 por medio de las cartas α y α1 (definidas enel problema referido). Consideramos la funcion φ : M −→ M1 definidapor

φ(s, 0) = (s, 0) s ∈ R

φ(s, 1) = (s, 0) s ∈ R \ 0 ;φ(0, 1) = (0, 1).

Demostrar que φ es una submersion de M sobre M1.

2.3. ¿Es Hausdorff una variedad cociente de una variedad Haus-dorff?

2.4. Si φ : M −→ M ′ y ψ : N −→ N ′ son submersiones, demostrarque

φ× ψ : M ×N −→M ′ ×N ′

es una submersion.

Page 77: Apuntes Geom Diferencial

3. LAS FIBRAS DE UNA SUBMERSION 71

2.5. Considerar la relacion de equivalencia ρ en R2 dada por (z, w) ∈ρ⇐⇒ |z| = |w|.

a) Demostrar que el conjunto cociente no admite la estructura devariedad cociente.

b) Demostrar que cuando la relacion de equivalencia se restringe aR2 \ 0 el conjunto cociente admite tal estructura.

2.6. Sea M una variedad diferenciable y ρ una relacion de equi-valencia en M . Denotamos por M ′ la estructura de variedad cocientedada al conjunto M/ρ (se supone que admite esta estructura). Seaf : M −→M1 una aplicacion invariante por la relacion.

a) Demostrar que si f es inmersion tambien es inmersion la aplica-cion f ′ : M ′ −→M1 inducida por f .

b) Demostrar que si f es submersion tambien es submersion laaplicacion f ′ : M ′ −→M1 inducida por f .

2.7. Sea S(2,R) la variedad de las matrices simetricas reales 2× 2 .a) Demostrar que el conjunto M de todas las matrices simetricas

reales 2× 2 con valores propios distintos es un subconjunto abierto deS(2,R) .

b) Consideramos la siguiente relacion de equivalencia ρ en M :

AρB ⇐⇒ B = T−1AT ;∃T ∈ GL(2,R).

Demostrar que si a M se le dota de estructura de subvariedad abiertade S(2,R) entonces el conjunto cociente admite la estructura de unavariedad cociente M ′.

c) Observad que las funciones con valores reales traza y determinan-te son invariantes diferenciables sobre M . Obtener las correspondientesfunciones inducidas en M ′.

3. Las fibras de una submersion

Proposicion 3.1. Si f : M → N es una submersion y q ∈ Codom f ,entonces S = f−1(q) es una subvariedad regular de dimension m− n .

Demostracion. En primer lugar, supongamos que dim(M) =m = n =dim(N) . Entonces para cada p ∈ f−1(q) , por el teorema1.1 , existe un entorno abierto U de p tal que f |U es un difeomorfismo.En consecuencia U ∩ f−1(q) = p . Es decir, f−1(q) es un espaciodiscreto que admite de modo natural estructura de variedad diferen-ciable 0-dimensional. Es tambien inmediato comprobar que S es unasubvariedad regular de dimension m− n = 0 .

En segundo lugar, supongamos que m > n . Consideremos la des-composicion canonica Rm = Rn × Rm−n . Sea pr2 : Rm → Rm−n lasegunda proyeccion canonica e in2 : Rm−n → Rm la inclusion definidapor in2(r2) = (0, r2) . Notemos que ambas son aplicaciones diferencia-bles.

Page 78: Apuntes Geom Diferencial

72 4. SUBVARIEDADES Y VARIEDADES COCIENTE

Para cada p ∈ S = f−1(q) existen cartas u en p y v en f(p) ta-les que Domu ⊂ Dom(fv) , v(q) = 0 y vfu−1 = pr1 |Dom(vfu−1) .Sea p′ ∈ Domu ∩ S , y supongamos que u(p′) = (r′1, r

′2) , entonces

r′1 = pr1 u(p′) = vf(p′) = v(q) = 0 . Por lo tanto u(p′) ∈ Codomu ∩(0×Rm−n) . Sea ahora (0, r′2) ∈ Codomu∩ (0×Rm−n) , entoncesu−1(0, r′2) ∈ Domu∩S . Luego u(Domu∩S) = Codomu∩(0×Rm−n) .

Denotemos por in : S → M la inclusion canonica. Para cada p ∈ Spodemos considerar la carta x = pr2 u in que es inyectiva y tienepor codominio el abierto in−1

2 (Codomu) . Supongamos que tenemosdos cartas del tipo anterior x = pr2 u in , x = pr2 u in . Estudie-mos la composicion xx−1 . Puesto que x−1 = in−1 u−1 in2 , entoncesxx−1 = pr2 u in in−1 u−1 in2 = pr2 uu

−1 in2 que es una funcion diferen-ciable. Ademas es claro que la reunion de los dominios de las cartasası definidas es S . Entonces S admite estructura de variedad (m−n)-dimensional.

Ahora vamos a ver que la inclusion in : S → M es una inmer-sion. Para p ∈ S consideremos las cartas x y u . Entonces u inx−1 =u in in−1 u−1 in2 = uu−1 in2 = in2 que es una inmersion. Luego in esuna inmersion en p para cada p ∈ S . Es decir que in es una inmersion.Para ver que es regular, queda por ver que la topologıa de S es masfina que la topologıa traza. Sea U un entorno abierto de p contenidoen el dominio de una carta x , entonces x(U) = in−1

2 W con W abiertode Rm contenido en codominio de u . Entonces U = S ∩ u−1W . LuegoU es un abierto de la topologıa traza.

Proposicion 3.2. Sea f : M → N diferenciable, q ∈ Codom f yf es submersion en todo punto p ∈ f−1(q) , entonces f−1(q) es unasubvariedad regular de dimension m− n .

Demostracion. Para cada p ∈ f−1(q) se tiene que si tomamoscartas x, y en p, f(p) , respectivamente, entonces detJy,xf es diferenciabley como su valor en p es no nulo tambien lo sera en un entorno abierto Upde p en M . En consecuencia si U =

⋃p Up , p ∈ f−1(q) , entonces f |U

es una submersion luego f−1(q) es una subvariedad regular de M .

Proposicion 3.3. Sea f : M → N una submersion, q ∈ Codom f ,p ∈ f−1(q) = S y j : S → M la inclusion canonica, entonces TpjTpS =KerTpf .

Demostracion. Notese que fj es una funcion constante. Enton-ces Tpf Tpj = 0 , de donde se tiene que Tpj (TpS) ⊂ KerTpf . Teniendoen cuenta que los espacios vectoriales anteriores tienen la misma di-mension se sigue que Tpj (TpS) = KerTpf .

Page 79: Apuntes Geom Diferencial

LAS FIBRAS DE UNA SUBMERSION 73

Problemas

3.1. Demostrar que la funcion f : R3 −→ R2 definida por:

f1(r1, r2, r3) = r21 − r2

2 + 2r1r3 + 2r2r3 − 1 ,f2(r1, r2, r3) = 2r1 + r2 − r3

determina en f−1(0) una estructura de variedad diferenciable.

3.2. Considerar las funciones f : R3 −→ R y g : R3 −→ R definidasen R3 por:

f(r1, r2, r3) = r21 + r2

2 − r23 − 1 ,

g(r1, r2, r3) = r1 + r2 − r3 − 1 .

a) Demostrar que f, g determinan una estructura de variedad dife-renciable en f−1(0) y en g−1(0).

b) Demostrar que la funcion

(f, g) : R3 −→ R2

no verifica las condiciones usuales de la proposicion 3.2 para que (f, g)−1(0)tenga estructura de subvariedad.

3.3. Sea f : R3 −→ R la aplicacion definida por

f(x, y, z) = x2y2 + y2z2 + z2x2 .

a) Encontrar los puntos en los que f es una submersion.b) Probar que el subconjunto (d > 0)

M = (x, y, z)|x2y2 + y2z2 + z2x2 = 3d4

es una subvariedad regular de R3 .c) Teniendo en cuenta que las unicas 1-variedades conexas Hausdorff

que existen son difeomorfas a S1 o a R . Describir las componentesconexas de la variedad obtenida a cortar M con el plano z = c enfuncion de los valores de c . Probar que M no es compacta.

d) Probar que cada recta que pasa por el origen y no es un eje cortaa M exactamente en dos puntos. Deducir que M es difeomorfa a la2-esfera menos seis puntos. Vease figura 6

Solucion: a) La matriz jacobiana de f es la siguiente

(2x(y2 + z2), 2y(x2 + z2), 2z(x2 + y2))

Es facil comprobar que el conjunto de soluciones de las expresionesanteriores igualadas a cero son exactamente los puntos de los ejes coor-denados. Entonces f es submersion en R3 menos los ejes coordenados.

b) Notemos que puesto que d > 0 la interseccion de M con los ejescoordenados es vacia. Entonces se tiene que f es una submersion entodos los puntos de M . Por lo tanto M es una subvariedad regular.

c) Si consideramos la aplicacion diferenciable (f, z) : R3 → R2 setiene que la interseccion de M y el plano z = c es la fibra de (3d4, c) .

Page 80: Apuntes Geom Diferencial

74 4. SUBVARIEDADES Y VARIEDADES COCIENTE

Figura 6. Esfera con seis agujeros

Comprobemos que (f, z) es una submersion, su matriz jacobiana es lasiguiente: (

2x(y2 + z2) 2y(x2 + z2) 2z(x2 + y2)0 0 1

)que tiene en todos los puntos de M rango 2. Entonces la interseccionde M con el plano z = c es una 1-variedad. Para c 6= 0 se tiene quex2 + y2 + c2 < 3d4

c2+ c2 por los que la interseccion es una variedad

compacta. En este caso se comprueba que solo tiene una componenteconexa.

Para c = 0 se tiene que x2y2 = 3d4 . En este caso no es compacta ytiene cuatro componentes cada una de las cuales es homeomorfa a R.Puesto que las subvariedades son regulares y son subconjuntos cerrados,si M fuera compacta entonces toda subvariedad cerrada tambien loserıa. La subvariedad obtenida para c = 0 no es compacta, luego M noes compacta.

3.4. Sea F : R3 −→ R la aplicacion definida por

F (x, y, z) = x3 + y3 + z3 .

a) Encontrar los puntos en los que F es una submersion.b) Probar que el subconjunto

M = (x, y, z)|x3 + y3 + z3 = 1es una subvariedad regular de R3 .

c) Teniendo en cuenta que las unicas 1-variedades conexas Hausdorffque existen son difeomorfas a S1 o a R . Describir las componentes co-nexas de la variedad obtenida a cortar M con el plano π de de ecuacionz = c en funcion de los valores de c . Probar que M no es compacta.Probar que M es homeomorfa a R2 .

d) Probar que cada recta que pasa por el origen o no corta a M ola corta exactamente en un punto. Vease figura 7

Page 81: Apuntes Geom Diferencial

LAS FIBRAS DE UNA SUBMERSION 75

Figura 7. Superficie difeomorfa a 2-plano

Solucion: a) La matriz jacobiana de F es la siguiente

(3x2, 3y2, 3z2)

Es facil comprobar que el conjunto de soluciones del sistema

3x2 = 0 , 3y2 = 0 , 3z2 = 0

esta formado exclusivamente por el punto (0, 0, 0) .b) Notemos que (0, 0, 0) 6∈M . Entonces se tiene que F es una sub-

mersion en todos los puntos de M . Por lo tanto M es una subvariedadregular.

c) Si consideramos la aplicacion diferenciable (F, z) : R3 → R2 setiene que la interseccion de M y el plano π de ecuacion z = c es la fibrade (1, c) . Veamos si (F, z) es una submersion, su matriz jacobiana esla siguiente: (

3x2 3y2 3z2

0 0 1

)que para c 6= 1 se tiene que si suponemos que simultaneamente x = 0y y = 0 , entonces c3 = 1 pero esto es imposible ya que c 6= 1 . Por lotanto (F, z) tiene en todos los puntos de M ∩ π rango 2. Entonces lainterseccion de M con el plano z = c es una 1-variedad. Para z = 1 setiene que x3 + y3 = 0 . Equivalentemente x + y = 0 . En este caso lainterseccion es una recta.

Notemos que puesto que M contiene a una recta, entonces M no escompacta con la topologıa relativa. Aplicando que M es una subvarie-dad regular tambien se tiene que M con su topologıa subyacente es nocompacta.

Para ver que M es homeomorfa a R2 consideremos la aplicacionφ : M → R2 definida por φ(x, y, z) = (x, y) que es continua por ser

Page 82: Apuntes Geom Diferencial

76 4. SUBVARIEDADES Y VARIEDADES COCIENTE

la restriccion de una continua. Su inversa ψ : R2 → M se define comoψ(x, y) = (x, y, (1− x3 − y3)

13 ) . Notemos que

φψ(x, y) = φ(x, y, (1− x3 − y3)13 ) = (x, y)

Si un punto (x, y, z) ∈M entonces z = (1−x3− y3)13 , por lo tanto

ψφ(x, y, z) = ψ(x, y) = (x, y, (1− x3 − y3)13 ) = (x, y, z)

Luego M es homoeomorfa a R2 .d) Un punto de la recta que pasa por el origen y tiene vector di-

reccional (a, b, c) es de la forma λ(a, b, c) si ademas esta en M se tieneque λ3(a3 + b3 + c3) = 1 cuya unica solucion es λ = 1

(a3+b3+c3)13

en el

caso que (a3 + b3 + c3) 6= 0 . Si (a3 + b3 + c3) = 0 la ecuacion no tienesolucion. Por lo tanto la correspondiente recta no corta a M .

3.5. Sea F : R → R una funcion C∞ y sea q un entero mayor quecero. Definamos la funcion f : R2 → R mediante la formula f(r, s) =F (r) − sq . Consideremos el conjunto de ceros de esta funcion S =(r, s)|f(r, s) = 0 .

a) Probar que si en conjunto de soluciones de sistema de ecuacionesF = 0 , dF

dr= 0 es vacio, entonces S es una subvariedad regular de

R2 de dimension uno. Es decir una curva sin singularidades.b) Probar que si F es un polinomio sin raices multiples, entonces S

es una subvariedad regular.c) Estudiar si S es o no es subvariedad regular en los siguientes

casos

1) Tomar F (r) = r2 y q = 2 .2) Tomar F (r) = r3 y q = 3 .

3.6. Sea F : R3 −→ R la aplicacion definida por

F (x, y, z) = x4 + y4 + z4 .

a) Encontrar los puntos en los que F es una submersion.b) Probar que el subconjunto

M = (x, y, z)|x4 + y4 + z4 = 1

es una subvariedad regular de R3 .c) Teniendo en cuenta que las unicas 1-variedades conexas Haus-

dorff que existen son difeomorfas a S1 o a R . Describir las componentesconexas de la interseccion obtenida a cortar M con el plano π de deecuacion z = c en funcion de los valores de c . Probar que M es com-pacta.

d) Probar que cada recta que pasa por el origen corta a M exacta-mente en dos puntos. Probar que M es difeomorfa a la 2-esfera. Veasefigura 8

Page 83: Apuntes Geom Diferencial

LAS FIBRAS DE UNA SUBMERSION 77

Figura 8. Superficie difeomorfa a la 2-esfera

Solucion: a) La matriz jacobiana de F es la siguiente

(4x3, 4y3, 4z3)

Es facil comprobar que el conjunto de soluciones del sistema

4x3 = 0 , 4y3 = 0 , 4z3 = 0

esta formado exclusivamente por el punto (0, 0, 0) .b) Notemos que (0, 0, 0) 6∈M . Entonces se tiene que F es una sub-

mersion en todos los puntos de M . Por lo tanto M es una subvariedadregular.

c) Si consideramos la aplicacion diferenciable (F, z) : R3 → R2 setiene que la interseccion de M y el plano π de ecuacion z = c es la fibrade (1, c) . Veamos si (F, z) es una submersion, su matriz jacobiana esla siguiente: (

4x3 4y3 4z3

0 0 1

)que para −1 < c < 1 se tiene que si suponemos que simultaneamentex = 0 y y = 0 , entonces c4 = 1 pero esto es imposible ya que −1 <c < 1 . Por lo tanto (F, z) tiene en todos los puntos de M ∩ π rango 2.Entonces la interseccion de M con el plano z = c es una 1-variedad. Eneste caso se tiene que y = (1− c4 − x4)

14 o bien y = −(1− c4 − x4)

14 .

Se trata de una variedad conexa y compacta. Luego es difeomorfa a la1-esfera.

Para c = 1 se tiene que x4 + y4 = 0 . Equivalentemente x = y = 0 .En este caso la interseccion es el punto (0, 0, 1) .

Para c = −1 se tiene que x4+y4 = 0 . Equivalentemente x = y = 0 .En este caso la interseccion es el punto (0, 0,−1) .

Para |c| > 1 la intersecion es el conjunto vacio.

Page 84: Apuntes Geom Diferencial

78 4. SUBVARIEDADES Y VARIEDADES COCIENTE

Para ver que M es compacta, notemos que si x4 ≤ 1 , entonces x2 ≤1 . De la condicion x4+y4+z4 = 1 se sigue que x2+y2+z2 ≤ 3 , luego Mes un conjunto acotado. Puesto que que M es una subvariedad regularsu topologıa es la euclidiana. Ademas es un subconjunto cerrado, luegoM es una variedad compacta.

d) Un punto de la recta que pasa por el origen y tiene vector di-reccional (a, b, c) es de la forma λ(a, b, c) si ademas esta en M se tie-ne que λ4(a4 + b4 + c4) = 1 cuyas soluciones son λ = 1

(a4+b4+c4)14

y

λ = − 1

(a4+b4+c4)14

.

Para cada punto de la esfera (x, y, z) le asociamos el punto de Mdeterminado por 1

(x4+y4+c4)14

(x, y, z) . Recıprocamente, a cada punto

(x, y, z) de M le corresponde el punto 1

(x2+y2+z2)12

(x, y, z) de la esfe-

ra. Las transformaciones anteriores son aplicaciones C∞ de R3 en R3 .Ademas adecuadas restricciones a subvariedades regulales son diferen-ciables.

3.7. Para a 6= 0 sea F : R3 −→ R la aplicacion definida por

F (x, y, z) = a ez cos(a x)− cos(a y) .

a) Encontrar los puntos en los que F es una submersion.b) Estudiar si el conjunto de ceros M de F es una subvariedad

regular de R3 . Vease figura 9 . Estudiar la compacidad de M .c) Tomemos a = π/2 y supongamos que en el punto p = (0, 0,− log a)

de la superficie, sea in : M → R3 la inclusion canonica y considermos lacarta (u, v) : M → R2 dada por u = x y v = y . Notemos que

(∂∂u

)p

y(∂∂v

)p

es una base de TpM y(∂∂x

)p,(∂∂y

)p,(∂∂z

)p

es una base de TpR3 .

Calcular in∗p(∂∂u

)p

y in∗p(∂∂u

)p

en terminos de la base del espacio vec-

torial tangente a R3 . Calcular la ecuacion del espacio tangente a Men el punto p .

d) Estudiar la interseccion de la superficie M con el plano y = mx , conm 6= 0 . Estudiar el numero de componentes conexas de la interseccion.

Solucion: a) La matrız jacobiana de F es la siguiente

(−(a2 ez sen(a x)

), a sen(a y), a ez cos(a x))

Es facil comprobar que(∂F∂x

)2+(a∂F∂z

)2> 0 , entonces se tiene que

o(∂F∂x

)6= 0 o

(∂F∂z

)6= 0 . Por lo tanto es rango de la matrız jacobiana

es uno y F es submersion en todos los puntos de R3 .b) Por lo que aplicando el teorema 4.2 del capıtulo 1 o la pro-

posicion 3.2 del capıtulo 3, se obtiene que el conjunto de ceros de laecuacion a ez cos(a x)− cos(a y) = 0 es una subvariedad regular de R3

de dimension dos.

Page 85: Apuntes Geom Diferencial

LAS FIBRAS DE UNA SUBMERSION 79

Figura 9. Superficie no compacta

Notemos que para cualquier valor de z se tiene que el punto ( πa2, πa2, z)

esta en la variedad M , en consecuencia el subconjunto M no es acota-do. Puesto que M es subvariedad regular se tiene que la topologıa de Mcomo variedad coincide con la topologıa de M como subespacio de R3 .Entonces puesto que M no es acotado y los subconjuntos compactos deR3 son cerrados y acotados se sigue que la superficie M no es acotada.

c) Se tiene que

x in = u , y in = v y z in = log cos(a v)a cos(a u)

Ademas

∂ log cos(a v)a cos(a u)

∂u= a tan au

∂ log cos(a v)a cos(a u)

∂v= −a tan av

in∗p(∂∂u

)p

= in∗p(∂∂u

)px(∂∂x

)p

+ in∗p(∂∂u

)py(∂∂y

)p

+ in∗p(∂∂u

)pz(∂∂z

)p

=(∂x in∂u

)p

(∂∂x

)p

+(∂y in∂u

)p

(∂∂y

)p

+(∂z in∂u

)p

(∂∂z

)p

=(∂∂x

)p

+ (a tan au)p(∂∂z

)p

=(∂∂x

)p

y similarmente se tiene que

Page 86: Apuntes Geom Diferencial

80 4. SUBVARIEDADES Y VARIEDADES COCIENTE

in∗p(∂∂v

)p

= in∗p(∂∂v

)px(∂∂x

)p

+ in∗p(∂∂v

)py(∂∂y

)p

+ in∗p(∂∂v

)pz(∂∂z

)p

=(∂x in∂v

)p

(∂∂x

)p

+(∂y in∂v

)p

(∂∂y

)p

+(∂z in∂v

)p

(∂∂z

)p

=(∂∂y

)p− (a tan av)p

(∂∂z

)p

=(∂∂y

)p

donde en las ultimas igualdades hemos particularizado la expresioncorrespondiente en el punto (0, 0,− log a)

La ecuacion del plano tangente a M en el punto p sera

z = − log a

notemos que por los calculos anteriores se trata de un plano perpendi-cular al eje de las zetas.

d) Consideremos la funcion (F,mx − y) : R3 → R2 cuya matrızjacobiana es la siguiente:(

− (a2 ez sen(a x)) a sen(a y) aez cos(a x)m −1 0

)En los puntos que cos ax 6= 0 la matrız tiene rango dos, sin embargo

si cos ax = 0 y suponemos que (x, y, z) esta en la interseccion, entoncescos amx = 0 . De aquı se desprende que ax = π

2+kπ y max = π

2+lπ =

m(π2

+ kπ) . Lo que implica que m debe ser cociente de impares. En

este caso la interseccion contine a una recta de ecuacion x = π+2kπ2a

,

y = m(π+2kπ)2a

. Dependiendo de si k y l tienen o no la misma paridad setiene que existe un punto en la recta anterior tal que la matrız jacobianatiene rango uno. Notemos que si m no es cociente de impares la rectaanterior no corta a la superficie.

En el caso que cos ax 6= 0 , y suponiendo que a > 0, cada inter-valo ( π

2a+ kπ

a, π

2a+ kπ

a+ π

a) contiene al abierto (posiblemente vacio)

x| cos amxa cos ax

> 0 que es una reunion finita de intervalos de la forma

(max π2a

+ kπa, π

2am+ lπ

ma,mın π

2a+ kπ

a+ π

a, π

2ma+ lπ

ma+ π

ma)

Cada uno de estos intervalos abiertos es difeomorfo a una compo-nente conexa de la interseccion, que sera no compacta. En definitiva lainterseccion siempre tiene un numero contable de componentes conexasno compactas.

3.8. Sea F : R3 −→ R la aplicacion definida por

F (x, y, z) = x2y2z2 − 4

a) Encontrar los puntos en los que F es una submersion.b) Estudiar si el conjunto de ceros M de F es una subvariedad

regular de R3 . Vease figura 10 . ¿Es M una variedad compacta? .

Page 87: Apuntes Geom Diferencial

LAS FIBRAS DE UNA SUBMERSION 81

c) Estudiar la interseccion de la superficie M con el plano z = a .Analizar el numero de componentes conexas de la interseccion.

d) Probar que M es difeomorfa a la 2-esfera menos su intersec-cion con los planos coordenados. ¿Cuantas componetes conexas tienela variedad M? Argumentar la respuesta.

Figura 10. Superficie difeomorfa a la 2-esfera unidadmenos los planos coordenados

Solucion: a) La matrız jacobiana de F es la siguiente

(2xy2z2, 2x2yz2, 2x2y2z)

Notemos que 2xy2z2 = 0 si y solo si 2x2yz2 si y solo si 2x2y2z = 0si y solo si xyz = 0 . Es decir si y solo si el punto esta en la reunion delos planos coordenados que tienes por ecuaciones x = 0 , y = 0 , z = 0 .Es decir F es submersion en (x, y, z)|x 6= 0 e y 6= 0 y z 6= 0

b) Notemos que si en un punto (x, y, z) la funcion F no es submer-sion entonces xyz = 0 y se tiene que F (x, y, z) = −4 por lo que noes un cero de la funcion F . Entonces F es submersion en cada unode sus ceros y aplicando el teorema 4.2 del capıtulo 1 o la proposicion3.2 del capıtulo 3, se obtiene que el conjunto de ceros de la ecuacionx2y2z2 − 4 = 0 es una subvariedad regular de R3 de dimension dos.

Puesto que M es subvariedad regular se tiene que la topologıa de Mcomo variedad coincide con la topologıa de M como subespacio de R3 .Entonces si vemos que M no es acotado puesto que los subconjuntoscompactos de R3 son cerrados y acotados, obtendremos que la superficieM no es compacta.

Si tomamos z = 1 , entonces x = ± 2y

con y 6= 0 . Cuando y → 0

se tiene que x → ∞ . De donde se concluye que la variedad M no esacotada.

Page 88: Apuntes Geom Diferencial

82 4. SUBVARIEDADES Y VARIEDADES COCIENTE

c) En el caso que a = 0 se tiene que F (x, y, 0) = −4 . Luego elplano z = 0 no corta a la superficie. Si a 6= 0 se tiene que x2y2 = 4

a2 o

equivalentemente (xy − 2a)(xy + 2

a) = 0 . Este conjunto es la reunion

disjunta de las hiperbolas xy − 2a

= 0 , xy + 2a

= 0 . Cada una de ellastiene dos componentes conexas. Por lo que la interseccion tiene un totalde cuatro componentes conexas. Ello puede verse geometricamente enla parte superior de la figura 10 .

d) Un punto de la recta que pasa por el origen y tiene vector di-reccional (a, b, c) es de la forma λ(a, b, c) si ademas esta en M se tie-

ne que λ6(a2b2c2) = 4 cuyas soluciones son λ =(

4(a2b2c2)

) 16

y λ =

−(

4(a2b2c2)

) 16

.

Para cada punto de la esfera (a, b, c) tal que abc 6= 0 le asociamos

el punto de M determinado por(

4(a2b2c2)

) 16

(a, b, c) . Recıprocamente,

a cada punto (x, y, z) de M le corresponde el punto 1

(x2+y2+z2)12

(x, y, z)

de la esfera. Las transformaciones anteriores son aplicaciones C∞ deR3 en R3 . Ademas adecuadas restricciones a subvariedades regularesson diferenciables.

Es facil ver que la 2-esfera menos su interseccion con los planoscoordenados tiene ocho componentes conexas. Tambien se puede probarque cada una de ellas es difeomorfa a R2 . Puesto que la variedadM es difeomorfa a la 2-esfera menos su interseccion con los planoscoordenados se concluye que tiene ocho componentes conexas.

Page 89: Apuntes Geom Diferencial

CAPıTULO 5

GRUPOS DE TRANSFORMACIONES

En 1872, Klein sistematizo la geometrıa usando la teorıa de gru-pos en algo que se llamo el Programa de Erlangen. Para Klein unageometrıa consistıa en el estudio de las propiedades de figuras que semantienen invariantes cuando se aplica un grupo de transformaciones.

La teorıa de grupos permitıa la sıntesis de los trabajos algebraicos ygeometricos de Monge, Poncelet, Gauss, Cayley, Clebsch, Grassmann yRiemann. Debe mencionarse en todos estos resultados la colaboraciondel gran matematico noruego: Sophus Lie (1842-1899), que junto conKlein comprendieron la importancia del uso de los grupos; de hecho,mientras Klein enfatizo resultados en en caso de los grupos disconti-nuos, Lie lo hizo en los continuos.

Este capıtulo esta dedicado al estudio de algunas propiedades delos grupos de transformaciones, la primera seccion se aborda el estudiode los grupos discontinuos y la segunda el de los grupos continuos.

1. Grupos discontinuos y variedades recubridoras

En esta seccion llamaremos transformacion de una variedad M aun difeomorfismo global y sobreyectivo de M en M . Denotaremos porDif(M) el grupo de todas las transformaciones de M .

Definicion 1.1. Una accion de un grupo G en una variedad M esuna aplicacion φ : G ×M → M tal que φ(1, p) = p , φ(g, φ(h, p)) =φ(gh, p) y ademas φ(g) : M → M , definida por φ(g)(p) = φ(g, p) , esdiferenciable. Si no hay lugar a confusion denotaremos φ(g, p) = g · p ya la transformacion φ(g) por φg . Se dice que la accion es eficiente sig · p = p para todo p implica que g = 1 . Diremos que es libre cuandoverifica que si para un p ∈ M se tiene que g · p = p , entonces g = 1 .Se dice que una accion es discontinua si para cada p ∈ M existe unentorno abierto U tal que si g · U ∩ U 6= ∅ entonces g = 1 .

En el caso que utilicemos notacion aditiva en el grupo G las pro-piedades anteriores se escriben equivalentemente del modo siguiente:φ(0, p) = p , φ(g, φ(h, p)) = φ(g+h, p) , esta notacion sera utilizada enalgunos ejemplos.

Observese que una accion discontinua es libre y una libre es efi-ciente. Una accion si no es eficiente se le puede asociar una eficiente

83

Page 90: Apuntes Geom Diferencial

84 5. GRUPOS DE TRANSFORMACIONES

dividiendo el grupo por el subgrupo normal formado por los elemen-tos no eficientes. Si en G se considera una estructura de variedad 0-dimensional entonces φ : G × M → M es diferenciable. Una accionφ : G × M → M determina un homomorfismo φ : G → Dif(M) yrecıprocamente. Si la accion es eficiente entonces φ es monomorfismo yse puede considerar que G es un subgrupo de Dif(M) . Para las accio-nes eficientes tambien se dice que G es un grupo de transformacionesy, en su caso, que es un grupo de transformaciones discontinuo.

Si un grupo G actua en una variedad M , entonces diremos quep, p′ ∈ M estan en la misma orbita si existe un g ∈ G tal que g · p =p′ . Denotaremos por G\M el espacio de todas las orbitas de M y laproyeccion canonica se denotara por pr : M → G\M . La orbita de pse denotara por pr(p) y tambien por G · p .

Proposicion 1.1. Si G actua discontınuamente en una variedadM , entonces G\M admite una unica estructura de variedad tal quepr : M → G\M es un difeomorfismo local. Ademas la proyeccion canoni-ca es una aplicacion recubridora regular.

Demostracion. Por ser G discontinuo, podemos encontrar unatlas A tal que si x ∈ A y g ∈ G , g 6= 1 , entonces g(Domx)∩Domx =∅ . Notese que pr |Domx es inyectiva, continua y abierta. Consideremosen G\M la siguiente familia de cartas x(pr |Domx)

−1|x ∈ A que veri-fica que la reunion de sus dominios es G\M .

Ahora supongamos que x, y ∈ A y veamos si el cambio

y(pr |Dom y)−1(x(pr |Domx)

−1)−1 = y(pr |Dom y)−1(pr |Domx)x

−1

es diferenciable. Sea p ∈ (Domx) ∩ (∪g∈Gg · Dom y) , entonces existeun unico g ∈ G tal que g · p = (pr |Dom y)

−1 pr(p) . Entonces Domx ∩φ−1g Dom y es un entorno abierto de p tal que

(pr |Dom y)−1(pr |Domx)|Domx∩φ−1

g Dom y = φg|Domx∩φ−1g Dom y .

De aquı se obtiene que (pr |Dom y)−1(pr |Domx) es diferenciable. Luego el

cambio de cartas es diferenciable.Dado p ∈ M , sea x una carta de A tal que p ∈ Domx . En-

tonces se tiene que x(pr |Domx)−1(pr |Domx)x

−1 = id |Codomx . Luegopr : M → G\M es un difeomorfismo local. La unicidad de tal estructuradiferenciable se deduce de modo inmediato del hecho que la proyeccionsea una submersion.

La ultima parte en un resultado bien conocido en la teorıa de espa-cios recubridores.

Proposicion 1.2. Si la aplicacion f : X →M es un homeomorfis-mo local de un espacio topologico X en una variedad M , entonces Xadmite una unica estructura diferenciable de modo que f : X → M esun difeomorfismo local. Si X es una cubierta conexa regular de M

Page 91: Apuntes Geom Diferencial

1. GRUPOS DISCONTINUOS Y VARIEDADES RECUBRIDORAS 85

entonces M se obtiene de X como el espacio de orbitas de la acciondiscontinua del grupo de transformaciones de la aplicacion recubridora.

Demostracion. Sea U un cubrimiento abierto de X tal que f(U)es abierto para cada U ∈ U , f(U) = Dom(xU) , donde xU es una cartade M , y la funcion f |U es un homeomorfismo. La familia de cartasxU(f |U)|U ∈ U verifica que la reunion de sus dominios es X .

Notese que

yV (f |V )(xU(f |U))−1 = yV (f |V )(f |U)−1x−1U

= yV (id |f(U∩V ))x−1U = (yV xU

−1)

es diferenciable. Luego el cambio de cartas es diferenciable.La unicidad se deduce de modo inmediato del hecho que la proyec-

cion sea una inmersion global.La ultima parte en un resultado bien conocido en la teorıa de espa-

cios recubridores.

Proposicion 1.3. Si G actua discontınuamente en una variedadM y dados p, p′ en orbitas distintas existen entornos abiertos U,U ′ dep y p′ tal que para todo g ∈ G , g · U ∩ U ′ = ∅ , entonces M y G\Mson variedades Hausdorff.

Demostracion. Notemos que si para todo g ∈ G , g ·U ∩U ′ = ∅ ,entonces tambien se tiene que para todo g, g′ ∈ G , g · U ∩ g′ · U ′ = ∅ .Dados dos orbitas distintas G · p 6= G · p′ . Entonces (∪g∈Gg · U) ∩(∪g′∈Gg′ ·U) = ∅ . Ello implica que G · p,G · p′ tienen entornos abiertoscon interseccion vacıa. Luego G\M es Hausdorff.

Para ver que M es Hausdorff tomemos p, p′ ∈ M , p 6= p′ . Si py p′ estan en distintas orbitas, para separar los puntos basta tomarlos entornos U,U ′ cuya existencia asegura la hipotesis. Si estan en lamisma orbita, entonces aplicando la definicion de accion discontinua seencuentra un entorno U de p tal que para cierto g ∈ G se tiene queg · U es un entorno de p′ y ademas U ∩ g · U = ∅ .

Definicion 1.2. Un subconjunto S = F se llama dominio funda-mental si es la clausura de un subconjunto F que tenga exactamenteun representante de cada orbita. Un dominio fundamental F se diceque es normal si cada punto p ∈ M tiene un entorno abierto V talque solo existen un numero finito de elementos g ∈ G de modo queg · V ∩ F 6= ∅ .

Proposicion 1.4. Sea∼ \F el cociente inducido en F por la acciondiscontinua de un grupo G en una variedad M . Entonces la biyeccion∼ \F → G\M es continua. Si ademas F es normal, entonces dichabiyeccion es un homeomorfismo.

Demostracion. De la propiedad universal del cociente se deduceinmediatamente que la biyeccion ∼ \F → G\M es continua. Veamos

Page 92: Apuntes Geom Diferencial

86 5. GRUPOS DE TRANSFORMACIONES

que tambien es abierta. Sea p ∈ F y sea V un entorno abierto de p enM tal que F ∩ V es saturado respecto la relacion inducida ∼ . Por serel dominio fundamental normal existe un entorno abierto W tal queg ·W ∩ F = ∅ para todo g salvo un conjunto finito. Ello implica queexiste un conjunto finito g1, · · · , gr tal que g1 · p, · · · , gr · p ∈ F y sig 6∈ g1, · · · , gr entonces g · p 6∈ F . Puesto que p esta en el saturadoF ∩ V , se tiene que g1 · p, · · · , gr · p ∈ V . Teniendo en cuenta queφg1 , · · · , φgr son continuas y que F es normal se deduce que existe unentorno abierto U de p en M tal que g1 · U ⊂ V, · · · , gr · U ⊂ V y sig 6∈ g1, · · · , gr entonces g ·U∩F = ∅ . Entonces p ∈ F ∩(∪g∈Gg ·U) =(F ∩ g1 · U) ∪ · · · ∪ (F ∩ gr · U) ⊂ F ∩ V . Luego ∼ \F → G\M esabierta.

Ejemplo 1.1. El grupo aditivo de los enteros Z actua libre y dis-contınuamente como un grupo de transformaciones en R si su accionesta determinada por la funcion:

Φ: Z× R −→ R

definida por Φ(n, s) = n + s . En este caso se puede tomar comodominio fundamental el subconjunto compacto [0, 1] , que es la clausurade [0, 1) , subconjunto con un representante de cada orbita.

Problemas

1.1. Sea G un grupo de transformaciones sobre una variedad M .Si φ(g) denota la transformacion correspondiente al elemento g ∈ G,demostrar que φ(g−1) = (φ(g))−1.

1.2. Sea Φ: G×M −→M la accion de un grupo G en una variedadM . Demostrar queG tambien actua a la derecha mediante la aplicacionΦ′ : M ×G −→M definida por Φ′(m, g) = Φ(g−1,m).

1.3. Demostrar que el grupo aditivo de los enteros Z actua librey discontinuamente como un grupo de transformaciones en R2 si suaccion esta determinada por la funcion:

Φ: Z× R2 −→ R2

definida por Φ(n, (z1, z2)) = (n+ z1, (−1)nz2).

Solucion: Para cada (a, b) punto de R2 consideremos el entornoabierto B × R , donde B es la bola de centro a y radio 1

2. Su-

pongamos que existe un elemento en la interseccion de B × R y deΦ(n, (B×R)) . Entonces existen b, b′ ∈ B tales que b = b′+ n . Enton-ces |n| ≤ |b− b′| < 1 y n entero implica que n = 0 . Por lo tanto Φ esuna accion discontinua.

1.4. Demostrar que el grupo aditivo de los enteros Z×Z actua librey discontinuamente como un grupo de transformaciones en R×R si su

Page 93: Apuntes Geom Diferencial

GRUPOS DISCONTINUOS Y VARIEDADES RECUBRIDORAS 87

accion esta determinada por la funcion:

Φ: (Z× Z)× R2 −→ R2

definida por Φ((n1, n2), (r1, r2)) = (n1 + r1, n2 + r2) .

1.5. Demostrar que el grupo G = 〈a, b|ab = ba−1〉 actua libre ydiscontinuamente como un grupo de transformaciones en R × R si suaccion esta determinada por la funcion:

Φ: G× R2 −→ R2

definida sobre los generadores por

Φ(a, (r1, r2)) = (r1 + 1, r2) ,

Φ(b, (r1, r2)) = (−r1, r2 + 1).

1.6. Sea Z el grupo aditivo de los numeros enteros, demostrar quela funcion:

Φ: Z× Z× R −→ Rdefinida por Φ(m,n, s) = s + αm + βn, siendo α y β numeros realesno nulos cuya razon es irracional, determina una accion de Z × Z enR como grupo de transformaciones. Demostrar que esta accion es librepero no discontinua.

1.7. Demostrar que un grupo finito que actua libremente como gru-po de transformaciones en una variedad Hausdorff M , actua tambiendiscontinuamente. Probar que en este caso la variedad cociente es Haus-dorff.

Solucion: Supongamos que G = g1, · · · gn con g1 = 1 . Sea ahoraun punto p de M . Puesto que la accion es libre se tiene que parai, j ∈ 1, · · · , n ,i 6= j , gip 6= gjp . Aplicando que M es Hausdroff porinduccion se obtiene que existen entornos abiertos Ui de gip tal que parai 6= j , Ui∩Uj = ∅ . Teniendo en cuenta que cada Φgi es continua se tieneque existe un entorno U de p tal que para i ∈ 1, · · · , n ,giU ⊂ Ui .Entonces U ∩ giU ⊂ U1 ∩ Ui = ∅ para i ∈ 2, · · · , n . Por lo tanto laaccion es discontinua.

Si tenemos dos puntos distintos p 6= p′ de modo que p′ no esta en laorbita de p sabemos que existe un entorno U de p tal que U ∩ gU = ∅para g 6= 1. Puesto que p′ 6= gip y la orbita de p es finita se puedereducir el tamano de U y encontrar U ′ tal que U ′∩giU = ∅ . Aplicandola proposicion 1.3 se obtiene que la variedad cociente es Hausdorff.

1.8. Sea M1 la variedad sobre el subespacio H de R2, que consistede los puntos (x, 0) (x ∈ R), junto con el punto (0, 1), definida por lascartas α y α1 cuyos dominios respectivos son U = (x, 0)|x ∈ R yU1 = (x, 0)|x 6= 0

⋃(0, 1)α : U −→ R ; α(x, 0) = x

α1 : U1 −→ R ;α1((x, 0)) = x; α1(0, 1) = 0.

Page 94: Apuntes Geom Diferencial

88 5. GRUPOS DE TRANSFORMACIONES

a) Demostrar que la funcion: Φ: M1 −→M1 definida por:

Φ(s, 0) = (−s, 0) s 6= 0 ; Φ(0, 0) = (0, 1) ;φ(0, 1) = (0, 0)

es una transformacion en M1

b) Demostrar que el grupo G, que consta de la transformacion iden-tidad y de Φ, actua sobre M1 como grupo de transformaciones. Com-probar que este grupo finito actua libremente pero no discontinuamentesobre M1.

Notar que este ejemplo demuestra que un grupo de transformacionesfinito que actua libremente en una variedad que no es Hausdorff, no esnecesariamente discontinuo.

1.9. Demostrar que el conjunto (r1, r2) ∈ R|0 ≤ r1 ≤ 1 es undominio fundamental normal para la relacion de equivalencia en R2

inducida por el grupo de transformaciones del problema 1.3 .

Solucion: Denotemos por E(a) la parte entera de un numero real.Entonces la orbita de (a, b) tiene un unico representante enF = (r1, r2) ∈ B × R|0 ≤ r1 ≤ 1 que es (a − E(a), (−1)E(a)b) .Vea-mos que F es normal. Sea (a,b) y tomemos como entorno abiertouno de la forma B × R , donde B es la bola de centro a y de ra-dio 1. Entonces si (r, s) ∈ B × R se tiene que a − 1 < r < a + 1 .Si para un entero n se verifica que 0 ≤ r + n ≤ 1 , se deduce que−a − 1 < −r ≤ n ≤ −r + 1 ≤ −a + 2 . Por lo tanto solo un numerofinito de enteros n satisface la desigualdad anterior. Ello implica que eldominio fundamental es normal.

1.10. Demostrar que el cuadrado unidad [0, 1]× [0, 1] es un dominiofundamental normal para la relacion de equivalencia en R2 inducidapor el grupo de transformaciones del problema 1.4.

1.11. Demostrar que el cuadrado unidad [−12, 1

2] × [−1

2, 1

2] es un

dominio fundamental normal para la relacion de equivalencia en R2

inducida por el grupo de transformaciones del problema 1.5.

1.12. Considerar las acciones discontinuas:

φ : Z× R2 → R2, φ(z, (a, b)) = (2z + a, b)

ψ : Z× R2 → R2, ψ(z, (a, b)) = (z + a, (−1)zb)

Denotemos por φ\R2 , ψ\R2 los espacios de orbitas de dichos grupos.a) Probar que existe f : φ\R2 → ψ\R2 tal que pf = q donde

p : R2 → φ\R2 y q : R2 → ψ\R2 son las proyecciones canonicas.b) Probar que f es un difeomorfismo local y que cada fibra de f

tiene dos puntos.

Solucion: De la ecuacion

φ(z, (a, b)) = (2z + a, b) = ψ(2z, (a, b))

Page 95: Apuntes Geom Diferencial

2. SISTEMAS DINaMICOS 89

se deduce que la q id : R2 → R2 factoriza a travEs de φ\R2 de modoque existe una unica f : φ\R2 → ψ\R2 tal que pf = q id . Puesto que pes una submersion se tiene que f es diferenciable. Ademas de que p, qsean difeomorfismos se obtiene que f tambien lo es.

2. Sistemas dinamicos

Definicion 2.1. Un sistema dinamico diferenciable consiste en unavariedad M junto con una accion diferenciable φ : R×M →M

Teniendo en cuenta que en R se utiliza notacion aditiva las con-diciones que debe satisfacer φ para ser accion se pueden expresar delmodo siguiente:

(i) ϕ(0, p) = p, ∀p ∈M ,(ii) ϕ(t, ϕ(s, p)) = ϕ(t+ s, p), ∀p ∈M, ∀t, s ∈ R .

En los sistemas dinamicos las orbitas tambien se denominan tra-yectorias.

Ejemplo 2.1. Sea M = R y consideremos la accion ϕ : R×R→ Rdada por φ(r, s) = r+ s . En este caso la accion es libre y hay solo unaorbita.

Ejemplo 2.2. Sea M = R y consideremos la accion φ : R×R→ Rdada por φ(r, s) = ers . En este caso la accion es eficiente pero no librey hay tres orbitas.

2.1. La resolucion de un sistema lineal holonomo 2x2. Re-cordemos los sistemas de ecuaciones diferenciales lineales en dimensiondos junto con una descripcion de las soluciones

Sea x = Ax, con A ∈ M(2 × 2,R) el sistema. Para resolverlo dis-tinguiremos tres casos:

1. Dos autovalores reales distintos. Sean v1 y v2 autovectores aso-ciados, respectivamente, a los autovalores λ1 y λ2, λ1 6= λ2 . Entoncesx = c1v1e

λ1t + c2v2eλ2t ; c1, c2 ∈ R .

2. Un autovalor real doble. Sea λ el autovalor con multiplicidad dos.Se pueden distinguir dos casos:

(a) Existen dos autovectores, v1 y v2 , linealmente independientesasociados a λ. En este caso la solucion general es x = c1v1e

λt+c2v2eλt =

(c1v1 + c2v2)eλt ; c1, c2 ∈ R(b) Existe un unico autovector v linealmente independiente asociado

a λ. En este caso x = veλt es solucion particular y existe otra dela forma x = (vt + u)eλt donde u ∈ R2 es un vector que habra quedeterminar sustituyendo en el sistema. La solucion general sera en estecaso x = c1ve

λt + c2(vt+ u)eλt = (c1v + c2(vt+ u))eλt ; c1, c2 ∈ R .3. Dos autovalores complejos. Sean λ = α + iβ y λ = α − iβ los

autovalores y v = a+ib y v = a−ib los autovectores asociados. Entoncesφ1(t) = Re(veλt) = (a cos βt− b sen βt)eαt

Page 96: Apuntes Geom Diferencial

90 5. GRUPOS DE TRANSFORMACIONES

-1.0 -0.5 0.0 0.5 1.0

-1.0

-0.5

0.0

0.5

1.0

Figura 1. Flujo asociado a valores propios distintos delmismo signo y contorno de la funcion de Morse asociada

-1.0 -0.5 0.0 0.5 1.0

-1.0

-0.5

0.0

0.5

1.0

Figura 2. Punto silla asociada a valores propios de dis-tinto signo y contorno de la funcion de Morse asociada

φ2(t) = Im(veλt) = (a sen βt+ b cos βt)eαt son soluciones particula-res linealmente independientes. Su solucion general sera: x = c1φ1(t) +c2φ2(t) = ((a cos βt− b sen βt)c1 + (a sen βt+ b cos βt)c2)eαt con c1, c2 ∈R.

Notemos que los anteriores casos dan lugar a las siguientes acciones.

Ejemplo 2.3. Sea M = R2 y consideremos la accion φ : R×R2 →R2 dada por φ(r, (r1, r2)) = (eλ1rr1, e

λ2rr2). Este caso corresponde aque es sistema tienen dos valores propios distintos λ1 6= λ2, ver Figuras2.1, 2.1.

Ejemplo 2.4. El el caso de tener un solo valor propio tenemos doscasos:

(a) Sea M = R2 y consideremos la accion φ : R × R2 → R2 dadapor φ(r, (r1, r2)) = (eλrr1, e

λrr2) = eλr(r1, r2). Ver Figura 2.1.(b) Ahora tomemos la accion φ : R×R2 → R2 dada por φ(r, (r1, r2)) =

eλr(r1 + rr2, r2). Ver Figura 2.1.

Page 97: Apuntes Geom Diferencial

SISTEMAS DINAMICOS 91

Figura 3. Punto de equilibrio no radial asociado a unsolo valor propio

Figura 4. Un foco estable asociado con dos valores pro-pios conjugados

Ejemplo 2.5. La siguiente accion corresponde con el caso de valo-res propios conjugados. Sea M = R2 y consideremos la accion φ : R ×R2 → R2 dada por φ(r, (r1, r2)) = eαr(r1cosβr+ r2 sen βr,−r1 sen βr+r2 cos βr) .

a) Si α < 0 , tenemos un foco estable, ver la Figura 2.1 (Si α > 0,un foco inestable).

b) Si α = 0, tenemos un centro estable , ver la Figura 2.1 .

En estos caso no existe una funcion de Morse.

Problemas

2.1. Sea M = C y consideremos la accion ϕH : R × C → R dadapor φ(r, u) = r + u . Probar que en este caso la accion es libre pero nodiscontinua, ver Figura 2.1

2.2. Sea M = C y consideremos la accion ϕV : R×C→ R dada porφ(r, u) = ir + u . Estudiar si la accion es eficiente, libre o discontinua.

Page 98: Apuntes Geom Diferencial

92 5. GRUPOS DE TRANSFORMACIONES

Figura 5. Un foco estable asociado con dos valores pro-pios puros imaginarios conjugados

Figura 6. Flujo horizontal

Figura 7. Fuente

2.3. Sea M = C y consideremos la accion φ : R × C → C dadapor φ(r, u) = eru . Probar que la accion es eficiente pero no libre yel origen es una trayectoria que solo tiene un punto y las demas sonhomeomorfas a R, ver Figura 2.1.

2.4. Sea M = C y consideremos la accion φ : R × C → C dadapor φ(r, u) = eiru . Probar que la accion es eficiente pero no libre y

Page 99: Apuntes Geom Diferencial

3. GRUPOS DE LIE 93

el origen es una trayectoria que solo tiene un punto y las demas sonhomeomorfas a S1 .

2.5. Sea M = S1 = z ∈ C||z| = 1 y consideremos la accionφ : R × S1 → S1 dada por φ(r, u) = eiru . Probar que la accion no eseficiente y hay una sola trayectoria circular.

2.6. Sea M = C y z = a + bi ∈ C consideremos la accion φC : R×C → C dada por φ(r, u) = erzu . Probar que si z = 0 todas lastrayectorias son unipuntuales, si z = 1 se obtiene el ejemplo anteriorde trayectorias radiales, si z = i, se tiene el caso de hojas circulares yfinalmente si a 6= 0, tendremos un caso con una trajectoria puntual ylas demas formadas por espirales.

2.7. SeaM = R2 y consideremos la aplicacion ψ : Z×(R2\(0, 0) →R2 \ (0, 0) dada por

ψ(z, (r1, r2)) = eαz(r1 cos βz + r2 sen βz,−r1 sen βz + r2 cos βz) .

a) Probar que ψ es una accion y estudiar el tipo de las orbitassegun los valores de α y β

b) Estudiar si la accion ψ es eficiente, libre o discontinua paraα = 1, β = 0 y para α = 0 β = 2π/3 .

c) Probar que para α = 1, β = 0 el espacio de orbitas es difeo-morfo a un toro y para α = 0 β = 2π/3 es difeomorfo a uncilindro.

3. Grupos de Lie

El matematico Sophus Lie (1849-1899) inicio el estudio de grupos detransformaciones continuas que dejan invariantes las soluciones de lossistemas de ecuaciones diferenciales. Observo que complicadas condi-ciones no lineales para la invarianza se podıan sustituir por condicioneslineales de invarianza infinitesimal, vease [30].

El metodo de Lie de la transformacion infinitesimal proporciona unatecnica ampliamente aplicable para hallar soluciones en forma analıticaa ecuaciones diferenciales ordinarias. Aplicados a ecuaciones diferen-ciales en derivadas parciales, el metodo de Lie proporciona solucionesinvariantes y leyes de conservacion. Explotando las simetrıas de lasecuaciones se pueden obtener nuevas soluciones a partir de las ya co-nocidas, y las ecuaciones se pueden clasificar en clases de equivalencia.

Por otro lado hemos de resaltar el importante papel que jueganlos grupos de Lie en mecanica cuantica en el estudio de las diferentespartıculas y sus interacciones.

Definicion 3.1. Un grupo de Lie G es un grupo cuyo conjuntosubyacente tiene una estructura de variedad diferenciable, de tal formaque la aplicacion producto π : G×G→ G, (g, h)→ gh y la que asociaa cada elemento su inverso, ι : G→ G, ι(g) = g−1 , son diferenciables.

Page 100: Apuntes Geom Diferencial

94 5. GRUPOS DE TRANSFORMACIONES

Proposicion 3.1. Sea G un grupo cuyo conjunto subyacente tieneuna estructura de variedad diferenciable y sea ψ : G×G → G la apli-cacion ψ(g, h) = gh−1 . Entonces π, ι son diferenciables si y solo si ψes diferenciable.

Demostracion. Si π, ι son diferenciables, entonces claramente ψ =p(id×ι) es diferenciable. Recıprocamente, si ψ es diferenciable, se tieneque ι = ψ(1, id) es diferenciable y por tanto π = ψ(id×ι) tambien loes.

Observacion 3.1. Para desarrollar buena parte de la teorıa, soloes necesario tomar derivadas de primer orden en G y en el fibradotangente TG , por lo tanto es nesesario suponer que G y π son de claseC2 . Sin embargo, no existe perdida de generalidad en suponer que Gy π son analıticas Cω , pues es posible probar que si π es clase C1

entonces π es analıtica en relacion a la estructura de variedad analıticacontenida en una estructura Ck , 1 ≤ k ≤ ∞ .

Dado g ∈ G , las traslaciones a irquierda y a derecha Lg : G → Gy Rg : G→ G , estan definidas respectivamente por

Lg(h) = gh , Rg(h) = hg .

Estas aplicaciones son diferenciables pues Lg = π(g, id) , Rg = π(id, g) .En realidad, ambas traslaciones, a idquierda y a derecha, son difeomor-fismos globales y suprayectivos ya que LgLg−1 = id , RgRg−1 = id .

En la definicion de grupo de Lie no es necesario exigir que la inver-sion ι : G → G sea diferenciable. Esto hecho es una consecuencia delteorema de la funcion implicita.

Proposicion 3.2. Sea G un grupo cuyo conjunto subyacente tieneuna estructura de variedad diferenciable, de tal forma que a aplica-cion producto π : G × G → G, (g, h) → gh es diferenciable, enton-ces ι : G → G es diferenciable. Ademas la aplicacion tangente Tιg =−(T1Lg−1)(TgRg−1) . En particular Tι1 = − idT1G , donde 1 denota elelemento neutro.

Demostracion. Consideremos la aplicacion producto π : G×G→G , π(g, h) = gh. Si aplicamos la proposicion 4.2 del capıtulo 3 se ob-tiene que T(g,h)π ∼= TgRh ⊕ ThLg . Tomando una carta x en el pun-to g, otra carta y en el punto h y una carta z en gh , se tiene que

la matriz jacobiana de TgRh es precisamente

((∂zi∂xj

)(g,h)

)que es no

singular. Para el caso que π(g, h) = gh = 1, ahora podemos apli-car el teorema de la funcion implıcita 4.1 del capıtulo 2 para poderasegurar que en un entorno abierto de h la aplicacion ι(h) = h−1 esC∞ . De donde se sigue que ι es C∞ . Notemos que ademas se veri-fica la igualdad π(ι(h), h) = 1 . Aplicando la regla de la cadena, sededuce la ecuacion (Tg−1Rg)(Tgι) + TgLg−1 = 0. De aquı se sigue que

Page 101: Apuntes Geom Diferencial

3. GRUPOS DE LIE 95

Tgι = −(Tg−1Rg)−1TgLg−1 = −(T1Rg−1)TgLg−1 . Para el caso g = 1, se

tiene que T1ι = − idT1G .

Ejemplo 3.1. El espacio Rn es un grupo de Lie abeliano con lasuma habitual de vectores.

Ejemplo 3.2. El espacio R∗ de los reales no nulos es un grupode Lie abeliano no conexo con el producto. Tambien los complejos nonulos C∗ con el producto es un grupo de Lie abeliano no conexo.

Ejemplo 3.3. El grupo discreto G es un grupo de Lie, es este casose considera una estructrura diferenciable 0-dimensional.

Ejemplo 3.4. Sea GL(n,R) el grupo de las transformaciones li-neales de Rn que es un abierto de la variedad de las matrices realesM(n,R) , veanse los ejemplos 3.7 y 3.8 del capıtulo 2. El productode las matrices A = (aik) , B = (bkj) es la matriz C = (cij) , dondecij =

∑k aikbkj . Es decir cij viene dado por una funcion polinomi-

ca de grado dos en las variables aik, bkj . Por lo que es una funciondiferenciable. Por esta razon GL(n,R) es un grupo de Lie.

Ejemplo 3.5. Analogamente GL(n,C) el grupo de las transforma-ciones lineales de Cn tiene estructura de grupo de Lie.

Ejemplo 3.6. Sean G,H grupos de Lie, el producto cartesiano conlas estructuras diferenciables y de grupo producto, es tambien un grupode Lie. Por ejemplo se puede ver facilmente que Z/2×R es isomorfo aR∗ .

La nocion de morfismo se define de modo natural en la categorıade los grupos de Lie.

Definicion 3.2. Supongamos que G,G′ son grupos de Lie, un ho-momorfismo de grupos de Lie es un homomorfismo de grupos f : G→G′ que ademas es diferenciable. Diremos que f es un isomorfismo siexiste la aplicacion inversa f−1 : G′ → G y esta es diferenciable.

Ejemplo 3.7. Consideremos los grupos de Lie R, R∗ estudiados enlos ejemplos 3.1 , n = 1 , y 3.3 . La aplicacion exponcial exp: R→ R∗es un homomorfismo de grupos de Lie.

Definicion 3.3. Sean G,H grupos de Lie y supongamos que Hes un subgrupo de G . Diremos que H es un subgrupo de Lie G si lainclusion es un encaje y si ademas es un encaje regular diremos que Hes un subgrupo de Lie regular de G .

Proposicion 3.3. Sea G un grupo de Lie y sea H un subgrupoque que ademas es subvariedad regular de G . Entonces H es un grupode Lie.

Page 102: Apuntes Geom Diferencial

96 5. GRUPOS DE TRANSFORMACIONES

Demostracion. Sea in : H → G la inclusion canonica. Conside-remos el diagrama conmutativo

G×GψG // G

H ×HψH //

in× in

OO

H

in

OO

Puesto que ψG es diferenciable (vease la proposicion ??) y in : H → Ges un encaje regular se tiene que por la proposicion 1.3 del capıtulo 4que ψH es diferenciable. Por lo tanto H es un grupo de Lie.

Ejemplo 3.8. La 1-esfera S1 = z ∈ C∗||z| = 1 consideradacomo los complejos de modulo 1, es un subrupo de C∗ y ademas es unasubvariedad regular. Entonces S1 es un grupo de Lie 1-dimensional,abeliano, conexo y compacto.

Ejemplo 3.9. Sea SL(n,R) = A ∈ GL(n,R)|det(A) = 1 el gru-po especial lineal. Para ver que es una subvariedad regular de GL(n,R)veamos que det : GL(n,R) → R es una submersion en los puntos deSL(n,R) . Para ello vamos a utilizar la caracterizacion de submer-siones dada en la proposicion 2.1 del capıtulo 4 . Sea entonces unaA ∈ det−1(1) = SL(n,R), sea Ar la matriz obtenida multiplicandopor r la primera fila. Notemos que s : R∗ → GL(n,R) , s(r) = Ar esdiferenciable. Ademas s(1) = A y dets = id por lo tanto det es unasubmersion en A para cada A . Entonces por la proposicion 3.2 delcapıtulo 4 se tiene que det−1(1) = SL(n,R) es una subvariedad regularde dimension n2−1 y por la proposicion anterior se sigue que SL(n,R)es un grupo de Lie .

Ejemplo 3.10. Sea O(n,R) = A ∈ GL(n,R)|ATA = I el grupoortogonal que es un subgrupo de GL(n,R) , donde AT denota la matriztraspuesta de A e I es la matriz identidad . Para ver que es una sub-variedad regular de GL(n,R) veamos que sim: GL(n,R) → S(n,R)es una submersion en los puntos de O(n,R) , donde S(n,R) denotala subvariedad de matrices simetricas y sim(B) = BTB asocia a B lamatriz simetrica BTB .

En primer lugar veremos que la aplicacion diferenciable sim es unasubmersion en el neutro I (la matriz identidad). Denotemos por xrs

las componentes de la carta canonica de GL(n,R) , xrs((akl)) = ars,y por yij para i ≤ j las componentes de la carta canonica de S(n,R),yij(akl) = aij .

Notemos que

yijsim =∑h

xhixhj

Page 103: Apuntes Geom Diferencial

GRUPOS DE LIE 97

De donde se tiene que

TI(sim)

(∂

∂xrs

)I

=∑i≤j

(∑h

(δhr δisxhj + xhiδhr δ

js))(I)

(∂

∂yij

)I

=∑i≤j

(δisδjr + δirδ

js)

(∂

∂yij

)I

Ahora se prueba con falicildad que TI(sim) es suprayectiva, por lotanto sim es una submersion en I . Para ver que TA(sim) es suprayec-tiva en A ∈ O(n,R) , notemos que (AB)TAB = BTATAB = BTB ,ası que simLA = sim . De aquı se sigue que TI(sim) = TA(sim)TILA yteniendo en cuenta que TILA es un isomorfimo, se obtiene que TA(sim)es epimorfismo.

Aplicando la proposicion 3.2 del capıtulo 4 se sigue que sim−1(I) =

O(n,R) es una subvariedad regular de dimension n(n−1)2

.

Ejemplo 3.11. El grupo especial ortogonal SO(n,R) = A ∈O(n,R)|det(A) = 1 es un subgrupo abierto (y cerrado) de O(n,R) .Basta observar que precisamente es el nucleo de homomorfismo de gru-pos de Lie, det : O(n,R)→ −1, 1 .

Problemas

3.1. Probar que la aplicacion modulo, C∗ → R∗ , z → |z| es unhomomorfismo de grupos de Lie.

3.2. Probar que el determinante det : GL(n,R)→ R∗ es un homo-morfismo de grupos de Lie. Demostrar el resultado analogo para loscomplejos y cuaterniones.

3.3. Probar que todo grupo de Lie es Hausdorff.

3.4. Probar que el grupo ortogonal O(n,R) y el grupos GL(n,R)tienen dos componentes conexas.

3.5. Probar que el grupo ortogonal O(n,C) y el grupos GL(n,C)tienen una componente conexa.

3.6. Probar que el grupo ortogonal O(n,H) y el grupos GL(n,H)tienen una componente conexa.

3.7. Probar que el grupo ortogonal O(n,R) es compacto.

3.8. Probar que el grupo SL(n,R) no es compacto.

3.9. Probar que el grupo SO(2,R) es difeomorfo a S1 .

3.10. Probar que el grupo SO(3,R) es difeomorfo a P 3(R) .

Page 104: Apuntes Geom Diferencial

98 5. GRUPOS DE TRANSFORMACIONES

3.11. Sea O(2,R) el grupo ortogonal real.a) Probar que O(2,R) tiene una estructura de 1-variedad compacta

con dos componentes conexas.b) Si M(2,R) denota las matrices 2×2, probar que la inclusion de

i : O(2,R)→M(2,R) es una subvariedad regular.c) Si I denota la matrız identidad, probar que (TIi)TIO(2) se puede

identificar con el espacio de matrices antisimetricas 2×2.

4. Encajes de la botella de Klein y del plano proyectivo real

En los siguientes problemas se representan la botella de Klein y elplano proyectivo real como espacios de orbitas de grupos discontinuosde transformaciones de R2 . Ası expresados es facil encontrar encajesregulares de los mismos en R4 .

4.1. Sea la aplicacion ψ : 〈a, b|bab = a〉×(R×R)→ (R×R) definidapor

ψ(a, (r, s)) = (r + 2π, (−1)s) ,

ψ(b, (r, s)) = (r, s+ 2π) ,

a) Probar que ψ es una accion discontinua que tiene por dominiofundamental [−π, π]× [−π, π] . La correspondiente variedad de orbitasK = 〈a, b|bab = a〉\(R× R) la denominaremos como botella de Klein.Considerar la aplicacion F : R2 → R4 definida por

F (x, y) = ((cos y + 1) cosx, (cos y + 1) senx, sen y cosx

2, sen y sen

x

2)

b) Comprobar que F (ψ(a, (x, y))) = F (x, y) , F (ψ(b, (x, y))) =F (x, y) y que por lo tanto existe una aplicacion inducida F : K → R4 .Probar que F es diferenciable.

c)Probar que F es un encaje de la botella de Klein en R4 .

4.2. Sea P 2(R) es cociente obtenido al identificar puntos antıpodasde la esfera unidad.

a) Probar que existe un grupo de transformaciones finito y discon-tinuo de la 2-esfera unidad cuyo espacio de orbitas es P 2(R) .

Sea F : R3 → R4 dada por

F (x, y, z) = (x2 − y2, xy, xz, yz), x, y, z ∈ R .

Sea S2 ⊂ R3 la esfera unidad y considerar la restriccion φ = F |S2 : S2 →R3 donde S2 es la esfera unidad centrada en el origen.

b)Probar que φ es diferenciable y una inmersion. Comprobar queφ(p) = φ(−p) .

c) Sea φ : P 2(R) → R4 la inducida por la formula φ[p] = φ(p) .Probar que φ es un encaje del plano proyectivo real en R4 .

Page 105: Apuntes Geom Diferencial

CAPıTULO 6

CAMPOS Y FORMAS

La nocion de campo vectorial se utiliza para estudiar numerososfenomenos. Citemos los campos de fuerzas, campos electricos, magneti-cos, etc., que son herramientas basicas para una fısica basica. Similar-mente, las formas se utilizan para desarrollar teorıas de integracion,medir longitudes, angulos, etc. Cuando la situacion lo requiere y lamodelizacion no se puede abordar mediante abiertos euclidianos, esnecesario utilizar variedades diferenciales y para este contexto presen-tamos a continuacion las definiciones y propiedades elementales de loscampos y las formas.

1. Fibrado tangente y cotangente

Sea M una variedad diferenciable m-dimensional, consideremos elconjunto de todos los vectores tangentes TM =

⊔p∈M TpM . Denote-

mos por π : TM → M la proyeccion que aplica un vector tangente enel punto de tangencia, π(vp) = p . Para cada carta x de M, conside-ramos una nueva carta x : TM → R2m definida por xi(vp) = xi(p) ,xm+i(vp) = vp(xi) , 1 ≤ i ≤ m . Notemos que Dom x = π−1(Domx) yCodom x = Codomx × Rm . Para x, y cartas de M el cambio de lasnuevas cartas viene dado por

yx−1 (a, r) = y(∑

ri

(∂∂xi

)x−1a

)= (y1x

−1a, · · · , ymx−1a,∑ri

(∂y1∂xi

)x−1a

, · · · ,∑ri

(∂ym∂xi

)x−1a

)

= (yx−1a, rJTyx−1(a)),

donde JTyx−1 denota traspuesta de la matriz jacobiana del cambio de

cartas. En consecuencia, la familia de las cartas x|x carta de M dotaa TM de una estructura de variedad diferenciable 2m-dimensional.

Definicion 1.1. Llamaremos fibrado tangente a la proyeccion na-tural π : TM → M . Tambien denominaremos fibrado tangente a lavariedad TM .

Recordemos que C∞(M) denota el espacio de todas las funcionesdiferenciables de M en R . Si p ∈ Dom f , f ∈ C∞(M) podemos definirla aplicacion lineal dfp : TpM → R mediante la expresion dfp(v) = v(f)para cada v ∈ TpM .

99

Page 106: Apuntes Geom Diferencial

100 6. CAMPOS Y FORMAS

Definicion 1.2. Una aplicacion lineal de la forma TpM → R dire-mos que es un vector cotangente en p a M . El espacio vectorial de losvectores cotangentes en p a M se denotara por T ∗pM .

Para una carta x de M tenemos las funciones xi : M → R y paracada punto p podemos considerar (dxi)p para i ∈ 1, · · · ,m .

Lema 1.1. Sea p un punto del dominio de una carta x de unavariedad M , entonces (dx1)p, · · · , (dxm)p es una base del espacio T ∗pM .

Demostracion. Sabemos que(

∂∂x1

)p, · · · ,

(∂

∂xm

)p

es una base de

TpM . Los vectores cotangentes anteriores verifican que (dxi)p

(∂∂xj

)p

=(∂xi∂xj

)p

. Lo que prueba que precisamente se trata de la base dual.

Sea M una variedad diferenciable, consideremos el conjunto detodos los vectores cotangentes T ∗M =

⊔p∈M T ∗pM . Denotemos por

∗π : T ∗M →M la proyeccion ∗π(wp) = p .Para cada carta x de M, consideramos una nueva carta ∗x : T ∗M →

R2m definida por ∗xi(wp) = xi(p) , ∗xm+i(wp) = wp

(∂∂xi

)p

, 1 ≤ i ≤ m .

Notemos que Dom ∗x = ∗π−1(Domx) y Codom ∗x = Codomx × Rm .El cambio de las nuevas cartas viene dado por∗y∗x−1(a, r) = ∗y (

∑i ri(dxi)x−1a)

= (yx−1a,(∑

i ri

(∂xi∂y1

)x−1a

), · · · ,

(∑i ri

(∂xi∂ym

)x−1a

))

= (yx−1a, r((Jyx−1(a))−1)).

Notemos que es una funcion C∞ . De este modo vemos que la familiade cartas ∗x|x carta de M dota a T ∗M de una estructura de variedaddiferenciable 2m-dimensional. Es interesante observar que la matriz queaparece en el cambio de cartas del fibrado cotangente es precisamentela traspuesta de la inversa de la que obtenemos en el fibrado tangente.

Definicion 1.3. Se llamara fibrado cotangente a la proyeccion na-tural ∗π : T ∗M →M .Como antes, se utiliza tambien este mismo nom-bre para designar a la variedad T ∗M .

Problemas

1.1. Demostrar que al espacio tangente TpM en cualquier puntop de M puede dotarsele de la estructura de subvariedad regular deTM . Demostrar tambien que esta estructura coincide con su estructuraestandar como espacio vectorial real.

Solucion: En primer lugar veamos que la proyeccion π : TM → Mes una submersion. Para ello sea vp un vector tangente en el punto pde M . Tomemos una carta x de M en p y sea x la carta inducidaen TM . Entonces la representacion coordenada de π viene dada por

Page 107: Apuntes Geom Diferencial

2. DEFINICIoN Y PROPIEDADES DE CAMPOS Y FORMAS 101

xπx−1(r, s) = r . La matriz jacobiana de π respecto las cartas mencio-nadas es de la forma (id, 0) y su rango sera maximo. En consecuenciaπ es una submersion.

Recordemos ahora el teorema que aseguraba que las fibras de unasubmersion eran subvariedades regulares de dimension 2m −m = m .Es de destacar que las cartas x, x inducen una carta y en el punto

vp de TpM = π−1(p) . De modo que y(vp) = y

(∑m1 si

(∂∂xi

)p

)=

(s1, · · · , sm) que es presisamente la carta asociada a la base(

∂∂x1

)p,

· · · ,(

∂∂xm

)p

en la estructura estandar del espacio vectorial TpM .

1.2. Demostrar que si una variedad diferenciable M admite unabase contable para su topologıa (es segundo numerable) el fibrado tan-gente TM tambien admite una base de esas caracterısticas (es tambiensegundo numerable).

Solucion: Sabemos que si una variedad tiene un atlas contable siy solo si es segundo numerable. Entonces puesto que M es segundonumerable se tiene que M tiene un atlas contable A que induce el atlasA = x|x ∈ A de TM que al tener como conjunto de ındices unconjunto contable es tambien contable. Por lo tanto TM es segundocontable.

1.3. Si pr : M ×M ′ −→ M y pr′ : M ×M ′ −→ M ′ son las proyec-ciones canonicas, demostrar que

(pr?, pr′?) : T (M ×M ′) −→ TM × TM ′

es un difeomorfismo.

1.4. Probar que TRm es difeormorfo a Rm × Rm .

1.5. Probar que TS1 es difeormorfo a S1 × R1 .

2. Definicion y propiedades de campos y formas

Un operador lineal sobre un abierto U de M es una aplicacionξ : C∞(M) → C∞(M) tal que Dom ξf = U ∩ Dom f y que verificala propiedad de linealidad siguiente: ξ(αf + βg) = αξf + βξg . Siademas se tiene que ξ(f · g) = (ξf) · g + f · (ξg) , diremos que es unoperador derivacion con dominio U . A continuacion analizaremos larelacion entre operadores derivacion y campos.

Definicion 2.1. Un campo X de vectores tangentes a una variedadM es una seccion diferenciable del fibrado tangente π : TM → M ;es decir, πX = id |DomX . Una 1-forma w de vectores cotangentes auna variedad M es una seccion diferenciable del fibrado cotangente∗π : T ∗M →M ; es decir, πw = id |Domw .

Page 108: Apuntes Geom Diferencial

102 6. CAMPOS Y FORMAS

Lema 2.1. Sea M una variedad diferenciable y sea x una carta dela variedad.

(i) Si X : M → TM es una seccion del fibrado tangente yX|Domx =

∑mi=1Ai

∂∂xi

, entonces X|Domx es diferenciable siy solo si A1, · · · , Am son diferenciables.

(ii) Si w : M → T ∗M es una seccion del fibrado cotangente yw|Domx =

∑mi=1B

idxi , entonces w|Domx es diferenciable siy solo si B1, · · · , Bm son diferenciables,

(iii) Sean X, Y campos tangentes a M y f, g ∈ C∞(M) entoncesla seccion del fibrado tangente fX + gY definida por (fX +gY )p = f(p)Xp + g(p)Yp es tambien diferenciable, por lo quefX + gY es un campo tangente a M con dominio Dom f ∩DomX ∩Dom g ∩DomY .

(iv) Sean u, v 1-formas cotangentes a M y f, g ∈ C∞(M) entoncesla seccion del fibrado cotangente fu + gv definida por (fu +gv)p = f(p)up + g(p)vp es tambien diferenciable; por lo quefu+ gv es una 1-forma cotangente a M con dominio Dom f ∩Domu ∩Dom g ∩Dom v .

Demostracion. Notese que xXp = (x(p), A1(p), · · · , Am(p)) . En-tonces por el ejercicio 1.7 del capıtulo 2, se tiene que X|Domx es dife-renciable si y solo si A1, · · · , Am son diferenciables.

Analogamente para las 1-formas. La verificacion de (iii) y (iv) esuna comprobacion rutinaria.

Dado X un campo tangente a M , podemos definir el siguienteoperador inducido ξ que aplica una funcion diferenciable f de M en Ren la funcion ξf cuyo dominio es DomX∩Dom f y que esta definida porξf(p) = Xp(f) . Recıprocamente, dado un operador derivacion ξ sobreun abierto U , podemos asociarle la seccion X del fibrado tangentecuyo dominio es U y que esta definida por Xp(f) = ξf(p) .

Proposicion 2.1. Si X es un campo tangente a una variedad M ,entonces el operador inducido ξ es un operador derivacion. Recıproca-mente, si ξ es un operador derivacion, entonces la seccion asociada Xes un campo tangente.

Demostracion. Sea X un campo y supongamos que ξ es el co-rrespondiente operador. En primer lugar probaremos que si f es dife-renciable entonces ξf tambien lo es. Sea x una carta y supongamosque X|Domx =

∑mi=1 Ai

∂∂xi

. Entonces para p ∈ DomX ∩ Domx se

tiene que ξf(p) = Xp(f) =(∑m

i=1 Ai∂∂xi

)p

(f) =∑m

i=1 Ai(p)(∂f∂xi

)p

=(∑mi=1Ai

∂f∂xi

)(p) . Entonces ξf |Domx =

∑mi=1Ai

∂f∂xi

, para cada carta x .

Por lo tanto, si aplicamos el lema 2.1 se obtiene que ξf es diferenciable.Ahora veamos que es lineal, en efecto, (ξ(αf+βg))(p) = Xp(αf+βg) =αXp(f) + βXp(g) = αξf(p) + βξg(p) = (αξf + βξg)(p) para cada

Page 109: Apuntes Geom Diferencial

2. DEFINICIoN Y PROPIEDADES DE CAMPOS Y FORMAS 103

p ∈ DomX ∩ Dom f ∩ Dom g , escalares α, β ∈ R y f, g ∈ C∞(M) .Para ver que ξ es una derivacion, sean f, g ∈ C∞(M) , entoncesξ(f · g)(p) = Xp(f · g) = f(p)Xp(g) + Xp(f)g(p) = f(p)ξg(p) +ξf(p)g(p) = (f(ξg) + (ξf)g)(p) .

Recıprocamente, supongamos que ξ es un operador derivacion condominio el abierto U . Para cada p ∈ U , se tiene que Xp es lineal, ya queXp(αf+βg) = (ξ(αf+βg))(p) = αξf(p)+βξg(p) = αXp(f)+βXp(g) .Ademas cada Xp verifica la propiedad de Leibniz: Si f, g ∈ C∞(p) ,entonces Xp(f · g) = ξ(f · g)(p) = (f(ξg) + (ξf)g)(p) = f(p)Xp(g) +Xp(f)g(p) . Finalmente, queda por probar que X es diferenciable. Seap ∈ U = DomX y sea x una carta de M en p tal que Domx ⊂U . Entonces X =

∑m1 X(xi)

∂∂xi

=∑m

1 ξ(xi)∂∂xi

. Puesto que ξ(xi)es difierenciable para 1 ≤ i ≤ m , aplicando el lema 2.1 se deduceque X es diferenciable en el punto p . Por lo tanto X es una secciondiferenciable.

Observacion 2.1. (Notacion) Dada la correspondencia biunıvocaentre campos tangentes y operadores derivacion, utilizaremos la mismaletra X para denotar el campo y el operador derivacion inducido. Elconjunto de todos los campos tangentes a una variedad M se denota-ra por Ξ(M) y los campos tangentes con dominio un abierto U porΞU(M) . En particular ΞM(M) denota los campos globales tangentesa la variedad M .

Un espacio vectorial real V provisto de una aplicacion bilineal an-tisimetrica [−,−] : V × V → V que satisfaga la identidad de Jacobi

[[u, v], w] + [[v, w], u] + [[w, u], v] = 0

se denomina algebra de Lie .

Definicion 2.2. Dados dos campos tangentes X, Y a una variedadM se llama corchete de Lie al campo tangente definido como el opera-dor lineal que aplica la funcion f ∈ C∞(M) en la funcion [X, Y ]f =X(Y f)− Y (Xf) . Notemos que Dom[X, Y ] = DomX ∩DomY .

Proposicion 2.2. El corchete de Lie dota al espacio vectorialΞU(M) de los campos tangentes a una variedad M con dominio unabierto U de estructura de algebra de Lie. En particular el espacio vec-torial ΞM(M) de los campos globales tiene una estructura de algebrade Lie.

Demostracion. No es difıcil comprobar que efectivamente loscampos tangentes globales de una variedad tiene estructura de espaciovectorial real. La identidad de Jacobi se desprende de las siguientesigualdades

[[X, Y ], Z] + [[Y, Z], X] + [[Z,X], Y ] =

(XY − Y X)Z − Z(XY − Y X) + (Y Z − ZY )X −X(Y Z − ZY )

Page 110: Apuntes Geom Diferencial

104 6. CAMPOS Y FORMAS

+(ZX −XZ)Y − Y (ZX −XZ) = 0.

Dada w una 1-forma de M , podemos definir el siguiente operadorlineal inducido ω que aplica un campo X tangente a M en la funcionωX cuyo dominio es Domw ∩ DomX y esta definida por ωX(p) =wp(Xp) .

Una aplicacion ω : Ξ(M) → C∞(M) diremos que es un operadorlineal sobre un abierto U si satisface que Dom(ωX) = U ∩ DomX yω(fX + gY ) = fωX + gωY donde f, g ∈ C∞(M) y X, Y ∈ Ξ(M) . Demodo recıproco, dado un operador lineal ω : Ξ(M)→ C∞(M) sobre unabierto U , podemos asociarle la 1-forma w tal que si v ∈ TpM y Ves un campo tal que Vp = v , entonces wp(v) = ωV (p) . Notemos queel lema siguiente prueba la existencia del campo V y la independenciadel campo V elegido.

Lema 2.2. Sea M una variedad diferenciable.

(i) Si ω : Ξ(M) → C∞(M) es un operador lineal y X un campotangente a M y W un abierto de M , entonces

(ωX)|W = ω(X|W ) ,

(ii) Si v es un vector tangente en un punto p ∈M , entonces existeun campo tangente V definido en p tal que Vp = v ,

(iii) Sea ω : Ξ(M)→ C∞(M) es un operador lineal sobre un abiertoU y sean V, V ′ son dos campos tangentes definidos en un puntop ∈ U , si Vp = V ′p , entonces ωV (p) = ωV ′(p) .

Demostracion. Para ver (i), notemos que X − X|W = 0(X −X|W ) , entonces 0 = 0ω(X−X|W ) = ω(0(X−X|W )) = ω(X−X|W ) =ω(X)− ω(X|W )) . De aquı se obtiene que (ωX)|W = ω(X|W ) .

Para (ii), sea x una carta en el punto p, entonces v = λ1

(∂∂x1

)p

+

· · ·+λm(

∂∂xm

)p

. Entonces el campo V = λ1

(∂∂x1

)+· · ·+λm

(∂

∂xm

)ve-

rifica que Vp = v .(iii) Sea X un campo definido en p tal que Xp = 0 y sea x una carta

en p . Supongamos que X|DomX =∑

i fi

(∂∂xi

). Entonces aplicando (i)

se tiene que

(ωX)(p) = (ω(X|Domx))(p) = ω(∑

i fi

(∂∂xi

))(p) =

∑i fi(p)ω

(∂∂xi

)(p) .

De la condicion Xp = 0 se desprende que f1(p) = 0, · · · , fm(p) = 0,luego (ωX)(p) = 0 .

Tomemos ahora X = V − V ′ , notemos que Xp = 0 , entonces0 = (ω(V − V ′))(p) = (ωV )(p)− (ωV ′)(p) .

Proposicion 2.3. Existe una correspondencia biunıvoca entre ope-radores lineales respecto funciones de la forma ω : Ξ(M) → C∞(M) y1-formas w : M → T ∗M de una variedad M .

Page 111: Apuntes Geom Diferencial

2. DEFINICIoN Y PROPIEDADES DE CAMPOS Y FORMAS 105

Demostracion. En primer lugar, veamos que si w es una 1-forma,el correspondiente operador es ω y X un campo diferenciable, entoncesωX es diferenciable. En el dominio de una carta x se tiene que w|Domx =∑Aidxi y para el campo X|Domx =

∑Bj

∂∂xj

. Entonces (ωX)|Domx =∑AiBi que por el lema 2.1 es diferenciable. Puesto que esto sucede

para cada carta x se sigue que ωX es diferenciable. Veamos ahoraque ω es lineal, en efecto, (ω(fX + gY ))(p) = wp(f(p)Xp + g(p)Yp) =f(p)wp(Xp)+g(p)wp(Yp) = f(p)ωX(p)+g(p)ωY (p) = (fωX+gωY )(p)para cada p ∈ Domw ∩ DomX ∩ DomY , f, g ∈ C∞(M) y X, Y ∈Ξ(M) .

Recıprocamente, supongamos que ω es un operador lineal con domi-nio el abierto U . Para cada p ∈ U , se tiene que wp es lineal, para verlosupongamos que v, v ∈ TpM y que V, V son campos que extienden v yv , entonces wp(αv + βv) = (ω(αV + βV ))(p) = αωV (p) + βωV (p) =αwp(v)+βwp(v) . Finalmente queda probar que w es diferenciable. Seap ∈ U = Domw y sea x una carta de M en p tal que Domx ⊂ U .Entonces w =

∑m1 ω(∂∂xi

)dxi . Puesto que para i ∈ 1, · · · ,m se tiene

que ω(

∂∂xi

)es diferenciable, aplicando lema 2.1 se obtiene que w es

diferenciable en p para cada p ∈ Domw . Entonces w es una secciondiferenciable.

Como consecuencia del resultado anterior utilizaremos la mismaletra, digamos ω , para denotar el operador lineal y la seccion.

Definicion 2.3. Una r-forma ω con dominio un abierto Domω deuna variedad M es una aplicacion multilineal (respecto funciones) yalternada de la forma ω : Ξ(M)× . . .(r × Ξ(M)→ C∞(M) verificandoque Dom(ω(X1, · · · , Xr)) = Domω ∩DomX1 ∩ · · · ,∩DomXr . Se to-man como 0-formas la funciones diferenciables f : M → R . Denotemospor Ωr(M) el espacio de las r-formas de M y por Ωr

U(M) el espaciode las r-formas con dominio un abierto U . En particular Ωr

M(M) es elespacio de las r-formas globales de M .

Dada una r-forma ω de una variedad M , su diferencial como la(r + 1)-forma dω definida por

dω(X0, · · · , Xr) =1

r + 1

r∑0

(−1)iXiω(X0, · · · , Xi, · · · , Xr)

+1

r + 1

∑0≤i<j≤r

(−1)i+jω( [Xi, Xj], · · · , Xi, · · · , Xj, · · · , Xr)

Dada una r-forma θ con dominio Dom θ y un abierto U de la va-riedad se tiene que la restriccion θ|U se define a traves de la siguiente

Page 112: Apuntes Geom Diferencial

106 6. CAMPOS Y FORMAS

formulaθ|U(X1, · · · , Xr) = θ(X1, · · · , Xr)|U

Es facil ver que la restriccion de r-formas verifica las siguientespropiedades:

Proposicion 2.4. Sea M una variedad y ω una r-forma,

(i) Si X1, · · · , Xr son campos tangentes a M , entoncesθ|U(X1, · · · , Xr) = θ(X1|U , · · · , Xr|U) .

(ii) Si ω|U = 0 para cada abierto U de un cubrimiento de la varie-dad, entonces ω = 0 ,

(iii) Si U es un abierto, entonces (dω)|U = d(ω|U) .

Proposicion 2.5. El operador d verifica para cualquier r-forma laecuacion ddω = 0 .

Demostracion. Veamos que ddω|Domx = 0 para cada dominio de

carta. Esto implica que ddω = 0 . Notese que si X0 =(

∂∂xi0

), . . . ,

Xr+1 =(

∂∂xir+1

), son los campos coordenados se tiene que el corchete

de Lie de dos de ellos es nulo, entonces

ddω(X0, · · · , Xr+1) = 1r+2

∑r+10 (−1)iXidω(X0, · · · , Xi, · · · , Xr+1) =

1r+1

1r+2

∑r+1i<j (−1)i(−1)j(XjXi−XiXj)ω(X0, · · · , Xi, · · · , Xj, · · ·Xr+1)

= 0

Definicion 2.4. Una r-forma ω se dice que es cerrada si dω = 0 .Una r-forma ε se dice que es exacta si existe una (r − 1)-forma θ talque dθ = ε . Sea Zr

U(M) = ω|ω r-forma cerrada con dominio U yBrU(M) = ε|ε r-forma exacta con dominio U . El r-esimo grupo de

cohomologıa de De Rham de un abierto U de la variedad M se definecomo:

HrDR(U) = Zr

U(M)/BrU(M) .

En particular se obtienen los grupos de cohomologıa de De Rham dela viariedad M .

Para cualquier r-forma ω con dominio U se tiene que ddω = 0 porlo que se obtiene el complejo de cocadenas siguiente:

0→ Ω0U(M)→ Ω1

U(M)→ · · ·ΩrU(M)→ · · ·

donde ΩrU(M) denota las r-formas de M con dominio U , el r-esimo

grupo de cohomologıa de De Rham de un abierto U de la variedad Mes el r-esimo grupo de cohomologıa del complejo de cocadenas anterior.

Observacion 2.2. Si M es una variedad paracompacta Hausdorff,se tiene que los grupos de cohomologıa de De Rham son isomorfos alos grupos de cohomologıa singular con coeficientes enteros

HrDR(U) ∼= Hsing(M,Z)

Page 113: Apuntes Geom Diferencial

DEFINICION Y PROPIEDADES DE CAMPOS Y FORMAS 107

Una demostracion de este resultado puede verse en [39] .

Problemas

2.1. Demostrar que la funcion que aplica cada punto de una va-riedad diferenciable M en el vector tangente nulo en ese punto es uncampo vectorial en M .

2.2. Sean x, y las cartas del atlas estereografico de S2 ver el ejemplo3.4 del capıtulo 2 . Si a, b ∈ R, demostrar que los campos vectoriales:

X1 = (ax1 − bx2) ∂∂x1

+ (bx1 + ax2) ∂∂x2

X2 = X2 = (−ay1 − by2) ∂∂y1

+ (by1 − ay2) ∂∂y2

definen un campo vectorial sobre S2.

Solucion: Recordemos que el cambio de cartas viene dado por

y1 = x1

(x1)2+(x2)2y2 = x2

(x1)2+(x2)2

x1 = y1(y1)2+(y2)2

x2 = y2(y1)2+(y2)2

Entonces la matriz del cambio viene dada por los siguientes coeficientes

∂y1∂x1

= −(x1)2+(x2)2

((x1)2+(x2)2)2∂y1∂x2

= −2x1x2

((x1)2+(x2)2)2

∂y2∂x1

= −2x1x2

((x1)2+(x2)2)2∂y2∂x2

= (x1)2−(x2)2

((x1)2+(x2)2)2

X1|Dom y = (ax1 − bx2) ∂∂x1

+ (bx1 + ax2) ∂∂x2

= (ax1 − bx2)((

−(x1)2+(x2)2

((x1)2+(x2)2)2

)(∂∂y1

)+(

−2x1x2

((x1)2+(x2)2)2

)(∂∂y2

))+ (bx1 + ax2)

((−2x1x2

((x1)2+(x2)2)2

)(∂∂y1

)+(

(x1)2−(x2)2

((x1)2+(x2)2)2

)(∂∂y2

))=

((ax1 − bx2)

(−(x1)2+(x2)2

((x1)2+(x2)2)2

)+ (bx1 + ax2)

(−2x1x2

((x1)2+(x2)2)2

))(∂∂y1

)+

((ax1 − bx2)

(−2x1x2

((x1)2+(x2)2)2

)+ (bx1 + ax2)

((x1)2−(x2)2

((x1)2+(x2)2)2

))(∂∂y2

)=

(−ax1(x1)2+ax1(x2)2+bx2(x1)2−bx2(x2)2−2bx2(x1)2−2ax1(x2)2

((x1)2+(x2)2)2

)(∂∂y1

)+

(bx1(x1)2−bx1(x2)2+ax2(x1)2−ax2(x2)2−2ax2(x1)2+2bx1(x2)2

((x1)2+(x2)2)2

)(∂∂y2

)=

((−ax1−bx2)(x1)2+(−ax1−bx2)(x2)2

((x1)2+(x2)2)2

)(∂∂y1

)+

((bx1−ax2)(x1)2+(bx1−ax2)(x2)2

((x1)2+(x2)2)2

)(∂∂y2

)=

((−ax1−bx2)

((x1)2+(x2)2)

)(∂∂y1

)+(

(bx1−ax2)((x1)2+(x2)2)

)(∂∂y2

)= (−ay1 − by2)

(∂∂y1

)+ (by1 − ay2)

(∂∂y2

)= X2|Domx

Puesto que ambos coinciden en la interseccion de los dominios setiene que efectivamente definen un campo en la esfera.

Page 114: Apuntes Geom Diferencial

108 6. CAMPOS Y FORMAS

2.3. Sean w = x0, x = x1, y = x2 las cartas del atlas de P 2(R) dadasen el ejemplo 3.5 del capıtulo 2 . Demostrar que los campos vectoriales:

X1 = (w1) ∂∂w1− (w2) ∂

∂w2

X2 = (−x1) ∂∂x1− (2x2) ∂

∂x2

X3 = (y1) ∂∂y1

+ (2y2) ∂∂y2

definen un campo vectorial sobre P 2(R) .

2.4. Si X, Y son campos vectoriales y f, g funciones diferenciablesdefinidas en una variedad diferenciable M con valores en R, demostrarque:

[fX, gY ] = fg[X, Y ] + f(Xg)Y − g(Y f)X

2.5. Si x es una carta de una variedad diferenciable M con dominioU y X, Y son campos vectoriales en M , demostrar que:

[X, Y ]|U =∑

(X(Y xj)− Y (Xxj))∂

∂xj

2.6. Demostrar que el espacio vectorial de las matrices reales n×ntiene estructura de algebra de Lie cuando se define la siguiente opera-cion:

[A,B] = AB −BA2.7. Demostrar que si φ : M → N es un difeomorfismo local e Y es

un campo tangente a N , entonces existe un unico campo X tangente aM tal que para cada p ∈ Dom f∩f−1 DomY se tiene que φ∗pXp = Yφp .

2.8. Demostrar que si φ : M → N es diferenciable y ω es una 1-forma de N , entonces existe una unica 1-forma φ∗w cotangente a M talque para cada p ∈ Dom f ∩ f−1 Domw se tiene que φ∗p(ωφp) = (φ∗ω)p .

2.9. En la 1-esfera S1 consideramos la aplicacion recubridoraf(t) = (cos 2πt, sen 2πt) que induce el campo Xp = f∗p

ddt s

si p =(cos 2πs, sen 2πs) y la 1-forma w tal que w(X) = 1. Probar que pa-ra cualquier funcion global h de S1 en R se tiene que dh 6= w y deducirque el primer grupo de cohomologıa de De Rham de S1 es no nulo.

Solucion: Supongamos que w = dh, entonces f ∗w = f ∗(dh) =d(hf) . Puesto que hf es una funcion acotada en algun punto s tieneun maximo en el que se anulara la primera derivada. Entonces

d(hf)s

(d

dt

)s

=

(d(hf)

dt

)s

= 0

Sin embargo en el mismo punto se verifica que

(f ∗w)s

(d

dt

)s

= wf(s)f∗s

(d

dt

)s

= wf(s)Xf(s) = 1

Lo que lleva a una contradiccion que viene de suponer que existe htal que dh = w . Por lo tanto el primer grupo de cohomologıa de DeRham de la S1 es no trivial.

Page 115: Apuntes Geom Diferencial

VARIEDADES PARALELIZABLES 109

3. Variedades paralelizables

Recordemos que asociado a una variedad M podemos considerar elespacio ΞM(M) de todos los campos globales tangentes a M que tieneuna estructura natural de C∞M (M)-modulo.

Definicion 3.1. Dados X1, · · · , Xr campos globales tangentes aM , diremos que son linealmente independientes si X1

p , · · · , Xrp son in-

dependientes en TpM para cada p ∈M . Una variedad M de dimensionm se dice paralelizable si existen X1, · · · , Xm campos globales e inde-pendientes.

Proposicion 3.1. Sea M una variedad, entonces son equivalentes:(i) M es paralelizable,(ii) Existe un difeomorfismo de TM con M ×Rm que conmuta con

las proyecciones y es lineal en las fibras.

Demostracion. (i) implica (ii): Supongamos que X1, · · · , Xm esuna paralelizacion de M . Entonces podemos considerar lar biyeccionΘ: M × Rm → TM definida por Θ(p, (λ1, · · · , λm)) = λ1X

1p + · · · +

λmXmp , que ademas conmuta con las proyecciones y es lineal en las

fibras. Si x es una carta de M y Xj =∑

(Xjxi)∂∂xi

, la representacioncoordenada de Θ es la siguiente:

xΘ(x−1×idRm)(r, λ) = (r, (∑

λj(Xjx1)x−1(r), · · · ,

∑λj(X

jxm)x−1(r)))

De aquı se deduce que detJx,(x×id)Θ (p, λ) 6= 0 . Por lo tanto la biyeccion

Θ es un difeomorfismo local, ello implica que Θ es un difeomorfismoglobal de M × Rm en TM .

(ii) implica (i): Es facil ver que el fibrado M×Rm tiene m seccionesdiferenciables independientes, basta considerar Si(p) = (p, (0, · · · , 1(i,· · · , 0)) . Entonces si Θ es el difeomorfismo dado, podemos tomar comoparalelizacion ΘS1, · · · ,ΘSm .

Notese que si los fibrados TM y M × Rm son difeomorfos (comofibrados vectoriales), entonces ΞM(M) es isomorfo al modulo de las sec-ciones diferenciables de M ×Rm . Una seccion M ×Rm es de la formaS(p) = (p, (f1(p), · · · , fm(p))) con f1, · · · , fm : M → R diferenciables.Esto implica que la correspondencia S → (f1, · · · , fm) es un isomorfis-mo de ΞM(M) en el C∞M (M)-modulo libre de rango m , C∞M (M)m .

Problemas

3.1. Probar que los abiertos de Rm y la 1-esfera son variedadesparalelizables.

Solucion: Si U es un abierto en Rm entonces si x es la carta inclusionse tiene que

(∂∂x1, . . . , ∂

∂xm

)es una paralelizacion de U .

Page 116: Apuntes Geom Diferencial

110 6. CAMPOS Y FORMAS

En el caso de la 1-esfera S1 podemos considerar sobre R el campoddt

. Este campo es invariante por traslaciones. Supongamos que tene-

mos una traslacion f(t) = t + z . Entonces f∗s(ddt

)s

=(dfdt

)s

(ddt

)s+z

=(d(t+z)dt

)s

(ddt

)s+z

=(ddt

)s+z

Ası que el grupo discontinuo de transforma-

ciones generado por la traslacion unidad preserva el campo ddt

. Enton-ces queda inducido de modo natural un campo en la variedad obtenidaal dividir por el grupo de traslaciones enteras. Ademas el inducido esno nulo si tenemos en cuenta que la proyeccion es un difeomorfismolocal.

3.2. Demostrar que el producto de variedades paralelizables es pa-ralelizable. Probar que los toros y cilindros de la forma S1 × · · · × S1 ,S1 × · · · × S1 × R · · · × R son paralelizables.

3.3. Probar que un grupo de Lie es paralelizable.

4. Variedades orientables

Dadas dos bases de un espacio vectorial de dimension mayor que unoe = (e1, . . . , em) , e′ = (e′1, . . . , e

′m) tal que el cambio de bases viene dado

por la matriz e′j =∑aijei . Diremos que e′ ∼ e si det(aij) > 0 . Se llama

orientacion del espacio vectorial real a cada una de las dos clases deequivalencia de la relacion anterior θ(e1, e2, . . . , em) , θ(−e1, e2 . . . , em) .

Definicion 4.1. Una orientacion de una variedadM es una seccionque asigna a cada p ∈M una orientacion θp del espacio vectorial TpMde tal modo que para cada p ∈ M existen m campos independientesX1, · · · , Xm definidos en un entorno abierto U de p de modo que θq =θ(X1

q , · · · , Xmq ) para cada q ∈ U . Una variedad se dice orientable si

admite una orientacion. Una variedad provista de una orientacion sedice que es una variedad orientada.

Proposicion 4.1. Sea θ una orientacion en una variedad M y θ′

una orientacion en un abierto conexo U . Entonces θ′ = θ|U o θ′ =−θ|U .

Demostracion. Sean los subconjuntos U+ = p ∈ U |θp = θ′p yU− = p ∈ U |θp = −θ′p . Si p ∈ U+, entonces existen paralelizaciones

X1, · · · , Xm y X ′1, · · · , X ′m que determinan las orientaciones θ y θ′ enun entorno abierto V del punto p contenido en U . Para cada q ∈ V setiene que X i

q =∑aji (q)X

′jq de modo que cada aji es un funcion diferen-

ciable con dominio V . Notese que det(aji (p)) > 0 , y por ser det(aji )una funcion continua, existe un entorno abierto W de p contenido enV tal que det(aji (q)) > 0 para cada q ∈ W . Entonces p ∈ W ⊂ U+ ,esto sucede para cada punto de U+ . Por lo que U+ es un abierto deM . Similarmente se prueba que U− es abierto. Aplicado que U es co-nexo se tiene que U = U+ o U = U− . Consecuentemente θ′ = θ|U oθ′ = −θ|U .

Page 117: Apuntes Geom Diferencial

4. VARIEDADES ORIENTABLES 111

Corolario 4.1. Una variedad orientable y conexa admite dosorientaciones.

Proposicion 4.2. Sean x, y un atlas de una variedad orientable

M con dominios conexos, entonces det(∂xi∂yj

)tiene signo constante en

Domx ∩Dom y .

Demostracion. Teniendo en cuenta que Domx es conexo y queM es orientable se puede aplicar la proposicion 4.1 para afirmar queexiste una orientacion θ en M compatible con la parametrizacion ∂

∂x1,

· · · , ∂∂xm

. Por otro lado la parametrizacion ∂∂y1, · · · , ∂

∂yminduce una

orientacion θ′ en el dominio de la carta y . Al ser este conexo, dela proposicion 4.1 se infiere que θ′ = θ|Dom y o θ′ = −θ|Dom y , en el

primer caso det(∂xi∂yj

)es positivo en Domx ∩ Dom y y en el otro caso

negativo.

Corolario 4.2. La banda de Moebius no es orientable.

Demostracion. Consideremos la banda de Moebius como el es-pacio cociente M = R2/(r, s) ∼ (r + z, (−1)zs) con z entero. Seap : R2 → M la proyeccion canonica. Tomemos el atlas de dos cartasx = (p|(0,1)×R)−1 , y = (p|(1/2,3/2)×R)−1 que tienen dominio conexo. Elcambio de cartas viene dado por

yx−1(r, s) = (r + 1,−s) si 0 < r < 1/2 ,

yx−1(r, s) = (r, s) si 1/2 < r < 1 .

Entonces det(∂yi∂xj

)(r,s)

vale 1 para 0 < r < 1/2 y vale −1 para 1/2 <

r < 1 . Si suponemos que M es orientable, entonces la proposicion

anterior implica que el signo de det(∂yi∂xj

)(r,s)

debe ser constante. En

consecuencia M no es orientable.

Proposicion 4.3. Sea φ : M → N un difeomorfismo local condominio M . Si θ es una orientacion en N , entonces φ induce de modonatural una orientacion φ∗θ en M .

Demostracion. Sea l : V → W un isomorfismo de espacios vec-toriales. Si una orientacion θ de W esta determinada por una base(b1, · · · , bm) , entonces (l−1b1, · · · , l−1bm) es una base de V que deter-mina una orientacion l∗θ en V . Ahora puesto que φ es un difeomorfis-mo local se tiene que para p ∈M , Tpφ determina una una orientacion(Tpφ)∗θφp en TpM . Sea U un entorno abierto de p tal que φ|U es undifeomorfismo. Si Y 1, · · · , Y m es una paralelizacion de θ en φU , en-tonces (Tφ|U)−1Y 1φ|U , · · · , (Tφ|U)−1Y mφ|U es una paralelizacion para(Tpφ)∗θφp con p ∈ U . Luego (φ∗θ)p = (Tpφ)∗θφp , p ∈ M define unaorientacion en M .

Page 118: Apuntes Geom Diferencial

112 6. CAMPOS Y FORMAS

Proposicion 4.4. Sean φ : M → N ψ : N → P difeomorfismolocal con dominios M y N , respectivamente. Si θ es una orientacion enP , entonces φ∗ψ∗θ = (ψφ)∗θ .

Demostracion. Para cada punto a ∈ M se tiene que Ta(ψφ) =(Tφ(a)ψ)(Taφ) . Ademas por ser difeomorfismo locales, si w ∈ Tψφ(a)Pentonces obtenemos que (Taφ)−1(Tφ(a)ψ)−1(w) = (Ta(ψφ))−1(w) . Dela definicion de orientacion inducida por un difeomorfismo local inme-diatamente se sigue que φ∗ψ∗θ = (ψφ)∗θ .

Proposicion 4.5. Sea G un grupo discontinuo de transformacionesde M . Entonces G\M es orientable si y solo si existe una orientacionθ en M que es preservada por cada transformacion del grupo.

Demostracion. Sea η : M → G\M la proyeccion canonica que esun difeomorfismo local. Denotaremos por φg la transformacion aso-ciada al elemento g ∈ G . Supongamos que G\M es orientable ysea θ una orientacion. Tomemos en M la orientacion η∗θ . Para ca-da g ∈ G la transformacion φg verifica que ηφg = η . Aplicando laproposicion anterior se tiene que φ∗gη

∗θ = η∗θ . Luego la orientacioninducida es preservada por las transformaciones del grupo. Recıproca-mente, supongamos que en M disponemos de una orientacion ε . Seab ∈ G\M , tomemos a ∈ M tal que η(a) = b . Si la orientacion εaesta determinada por la base (e1, · · · , em) y Taη es la aplicacion tan-gente, que es un isomorfismo, entonces (Taη(e1), · · · , Taη(em)) es unabase que determina una orientacion en Tb(G\M) . Si hubieramos to-mado otro a′ ∈ M verificando η(a′) = b , entonces existirıa un g ∈ Gtal que φg(a) = a′ . Supongamos que εa′ esta determinada por la ba-se (e′1, · · · , e′m) , puesto que φ∗gε = ε , si (Taφg)

−1(e′j) =∑αijei con

det(αij) > 0 . Entonces se tiene que (Ta′η(e′j) =∑αijTaη(ei) . En con-

secuencia (Taη(e1), · · · , Taη(em)) y (Ta′η(e′1), · · · , Taη(e′m)) determinanla misma orientacion en b . Finalmente observemos que existe un en-torno abierto U de a y una paralelizacion X1, · · · , Xm definida en U talque ε es compatible con X1, · · · , Xm , η|U es un difeomorfismo. Enton-ces (η|U)−1X1(T (η|U)), · · · , (η|U)−1Xm(T (η|U)) es un paralelizacion deθ en ηU entorno abierto de b .

Ejemplo 4.1. La 1-esfera S1 se puede obtener como la variedadde las orbitas del grupo de traslaciones enteras φ : Z×R→ R definidopor la accion φ(z, r) = z + r . Notese que la orientacion inducida porla paralelizacion canonica d

dtes invariante por traslaciones. Entonces,

la proposicion anterior implica que S1 es orientable.

Para ver que las demas esferas Sn , n ≥ 2 son orientables, es sufi-ciente aplicar el siguiente resultado al atlas estereografico.

Proposicion 4.6. Si una variedad M admite un atlas de dos cartasx, y de modo que Domx∩Dom y es conexo, entonces M es orientable.

Page 119: Apuntes Geom Diferencial

4. VARIEDADES ORIENTABLES 113

Demostracion. La carta x induce una orientacion θ en su do-minio, y respectivamente una orientacion τ es inducida por y . En lainterseccion conexa Dom x ∩ DomY se tiene que θ|Dom y = τ |Domx oθ|Domx = −τ |Dom y . En el primer caso, θ ∪ τ define una orientacion enM y en el segundo θ ∪ −τ .

Corolario 4.3. La espacio proyectivo P n(R) es orientable si ysolo si n es impar.

Demostracion. El espacio proyectivo P n(R) es el espacio de orbi-tas de la accion del cıclico de orden dos generado por la aplicacionantıpoda a de Sn+1 . Si θ es una de las dos orientaciones de Sn+1 ,entonces por la proposicion 4.1 , se tiene que a∗θ = θ o a∗θ = −θ .Para determinar se se verifica una u otra bastara ver que ocurre enun punto. Si tomamos la carta x de la proyeccion estereografica sucedeque el punto E = (1, 0, · · · , 0) y −E estan en Domx , entonces

x1 =s1

1− sn+1

, · · · , xn =sn

1− sn+1

xa(s1, · · · , sn+1) = x(−s1, · · · ,−sn+1) = (−s1

1 + sn+1

, · · · , −sn1 + sn+1

) .

Notemos que

(x1)2 + · · ·+ (xn)2 =1− s2

n+1

(1− sn+1)2=

1 + sn+1

(1− sn+1)=−xiaxi

.

De donde se tiene que

xia =−xi

(x1)2 + · · ·+ (xn)2 .

Calculando las derivadas parciales se obtiene:

∂(xia)

∂xj=−δij((x1)2 + · · ·+ (xn)2) + 2xixj

((x1)2 + · · ·+ (xn)2)2

De donde (∂(xia)

∂xj

)E

=

1 if i = j = 1,

−1 if i = j > 1,

0 en otro caso

En consecuencia se tiene que a∗θ = (−1)n−1θ . Luego a preserva laorientacion si y solo si n es impar. Aplicando la proposicion anterior setiene que RP n es orientable si y solo si n es impar.

Proposicion 4.7. Si θ es una orientacion en M , entonces existeun atlas compatible con θ .

Demostracion. Tomemos un atlas con dominios conexos y encaso que una carta no sea compatible se le cambia una coordenada designo.

Page 120: Apuntes Geom Diferencial

114 6. CAMPOS Y FORMAS

Problemas

4.1. Probar que una variedad paralelizable es orientable.

4.2. Demostrar que la botella de Klein es no orientable.

4.3. Considerar el toro como el espacio de orbitas de R2 bajo laaccion del grupo discontinuo de traslaciones enteras. Probar que existeuna orientacion en R2 que es preservada por estas traslaciones y deducirque el toro es orientable.

4.4. Demostrar que la variedad producto M ×M ′ es orientable si ysolo si las variedades M y M ′ lo son.

4.5. Demostrar que el fibrado tangente TM de cualquier variedadM es una variedad orientable.

5. Curvas integrales

Una curva es una funcion diferenciable σ : R → M . Sea X uncampo de vectores tangentes. Se dice que σ es una curva integral de Xsi para cada s ∈ σ−1 DomX se tiene que Tsσ

(ddt

)s

= Xσ(s) .Sea x : M → Rm una carta y sea c = xσ . Si σ es una curva integral

de X en el dominio de la carta, entonces para cada s ∈ σ−1 DomX ∩Domx se tiene que por un lado

σ∗(ddt

)=(σ∗

ddt

)x1

∂∂x1

+ · · ·+(σ∗

ddt

)xm

∂∂xm

=(d(x1σ)dt

)∂∂x1

+ · · ·+(d(xmσ)dt

)∂

∂xm

=(dc1dt

)∂∂x1

+ · · ·+(dcmdt

)∂

∂xm

y, por otro, si suponemos que X =∑m

i fi∂∂xi

. Para cada punto de lacurva se tiene que

Xσ = f1σ∂∂x1

+ · · ·+ fmσ∂

∂xm= f1x

−1xσ ∂∂x1

+ · · ·+ fmx−1xσ ∂

∂xm

Entonces σ es curva integral en el dominio de la carta si y solo sic = xσ y fi = fix

−1 satisfacen las ecuaciones diferenciales:dc1dt

= f1(c1, . . . , cm)...

dcmdt

= fm(c1, . . . , cm)

Es decir si y solo si c : R → Rm es una solucion de la del sistema deecuaciones diferenciales de primer grado anterior.

Recordemos que

Proposicion 5.1. Sea f : Rm → Rm una funcion C∞ , a ∈ Dom f .Entonces existe un entorno abierto V de a en Dom f y un ε > 0 talesque para cada t0 en R y cada r ∈ V existe una unica curva cr tal queDom cr = (t0 − ε, t0 + ε) , cr(t0) = r y

Page 121: Apuntes Geom Diferencial

5. CURVAS INTEGRALES 115

dcr1dt

= f1(cr1, . . . , crm)

...dcrmdt

= fm(cr1, . . . , crm)

Ademas la funcion (t0 − ε, t0 + ε)× V → Rm , (s, r)→ cr(s) es C∞ .

Como consecuencia del teorema de existencia y unicidad anteriorse obtiene el siguiente resultado:

Proposicion 5.2. Sea X un campo de vectores tangentes a unavariedad M , p ∈ DomX . Entonces existe un entorno abierto U de pen DomX y un ε > 0 tales que para cada t0 en R y cada q ∈ U existeuna unica curva σq tal que Dom σq = (t0 − ε, t0 + ε) , σq(t0) = q yσ∗s(ddt

)s

= Xσ(s) . Es decir que σq es una curva integral de X . Ademasla funcion (t0 − ε, t0 + ε)× U →M , (s, q)→ σq(s) es diferenciable.

Definicion 5.1. Una curva integral σ se dice completa si Domσ =R . Un campo se dice completo si todas sus curvas integrales son com-pletas.

Proposicion 5.3. Los campos de una variedad compacta son com-pletos.

Demostracion. Sea X un campo en una variedad compacta Mpara cada p ∈ M existe un entorno abierto U y un ε > 0 satisfa-ciendo las propiedades de la proposicion anterior. Estos abiertos for-man un cubrimiento abierto de M y por ser M compacta se puedeextraer un subcubrimiento finito U1, · · · , Un que tiene asociados losreales ε1, · · · , εn . Tomemos δ = mınε1, · · · , εn . Veamos que X escompleto, sea σ : (a, b) → M una curva integral con dominio conexo(a, b) veamos que siempre se puede extender el dominio a un nuevointervalo (a − δ

2, b + δ

2) . Si para el punto σ(b − δ

2) aplicamos la pro-

posicion anterior tomando t0 = b − δ2, encontramos una nueva curva

integral σσ(b− δ2

) : (b − 3δ2, b + δ

2) → M . Por la unicidad probada en

la proposciocion anterior tenemos que σ|(b− 3δ2,b) = σσ(b− δ

2)|(b− 3δ

2,b) . De

aquı se sigue que σ admite una extension diferenciable a (a, b+ δ2) . De

modo analogo se puede extender a (a− δ2, b+ δ

2) . Esto implica que la

curva integral σ se puede extender a (−∞,+∞) ya que en caso con-trario, el dominio abierto conexo de la extension maximal serıa de laforma (a′.b′), y podrıamos aplicar de nuevo el argumento anterior parallegar a ver que la extension considerada no era la maximal.

El siguiente resultado relaciona las curvas integrales de campos conlos sistemas dinamicos diferenciables que hemos visto en la seccion 2del capıtulo anterior:

Proposicion 5.4. Sea X un campo completo en una variedad di-ferenciable M y denotemos por σp : R → M a la unica curva integral

Page 122: Apuntes Geom Diferencial

116 6. CAMPOS Y FORMAS

de campo tal que σp(0) = p y(dσp

dt

)0

= Xp . Entonces la aplicacionφ : R ×M → M definida por φ(t, p) = σp(t) es un sistema dinamicodiferenciable.

Demostracion. Veamos las propiedades que verifica φ : R×M →M . Para cada p ∈M se tiene que φ(0, p) = σp(0) = p . Sea la traslacionλs : R→ R dada por λs(t) = t+ s . Notemos que

φ(t+ s, p) = σp(t+ s) = σpλs(t)

φ(t, φ(s, p)) = σφ(s,p(t) = σσp(s)(t) .

Entonces tenemos

d(σpλs)

dt=

(dσp

dt

)λs

= Xσpλs

por lo que σpλs es curva integral del campo. En cuanto a las condicionesiniciales se verifica

σpλs(0) = σp(s), σσp(s)(0) = σp(s)

y ademas (d(σpλs)

dt

)0

= Xσp(s),(dσσ

p(s)

dt

)0

= Xσp(s)

Aplicando las propiedad de unicidad se obtiene que σpλs = σσp(s) .

Luego σpλs(t) = σσp(s)(t) . Por lo tanto φ(t+ s, p) = φ(t, φ(s, p)) . Por

otro lado, por las propiedades que se desprenden del teorema de exis-tencia y unicidad de soluciones de sistemas de ecuaciones diferencialesse obtiene tambien que φ es diferenciable.

Problemas

5.1. Encontrar las curvas integrales del siguiente campo definido enR2 :

X = 2x( ∂∂x

) + 2y( ∂∂y

)

Solucion: El sistema de ecuaciones diferenciales asociado a este cam-po es el siguiente:

dxdt

= 2x dydt

= 2y

equivalentemente:dxx

= 2dt dyy

= 2dt

que tiene como solucion:

x = ae2t y = be2t

Page 123: Apuntes Geom Diferencial

CURVAS INTEGRALES 117

5.2. Encontrar las curvas integrales del siguiente campo definido enR2 :

X = −2x( ∂∂x

)− 2y( ∂∂y

)

Solucion: El sistema de ecuaciones diferenciales asociado a este cam-po es el siguiente:

dxdt

= −2x dydt

= −2y

equivalentemente:dxx

= −2dt dyy

= −2dt

que tiene como solucion:

x = ae−2t y = be−2t

5.3. Si x, y son las cartas del atlas estereografico de S2 . Probar quelos campos

X = 2x1( ∂∂x1

) + 2x2( ∂∂x2

)

Y = −2y1( ∂∂y1

)− 2y2( ∂∂y

)

determinan un campo sobre la 2-esfera. Representar graficamente lascurvas integrales de dicho campo.

5.4. Encontrar las curvas integrales de los siguiente campos defini-dos en R2 :

x( ∂∂x

)

y( ∂∂x

)− x( ∂∂y

)

y( ∂∂x

)− y3( ∂∂y2

)

5.5. Considerar los siguientes campos vectoriales de R2

X = −y ∂∂x

+ y ∂∂y

Y = x ∂∂x

+ y ∂∂y

a) Calcular el corchete de Lie [X, Y ] en la base ∂∂x

, ∂∂y

.

b) Encontrar las curvas integrales de X y de Y .

Solucion: a) Calculemos [X, Y ]x y [X, Y ]y

XY x− Y Xx = Xx− Y (−y) = −y − (−y) = 0

XY y − Y Xy = Xy − Y y = −y − y = 0

Si tenemos en cuenta que [X, Y ] = [X, Y ]x ∂∂x

+ [X, Y ]y ∂∂y

se tiene que

[X, Y ] = 0 .b) El sistema de ecuaciones diferenciales asociado al campo X es el

siguiente:

dxdt

= −y dydt

= y

Page 124: Apuntes Geom Diferencial

118 6. CAMPOS Y FORMAS

equivalentemente:dxdt

= −y dyy

= dt

que tiene como solucion:

x = −bet + b+ a y = bet

Una representacion de la solucion anterior puede verse en la siguien-te figura:

El sistema de ecuaciones diferenciales asociado al campo Y es el si-guiente:

dxdt

= x dydt

= y

equivalentemente:dxx

= dt dyy

= dt

que tiene como solucion:

x = aet y = bet

5.6. Considerar los siguientes campos vectoriales en R2

X = x∂

∂x− y ∂

∂y

Y = x∂

∂x+ y

∂y

a) Estudiar si (X, Y ) es una paralelizacion de R2 .b) Calcular el corchete de Lie [X, Y ] .c) Calcular las curvas integrales de los campos X e Y .d) Denotemos por u, v las cartas del atlas estereografico de la 2-

esfera.

u(r1, r2, r3) = (r1

1− r3

,r2

1− r3

)

v(r1, r2, r3) = (r1

1 + r3

,r2

1 + r3

)

y considerar los campos

u1∂

∂u1

− u2∂

∂u2

Page 125: Apuntes Geom Diferencial

CURVAS INTEGRALES 119

v1∂

∂v1

+ v2∂

∂v2

Estudiar si estos campos coinciden en la interseccion de sus dominios.

5.7. Considerar los siguientes campos vectoriales en R2

X = y∂

∂x

Y =x2

2

∂y

a) Estudiar si (X, Y ) es una paralelizacion de R2 .b) Calcular el corchete de Lie [X, Y ] .c) Calcular las curvas integrales de los campos X , Y y [X, Y ] .

Decir si son o no completos.d) Denotemos por u, v las cartas del atlas estereografico de la 2-

esfera.

u(r1, r2, r3) = (r1

1− r3

,r2

1− r3

)

v(r1, r2, r3) = (r1

1 + r3

,r2

1 + r3

)

y considerar los campos

K = u2∂

∂u1

L =(v1)2

2

∂v2

Estudiar si estos campos coinciden en la interseccion de sus dominios.

Solucion: a) Notemos que si x = 0 o y = 0 se tiene respectivamenteque en los puntos de la foma p = (0, y) el campo X se anula Xp = 0 oen los de la forma p = (x, 0) el campo Y se anula Yp = 0 . Entoncesse tiene que (Xp, Yp) no son linealmente independientes sobre puntosde los ejes coordenados. Por lo tato (X, Y ) no es una paralelizacion deR2 .

b) Notemos que

(5.1) Xx = y∂

∂xx = y

(5.2) Y y =x2

2

∂yy =

x2

2

(5.3) Y x = 0

(5.4) Xy = 0

De la ecuaciones anteriores se tiene que

(5.5) (XY − Y X)x = −x2

2

Page 126: Apuntes Geom Diferencial

120 6. CAMPOS Y FORMAS

(5.6) (XY − Y X)y = Xx2

2= yx

Por lo tanto

[X, Y ] = −x2

2

∂x+ yx

∂y

c) Al campo X le corresponde el sistema de ecuaciones diferenciales

dx

dt= y ,

dy

dt= 0

que obviamente tiene por solucion x = bt + a, y = b. Notemos quecada curva integral esta definida para todo t de donde se concluye queel campo X es completo.

Al campo Y le corresponde el sistema de ecuaciones diferenciales

dx

dt= 0 ,

dy

dt=x2

2

que tiene por solucion x = a, y = a2

2t + b. Como antes cada curva

integral esta definida para todo t de donde se concluye que el campoY es completo.

Para el campo [X.Y ] se obtiene el sistema

dx

dt= −x

2

2,

dy

dt= yx .

Notemos que la primera ecuacion es equivalente a −dxx2 = dt

2de donde

se obtiene que 1x

= t+C2

; es decir , x = 2t+C

. Si para t = 0 imponemos

que x = a la solucion toma la siguiente forma x = 2t+ 2

a

. De la segunda

ecuacion se tiene que dyy

= 2dtt+ 2

a

. Que implica que log y = 2 log(t+ 2a)+D

o equivalentemente y = eD(t+ 2a)2 . Imponiendo que para t = 0 , y = b

la solucion toma la siguiente forma y = ba2

4(t + 2

a)2 . Por lo tanto la

curva integral que para t = 0 pasa por el punto (a, b) viene dada por

x =2

t+ 2a

, y =ba2

4(t+

2

a)2

Notemos que para t = − 2a

la curva integral no esta definida. Por lotanto el campo [X, Y ] no es completo.

d) Para el punto p = (1, 0,0) se tiene que u1(p) = 1 , u2(p) = 0 ,v1(p) = 1 , v2(p) = 0 . Entonces Kp = 0 y Lp = 1

2∂∂v2

. Ello implica queKp = 0 y Lp 6= 0 . Por lo tanto K,L no coinciden en la interseccion.

5.8. Considerar los siguientes campos vectoriales en R2

A = −y ∂∂x

+ x∂

∂y

B = x(1− x2 − y2)∂

∂x+ y(1− x2 − y2)

∂y

Page 127: Apuntes Geom Diferencial

CURVAS INTEGRALES 121

a) Encontrar el conjunto de puntos de R2 en los que los camposA,B son independientes.

b) Calcular el corchete de Lie [A,B] .c) Calcular las curvas integrales del campo A . ¿Son sus curvas

integrales completas? ¿Es A un campo completo?

Solucion:a) Puesto que conocemos las funciones que determinan los campos

A,B en funcion de los campos coordenados ∂∂x, ∂∂y

y estos ultimos son

independientes. Entonces A,B seran independientes en aquellos puntosen los que sea no nulo el determinante de la matriz:(

−y xx(1− x2 − y2) y(1− x2 − y2)

)Este se anula en los puntos que satisfacen la ecuacion:

−(x2 + y2)(1− x2 − y2) = 0

que corresponden a la reunion del origen con la circunferencia unidad:

(0, 0) ∪ (x, y)|(1− x2 − y2) = 0Entonces seran independientes en el complementario

R2 \ ((0, 0) ∪ (x, y)|(1− x2 − y2) = 0)b) Un campo siempre se puede expresar en relacion a los campos

coordenados mediante la formula:

[A.B] = [A,B]x∂

∂x+ [A,B]y

∂y

Teniendo en cuenta que

Ax = −y Ay = xBx = x(1− x2 − y2) By = y(1− x2 − y2)

se obtiene que

[A,B]x = ABx−BAx = A(x(1− x2 − y2))−B(−y)= −y(1− 3x2 − y2)− x(x2y) + y(1− x2 − y2) = 0

[A,B]y = ABy −BAy = A(y(1− x2 − y2))−B(x)= −(−y)2xy + x(1− x2 − 3y2)− x(1− x2 − y2) = 0

Por lo que el corchete se anula [A,B] = 0 .c) Las curvas integrales del campo A se obtienen como solucion del

siguiente sistema de ecuaciones diferenciales:

dx

dt= −y ,

dy

dt= x

que tiene por solucion

x = A cos t−B sen t , y = B cos t+ A sen t .

Notemos que para t = 0 pasa por (A,B) .

Page 128: Apuntes Geom Diferencial

122 6. CAMPOS Y FORMAS

Las curvas integrales estan definidas para todo t . Entonces todaslas curvas integrales son completas y en consecuencia el campo es com-pleto.

Page 129: Apuntes Geom Diferencial

CAPıTULO 7

VARIEDADES Y CONEXIONESRIEMANNIANAS

1. Conexiones y derivada covariante

Sea Ξ(M, p) el conjunto de todos los campos tangentes de una va-riedad M definidos en el punto p ∈ M . Una conexion lineal en unpunto p es un operador lineal que asocia a cada vector tangente v en py a cada campo tangente X definido en p un vector tangente en p quese denotara por OvX y que verifica las siguientes propiedades:

(C1) Ov(αX + βY ) = αOvX + βOvYα, β ∈ R, v ∈ TpM, X, Y ∈ Ξ(M, p),

(C2) Oau+bvX = aOuX + bOvXa, b ∈ R, u, v ∈ TpM, X ∈ Ξ(M, p),

(C3) OvfX = v(f)Xp + f(p)OvX f ∈ C∞p (M, p), X ∈ Ξ(M, p).

Proposicion 1.1. Si v ∈ TpM y X, Y ∈ Ξ(M, p) tal que en unentorno abierto U de p se tiene que X|U = Y |U , entonces OvX = OvY .

Demostracion. Notese que Dom(X−X|U) = U y que X−X|U =0|U = 0(0|U) . Entonces Ov(X − X|U) = Ov(0|U) = Ov(0(0|U)) =0Ov(0|U) = 0 . Por lo tanto Ov(X) = Ov(X|U) . Analogamente seobtiene que OvY = Ov(Y |U) . Entonces OvX = OvY .

Si tenemos una conexion lineal para cada punto p ∈ M y V,Xson campos tangentes a una variedad M se puede definir la seccion delfibrado tangente OVX, cuyo dominio es DomV ∩DomX, y esta definidapor (OVX)p = OVpX .

Definicion 1.1. Llamaremos una conexion lineal sobre una varie-dad M a una familia que se obtiene tomando una conexion lineal encada punto p de M , de tal modo que si V,X son campos tangentes aM , entonces la seccion del fibrado tangente OVX es diferenciable.

Definicion 1.2. Sea x la carta identidad de Rm. Dados p ∈ Rm ,v ∈ TpRm y un campo X =

∑i fi

∂∂xi

definido en p . Entonces se define

OvX = v(f1)(

∂∂x1

)p+· · ·+v(fm)

(∂

∂xm

)p

. Notese que si tenemos campos

V y X =∑

i fi∂∂xi

, entonces OVX = (V f1) ∂∂x1

+ · · ·+ (V fm) ∂∂xm

.

Proposicion 1.2. El operador O definido en Rm es una conexionlineal.

123

Page 130: Apuntes Geom Diferencial

124 7. VARIEDADES Y CONEXIONES RIEMANNIANAS

Demostracion. Dados p ∈ Rm , v ∈ TpRm , camposX =∑

i fi∂∂xi

,

Y =∑

i gi∂∂xi

definidos en p y escalares α, β ∈ R . Entonces Ov(αX +

βY ) = v(αf1+βg1)(

∂∂x1

)p+· · ·+v(αfm+βgm)

(∂

∂xm

)p

= αv(f1)(

∂∂x1

)p+

· · ·+αv(fm)(

∂∂xm

)p+βv(g1)

(∂∂x1

)p+· · ·+βv(gm)

(∂

∂xm

)p

= αOv(X)+

βOv(Y ) . Con esto queda verificada la propiedad C1 . De modo ruti-nario se comprueba que se verifican el resto de las propiedades.

Proposicion 1.3. El operador O que asocia a dos campos tangen-tes V y X el campo tangente OVX , verifica las siguientes propiedades:

(CL1) OV (αX + βY ) = αOVX + βOV Yα, β ∈ R, V,X, Y ∈ Ξ(M),

(CL2) OaU+bVX = aOUX + bOVXa, b ∈ R, U, V,X ∈ Ξ(M),

(CL3) OV fX = V fX + fOVX f ∈ C∞(M), V,X ∈ Ξ(M).

Demostracion. CL1) Para un punto p ∈ M que este en el do-minio se tiene (OV (αX+βY ))p = OVp(αX+βY ) = αOVpX+βOVpY =(αOVX+βOV Y )p . Por lo tanto OV (αX+βY ) = αOVX+βOV Y, α, β ∈R, V,X, Y ∈ Ξ(M) .

CL2) Para cada p en el dominio de definicion se tiene que (OaU+bVX)p =OaUp+bVpX = aOUpX+bOVpX = (aOUX+bOVX)p . ConsecuentementeOaU+bVX = aOUX + bOVX a, b ∈ R, U, V,X ∈ Ξ(M) .

CL3) Para cada p en el dominio de definicion se tiene que (OV fX)p= OVpfX = (Vpf)Xp + f(p)OVpX = (V fX + fOVX)p . Por lo tantoOV fX = V fX + fOVX f ∈ C∞(M), V,X ∈ Ξ(M).

Sea σ : R → M una curva (diferenciable). Un campo tangente aM a lo largo de σ es una funcion diferenciable X : R → TM tal queπX = σ . Uno de estos campos es precisamente el campo velocidad de lacurva σ que se define del modo siguiente: σ(s) = Tsσ

(ddt

)s

. Notese quelos campos tangentes a M a lo largo de σ tienen estructura natural deespacio vectorial real y tambien de modulo sobre el anillo de funcionesC∞Domσ(R) .

Si M tiene una conexion O entonces para cada s ∈ Domσ se puededefinir un operador lineal Dσ(s) que asocia a cada campo X tangentea M a lo largo de σ un vector tangente a M en el punto σ(s) . Supon-gamos que X1, . . . , Xm es una paralelizacion definida en un entornoabierto de σ(s) . El campo X se puede expresar de modo unico comoX(t) =

∑iAi(t)X

iσ(t) Entonces definimos

Dσ(s)X =∑i

((dAidt

)s

X iσ(s) + Ai(s)Oσ(s)X

i

)Lema 1.1. El operador Dσ(s) es independiente de la paralelizacion

elegida.

Page 131: Apuntes Geom Diferencial

1. CONEXIONES Y DERIVADA COVARIANTE 125

Demostracion. Supongamos que X1, . . . , Xm e Y 1, . . . , Y m sonparalelizaciones definidas en un entorno abierto de σ(s) . El cam-po X se puede expresar como X(t) =

∑iAi(t)X

iσ(t) o bien X(t) =∑

iBi(t)Yiσ(t) . Supongamos queX i =

∑aijY

j . EntoncesX =∑

iAiXiσ =∑

iAi∑

j(aijσ)Yσj =

∑j(∑

iAi(aijσ))Yσj . De aquı se deduce que Bj =∑

iAi(aijσ) . Entonces se tiene∑j

((dBjdt

)sY jσ(s) +Bj(s)Oσ(s)Y

j)

=∑

j

((d(

Pi Ai(a

ijσ))

dt

)sY jσ(s) +

∑iAi(s)a

ij(σ(s))Oσ(s)Y

j)

=∑

j

(∑i(dAidt

(aijσ) + Aid(aijσ)

dt

)sY jσ(s) +

∑j

∑iAi(s)a

ij(σ(s))Oσ(s)Y

j

=∑

i

∑j

[(dAidt

)saij(σ(s))Y j

σ(s) + Ai(s)(d(aijσ)

dt

)sY jσ(s)

]+∑

i

∑j

[Ai(s)a

ij(σ(s))Oσ(s)Y

j]

=∑

i

[(dAidt

)s

∑j a

ij(σ(s))Y j

σ(s) + Ai(s)∑

j

(d(aijσ)

dt

)sY jσ(s)

]+∑

i

[Ai(s)

∑j a

ij(σ(s))Oσ(s)Y

j]

=∑

i

(dAidt

)sX iσ(s) +

∑iAi(s)Oσ(s)

∑j a

ijY

j

=∑

i

((dAidt

)sX iσ(s) + Ai(s)Oσ(s)X

i)

Notese que si X es un campo tangente a M a lo largo de σ, eloperador anterior permite definir un nuevo campo DσX tangente a Ma lo largo de σ , mediante la formula (DσX)(s) = Dσ(s)X .

Definicion 1.3. Si V e Y son campos tangentes a una variedad Mcon una conexion lineal O diremos que OV Y es la derivada covariantede Y segun el campo V . Si X es un campo tangente a M a lo largo deuna curva σ diremos que DσX es la derivada covariante de X .

Sea x es una carta en σ(s) y consideremos la paralelizacion ∂∂x1,

· · · , ∂∂xm

, entonces si el campo X tangente a M a lo largo de σ se

expresa como X =∑

iAi

(∂∂xi

se tiene que en el dominio de la carta

se verifica

DσX =∑i

(dAidt

(∂

∂xi

+ AiOσ∂

∂xi

)Lema 1.2. Sea σ : R → M una curva y sea Y un campo tangente

a M que induce una campo X = Y σ tangente a M a lo largo de M .Entonces DσX = OσY .

Demostracion. Supongamos que Y 1, . . . , Y m es una paraleliza-cion definida en un entorno abierto U de σ(s) . Si el campo Y verificaque Y |U =

∑iAiY

i . La parte del campo X que es tangente dentro deese entorno de puede expresar como X(t) =

∑iAiσ(t)Y i

σ(t) . Entoncesse tiene

Page 132: Apuntes Geom Diferencial

126 7. VARIEDADES Y CONEXIONES RIEMANNIANAS

Dσ(s)X =∑

i

((d(Aiσ)dt

)sY iσ(s) + Aiσ(s)Oσ(s)Y

i)

=∑

i

((σ(s)(Ai))Y

iσ(s) + Ai(σ(s))Oσ(s)Y

i)

= Oσ(s) (∑

iAiYi)

= Oσ(s)Y

Corolario 1.1. Sea v un vector tangente a M en un punto p ysean X,X ′ campos definidos en p . Si existe una curva σ que pase porp a velocidad v de modo que Xσ = X ′σ , entonces OvX = OvX ′ .

Demostracion. Por la proposicion anterior se tiene que OvX =Oσ(s)X = Dσ(s)(Xσ) = Dσ(s)(X

′σ) = Oσ(s)X′ = OvX ′ .

Dada x una carta de M , si consideramos la paralelizacion local∂∂x1, . . . , ∂

∂xm, entonces la conexion O determina la funciones Γkij : M →

R mediante la expresion:

O ∂∂xi

∂xj=∑k

Γkij∂

∂xk

Si denotaremos por Γkij = Γkijx−1 la representacion coordenada de en la

carta x, entonces las derivadas covariantes de los campos coordenadostambien se puede expresar como

O ∂∂xi

∂xj=∑k

(Γkij x)∂

∂xk=∑k

Γkij(x1, · · · , xm)∂

∂xk

Ambos tipos de funciones, Γkij,Γkij, se denominaran como los sımbolos

de Christoffel de la conexion en la carta x .Dada una curva σ y una carta x denotemos por c = xσ la repre-

sentacion coordenada de la curva. Si X es una campo tangente a Ma lo largo de la curva σ, supongamos que en el dominio de la carta

x se tiene que X =∑

j Aj

(∂∂xj

. Notemos que σ =∑

dcidt

(∂∂xi

.

Page 133: Apuntes Geom Diferencial

1. CONEXIONES Y DERIVADA COVARIANTE 127

Sustituyendo σ se obtiene

OσX =∑j

(dAjdt

(∂

∂xj

+ AjOP dcidt

( ∂∂xi

∂xj

)

=∑j

(dAjdt

(∂

∂xj

+ Aj∑i

dcidt

(O ∂

∂xi

∂xj

)

=∑k

dAkdt

(∂

∂xk

+∑i,j

Ajdcidt

(O ∂

∂xi

∂xj

=∑k

dAkdt

(∂

∂xk

+∑i,j

Ajdcidt

(∑k

Γkij∂

∂xk

=∑k

dAkdt

(∂

∂xk

+∑k

∑i,j

dcidt

ΓkijσAj

(∂

∂xk

=∑k

(dAkdt

+∑i,j

dcidt

Γkij(c)Aj

)(∂

∂xk

Definicion 1.4. Se dice que un campo X tangente a una variedadM a lo largo de una curva σ es paralelo si DσX = 0 .

Teniendo en cuenta los calculos anteriores se tiene el resultado si-guiente.

Proposicion 1.4. Sea X un campo tangente a una variedad Ma lo largo de una curva σ y sea x una carta cuyo dominio corte a

la curva y tal que X =∑

j Aj

(∂∂xj

en σ−1(Domx) y denotemos

c = xσ . Entonces X es paralelo en σ−1(Domx) si y solo si se verificanlas ecuaciones

dAkdt

+∑i,j

dcidt

Γkij(c)Aj = 0 , k = 1 · · ·m.

que se llaman el sistema de ecuaciones diferenciales de los campos pa-ralelos a lo largo de la curva σ en el dominio de la carta x .

Proposicion 1.5. Sea una variedad M con una conexion linealO , una curva σ con dominio conexo y t0 ∈ Domσ . Entonces dadoX0 ∈ Tσ(t0)M existe unico campo X tangente a M a lo largo de σ quees paralelo y tal que X(t0) = X0 .

Demostracion. Si la curva σ esta dada, entonces una carta x enσ(t0) determina funciones c = xσ y sus componentes ci = xiσ de talmodo que el sistema anterior es un sistema de ecuaciones diferencia-les de primer orden en las variables A1, · · · , Am . Ademas existe unentorno V (t0) = (t0 − ε, t0 + ε) ⊂ σ−1(Domx) tal que para cada valo-res iniciales de las variables A1, · · · , Am existen unas unicas funciones

Page 134: Apuntes Geom Diferencial

128 7. VARIEDADES Y CONEXIONES RIEMANNIANAS

definidas en V (t0) , que sean solucion del sistema y que tengan los va-lores iniciales dados. Ahora para cada t ∈ Domσ , si t > t0 se tieneque [t0, t] ⊂ Domσ por ser el dominio de la curva conexo. Ademasal ser [t0, t] compacto existe una particion t0 < t1 < · · · < tk = tde modo que en cada subintervalo verifica condiciones de existencia yunicidad del correspondiente sistema de ecuaciones diferenciales. Cada

X0 ∈ Tσ(t0)M determina valores iniciales X0 =∑

j Aj(t0)(

∂∂xj

)σ(t0)

de

funciones A1, · · · , Am definidas en V (t0) y que son unicas verificandoel sistema de ecuaciones y con los valores iniciales dados. Tomemos

entonces el campo X =∑

j Aj

(∂∂xj

. Ahora procedemos de modo

inductivo tomando X1 ∈ Tσ(t1)M hasta obtener el campo X definidoen un entorno de [t0, t] . Analogamente se procede en el caso t < t0 .La unicidad de las soluciones de ecuaciones diferenciales anteriores ga-rantizan que el campo X es el unico campo paralelo definido en Domσcon valores iniciales X0 .

Definicion 1.5. Se dice que una curva σ de una variedad M esuna geodesica respecto O si σ es paralelo; es decir, si Dσσ = 0 .

Proposicion 1.6. Sea una variedad M con una conexion lineal O .Dado un v ∈ TpM , entonces existe una unica geodesica γ que pasa porp a velocidad v. Denotaremos esta geodesica por γ(t, p, v) .

Demostracion. Sea x una carta en el punto p , de manera que

σ =∑ dck

dt

(∂∂xk

, entonces σ es paralelo a lo largo de σ y en el

dominio de la carta si y solo si se verifica la ecuacion diferencial desegundo orden siguiente:

d2ckdt

+∑i,j

dcidt

Γkij(c)dcjdt

= 0 , k = 1, · · · ,m.

Esta tiene solucion unica bajo las condiciones iniciales correspondien-tes; en este caso que pase por el punto p a velocidad v .

Definicion 1.6. La funcion expp : TpM →M definida por expp(v) =γ(1, p, v) se llama exponencial.

Problemas

1.1. Sea σ una curva en una variedad M y sea Dσ el operadorderivada covariante que actua sobre los campos X tangentes a M a lolargo de σ . Probar que este operador verifica las siguientes propiedades:

D1: Dσ(αX + βY ) = αDσX + βDσY α, β ∈ R, X, Y tan-gentes a M a lo largo de σ ,

D2: DσφX = dφdtX+φDσX φ ∈ C∞Domσ(R), X tangente a M

a lo largo de σ .

Page 135: Apuntes Geom Diferencial

2. VARIEDADES RIEMANNIANAS 129

Solucion: D1) Sea (Z1, · · · , Zm) una paralelizacion en un punto dela curva. Supongamos que respecto dicha paralizacion se tiene que X =∑m

i fiZi , Y =

∑mi giZ

i . Entonces

Dσ(αX + βY ) = Dσ(∑m

i (αfi + βgi)Zi)

=∑m

i

(d(αfi+βg

i)dt

Zi + (αfi + βgi)OσZi)

= α∑m

i

(dfidtZi + fiOσZi

)+ β

∑mi

(dgi

dtZi + giOσZi

)= αDσX + βDσY

D2) Con la misma notacion:

DσφX = Dσ(∑m

i φfiZi)

=∑m

i

(d(φfi)dt

Zi + φfiOσZi)

=∑m

idφdtfiZ

i +∑m

i

(φdfidtZi + φfiOσZi

)= dφ

dtX + φDσX

1.2. Sea σ una curva en R3 y sea Dσ el operador derivada cova-riante que actua sobre los campos X tangentes a R3 a lo largo de

σ . Sea r : R3 → R3 la carta identidad. Sean(

∂∂r1

)σ,(

∂∂r2

)σ,(

∂∂r1

la restriccion de los campos coodenados sobre la curva. Supongamosque un campo X tangente a R3 a lo largo de σ es tal que X =

f1

(∂∂r1

+ f2

(∂∂r2

+ f3

(∂∂r1

. Probar que la derivada covariante

de X coincide con la derivada normal; es decir:

DσX =df1

dt

(∂

∂r1

+df2

dt

(∂

∂r2

+df3

dt

(∂

∂r1

.

1.3. O la conexion riemanniana de R3 . Supongamos que Σ ⊂ R3 esuna subvariedad. Notese que para cada p ∈ Σ se tiene la desomposicioncanonica TpR3 ∼= TpΣ⊕norTpΣ . Por lo que cada v ∈ TpR3 descomponede modo canonico como v = tanv + norv . Definamos el operador Opara la variedad Σ por la formula OuX = tanOuX donde X es unaextension local de X a R3 . Probar que O es la conexion riemannianade Σ .

1.4. O la conexion riemanniana de R3 . Supongamos que Σ ⊂ R3

es una subvariedad con la conexion inducida O . Sea σ : R → Σ unacurva y sea X un campo tangente a Σ a lo largo de σ . Probar que Xes paralelo (OσX = 0) si y solo si OσX es normal a Σ .

1.5. Utilizar los problemas anteriores para probar que el unico pa-ralelo de la 2-esfera que es geodesica es el ecuador.

2. Variedades riemannianas

Definicion 2.1. LLamaremos producto interno de un espacio vec-torial real a una aplicacion 〈−,−〉 : V × V → R bilineal simetrica ydefinida positiva. Una metrica riemanniana asocia a cada p de M

Page 136: Apuntes Geom Diferencial

130 7. VARIEDADES Y CONEXIONES RIEMANNIANAS

un producto interno 〈−,−〉p : TpM × TpM → R de forma diferen-ciable; es decir, si X, Y son campos tangentes entonces la funcion〈X, Y 〉(p) = 〈Xp, Yp〉 , definida para p ∈ DomX ∩ DomY , es dife-renciable. Un variedad M provista de un metrica riemanniana diremosque es una variedad riemanniana. Con las mismas condiciones de dife-renciabilidad anteriores si cada producto 〈−,−〉p : TpM ×TpM → R esbilineal simetrico y no singular se dice que es una metrica pseudorie-manniana y que (M, 〈−,−〉) es una variedad pseudoriemanniana.

Ejemplo 2.1. En Rn podemos considerar la siguiente metrica: Para

cada p ∈ Rn si u =∑

k uk

(∂∂rk

)p

y v =∑

k vk

(∂∂rk

)p

, entonces se

define la metrica mediante la formula 〈u, v〉p =∑

k ukvk .

Proposicion 2.1. Sea N una variedad con una metrica 〈−,−〉 .Si f : M → N es un inmersion global, entonces f induce una metricariemanniana en M definida por la formula

〈u, v〉p = 〈f∗pu, f∗pv〉fp.

Demostracion. Del hecho de que para cada p ∈ M el productointerno 〈−,−〉f(p) sea bilineal, simetrico y definido positivo, y teniendoen cuenta que Tpf es monomorfismo, se tiene de modo rutinario que〈−,−〉p es un producto interno. Si f : M → N es un inmersion global,entonces por la proposicion 1.1 para cada p ∈M existen cartas x de Men p e y de N en f(p) tal que Dom x ⊂ Dom(yf) , yif |Domx = xi para1 ≤ i ≤ m y ademas yif |Domx = 0 para m ≤ i ≤ n . De aquı se deduce

que f |Domx es inyectiva y ademas Tpf(

∂∂xi

)p

=(

∂∂yi

)f(p)

para 1 ≤

i ≤ m . Por lo que la funcion 〈 ∂∂xi, ∂∂xj〉 = 〈 ∂

∂yi, ∂∂yj〉f es diferenciable.

Ahora dados dos campos X, Y definidos en un punto p si tomamoscartas x e y como las anteriores se tiene que si X|Domx =

∑iAi

∂∂xi

,

Y |Domx =∑

j Bj∂∂xj

, entonces 〈X, Y 〉|Domx =∑

i,j AiBj〈 ∂∂xi ,∂∂xj〉 que

es diferenciable. Por lo tanto se tiene que 〈X, Y 〉 es diferenciable.

Definicion 2.2. Se dice que f : M → N es una inmersion isometri-ca si para p ∈ Dom f y u, v ∈ TpM se tiene que 〈u, v〉p = 〈f∗pu, f∗pv〉fp .Si ademas f es un difeomorfismo local diremos que f es una isometrıalocal y si f es un difeomorfismo global de M sobre N diremos que Mes isometrica a N .

Ejemplo 2.2. Si Σk es una subvariedad de Rn entonces la inclusioni : Σk → Rn es una inmersion que induce la siguiente metrica: Para cadap ∈ Σk si u, v ∈ TpΣk , entonces 〈u, v〉p = 〈i∗pu, i∗pv〉ip . Por ejemplo, laesfera unidad Sn−1 tiene estructura natural de variedad riemannianainducida por la inclusion natural Sn−1 → Rn .

Ejemplo 2.3. En Rn+ = (r1, . . . , rn)|rn > 0 podemos conside-

rar la siguiente metrica: Para cada p = (r1, . . . , rn) ∈ Rn+ si u =

Page 137: Apuntes Geom Diferencial

VARIEDADES RIEMANNIANAS 131∑k uk

(∂∂rk

)p

y v =∑

k vk(∂∂rk

)p

, entonces se define 〈u, v〉p = 1r2n

∑k ukvk .

Esta variedad riemanniana se denomina espacio hiperbolico n-dimensionaly se denota por Hn .

Ejemplo 2.4. En Rn podemos considerar la siguiente metrica: Pa-

ra cada p = (r1, . . . , rn) ∈ Rn si u =∑

k uk

(∂∂rk

)p

y v =∑

k vk

(∂∂rk

)p

,

entonces se define 〈u, v〉p = −u1v1 +∑n

k=2 ukvk . Esta variedad pseu-doriemanniana se denomina espacio de Minkovski n-dimensional y lodenotaremos por Mn .

Recordemos que una producto interno en un espacio vectorial Vcon una base (e1, . . . en) queda determinado por la matriz simetrica(〈ei, ej〉) . De modo similar una metrica riemanniana en el dominiode una carta x queda determinado por las funciones definidas porgij = 〈 ∂

∂xi, ∂∂xj〉 que llamaremos coeficientes de la metrica respecto de

la carta x . A las representaciones coordenadas las denotaremos porgij = gijx

−1 .

Problemas

2.1. Geometrıa del catenoide y helicoide. Veanse las Figuras 1 2 .a) Probar que el subconjunto

H = (u cos v, u sen v, v)|u, v ∈ Res una subvariedad regular de dimension dos de R3 . A la subvarie-dad regular H la llamaremos Helicoide, demostrar que el Helicoide esdifeomorfo a R2 .

b) Probar que la aplicacion f : R2 → R3 ,

f(θ, z) = (cos θ cosh z, sen θ cosh z, z)

es una inmersion no inyectiva y que C =Imf tiene estructura de sub-variedad regular de R3 de dimesion dos. A la subvariedad regular C lallamaremos Catenoide, demostrar que el Cateniode es difeomorfo a uncilindro S1 × R .

c) Probar que la aplicacion del Helicoide en el Catenoide g : H → Cdefinida por

g(u cos v, u sen v, v) = (√

1 + u2 cos v,√

1 + u2 sen v, arcsenhu)

es un difeomorfismo local. ¿Es tambien una aplicacion recubridora?d) Sea φ la carta del Helicoide definida por φ(u cos v, u sen v, v) =

(u, v) . Calcular los coeficientes de la metrica gHij (u, v) del Helicoide.e) Sea ψ la carta del Catenoide cuyo dominio es el subconjunto

(cos θ cosh z, sen θ cosh z, z)|0 < θ < 2πy esta definida por ψ(cos θ cosh z, sen θ cosh z, z) = (θ, z) . Calcular loscoeficientes de la metrica gCij(θ, z) del Catenoide.

Page 138: Apuntes Geom Diferencial

132 7. VARIEDADES Y CONEXIONES RIEMANNIANAS

f) Probar que la aplicacion del Helicoide en el Catenoide g : H → Cdefinida por

g(u cos v, u sen v, v) = (√

1 + u2 cos v,√

1 + u2 sen v, arcsenhu)

es una isometrıa local.

Figura 1. Helicoide

Solucion: a) Definamos la funcion ϕ : R3 → R mediante la formulaϕ(x, y, z) = x sen z − y cos z . Notemos que un punto del helicoideverifica que

x sen z − y cos z = u cos v sen v − u sen v cos v = 0

y recıprocamente si se verifica que x sen z − y cos z = 0 , entonces sihacemos que v = z y suponemos que cos v 6= 0, se tiene que podemostomar u = x

cos v. Entonces

(u cos v, u sen v, v) = (x,x

cos vsen v, z) = (x, y, z)

y de modo similar se procede si sen v 6= 0 . Por lo tanto el heliciode esla fibra del cero de la funcion ϕ .

Si calculamos las derivadas parciales se tiene ∂ϕ∂x

= sen z , ∂ϕ∂y

=

cos z , ∂ϕ∂z

= x cos z + y sen z . Puesto que el seno y el coseno no se

Page 139: Apuntes Geom Diferencial

VARIEDADES RIEMANNIANAS 133

Figura 2. Catenoide

anular simultaneamente se tiene que efectivamente el rango es uno yel Helicoide H es una subvariedad regular de R3 . Para lo cual hemosaplicado el teorema 4.2 del capıtulo 2 .

Para ver que es difeomorfo a R2 consideremos la aplicacion h : R2 →H dada por h(u, v) = (u cos v, u sen v, v) . Notemos que inh : R2 → R3

es inyectiva y su matrız jacobiana : cos v −u sen vsen v u cos v

0 1

tiene rango dos . Por lo tanto la aplicacion h es un homeomorfismolocal. Entonces h es un difeomorfismo.

b) Definamos la funcion ψ : R3 → R mediante la formula ψ(x, y, z) =x2 + y2 − cosh2 z . Notemos que un punto del catenoide verifica que

x2 + y2 − cosh2 z = cos2 θ cosh2 z + sen2 θ cosh2 z − cosh2 z = 0

y recıprocamente si se verifica que 0 = x2 + y2 − cosh2 z , entonces(x

cosh z

)2+(

ycosh z

)2= 1 . Por lo tanto existe θ tal que cos θ = x

cosh zy

sen θ = ycosh z

. Entonces

(cos θ cosh z, sen θ cosh z, z) = (x, y, z)

Page 140: Apuntes Geom Diferencial

134 7. VARIEDADES Y CONEXIONES RIEMANNIANAS

Si calculamos las derivadas parciales se tiene ∂ψ∂x

= 2x , ∂ψ∂y

= 2y ,∂ϕ∂z

= −2 cosh z senh z .Puesto para ningun punto del catenide se anulan todas las derivadas

parciales se sigue que efectivamente el rango es uno y el Catenoide Ces una subvariedad regular de R3 .

Para ver que es difeomorfo a S1 × R consideremos la aplicacionh : R2 → C dada por f(θ, z) = (cos θ cosh z, sen θ cosh z, z) . Notemosque in f : R2 → R3 es pediodica en θ y su matrız jacobiana : − sen θ cosh z cos θ senh z

cos θ cosh z sen θ senh z0 1

tiene rango dos . Por lo tanto la aplicacion f es un homeomorfismolocal que factoriza a un difeomorfismo f : S1 ×R→ C . Entonces f esun difeomorfismo.

c) Definamos la aplicacion g : H → C por la formula

g(u cos v, y sen v, v) = (√

1 + u2 cos v,√

1 + u2 sen v, arcsenhu) .

La representacion coordenada de este cambio viene dado por θ = v yz = arcsenhu . Por lo que g es diferenciable. La matrız jacobiana(

0 1√1+u2

1 0

)tiene rango dos . Por lo tanto la aplicacion g es un homeomorfismolocal.

En este caso efectivamente se trata de una aplicacion recubridora.Un abierto del catenoide de la forma p ∈ C|θ0 < θ(p) < θ0 + 2πtiene como preimagen la reunion disjunta de abiertos de la forma p ∈H|θ0 + 2(k − 1)π < v(p) < θ0 + 2kπ donde k es un entero ademas larestriccion de g a cada uno de estos abiertos es un homeomorfismo.

d) Sea φ = (u, v) del heliciode y (x, y, z) la carta identidad de R3 .Denotemos por in : H → R3 la inclusion del helicoide en R3 , entoncesse verifica que x in = u cos v y in = u sen v y z in = v . entonces setiene que in∗

∂∂u

= in∗∂∂u

(x) ∂∂x

+ in∗∂∂u

(y) ∂∂y

+ in∗∂∂u

(x) ∂∂z

= ∂x in∂u

∂∂x

+∂y in∂u

∂∂y

+ ∂z in∂u

∂∂z

= cos v ∂∂x

+sen v ∂∂x

. Similarmente se tiene que in∗∂∂v

=

in∗∂∂v

(x) ∂∂x

+ in∗∂∂v

(y) ∂∂y

+ in∗∂∂v

(x) ∂∂z

= ∂x in∂v

∂∂x

+ ∂y in∂v

∂∂y

+ ∂z in∂v

∂∂z

=

−u sen v ∂∂x

+ u cos v ∂∂x

+ ∂∂z

.Por lo tanto los coeficientes metricos en la carta son los siguientes

gH11(u, v) = 1 gH12(u, v) = 0 gH22(u, v) = 1 + u2

e) Sea ψ = (θ, z) del heliciode y (x, y, z) la carta identidad de R3 .Denotemos por in : C → R3 la inclusion del catenoide en R3 , entoncesse verifica que x in = cos θ cosh z y in = u sen θ cosh z y z in = z . en-tonces se tiene que in∗

∂∂θ

= in∗∂∂θ

(x) ∂∂x

+ in∗∂∂θ

(y) ∂∂y

+ in∗∂∂θ

(x) ∂∂z

=

Page 141: Apuntes Geom Diferencial

3. LONGITUDES DE CURVAS Y VOLUMENES 135

∂x in∂θ

∂∂x

+ ∂y in∂θ

∂∂y

+ ∂z in∂θ

∂∂z

= − sen θ cosh z ∂∂x

+ cos θ cosh z ∂∂x

+ . Simi-

larmente se sigue que in∗∂∂z

= in∗∂∂z

(x) ∂∂x

+ in∗∂∂z

(y) ∂∂y

in∗∂∂z

(x) ∂∂z

=∂x in∂z

∂∂x

+ ∂y in∂z

∂∂y

+ ∂z in∂z

∂∂z

= cos θ senh z ∂∂x

+ sen θ senh z ∂∂x

+ ∂∂z

.

Por lo tanto los coeficientes metricos en la carta son los siguientes

gC11(θ, z) = cosh2 z gC12(θ, z) = 0 gC22(θ, z) = 1 + senh2 z

f) Para ver que g : H → C definida por

g(u cos v, u sen v, v) = (√

1 + u2 cos v,√

1 + u2 sen v, arcsenhu)

es una isometrıa local. Teniendo en cuenta el apartado c) se tieneg∗

∂∂u

= g∗∂∂u

(θ) ∂∂θ

+ g∗∂∂u

(z) ∂∂z

= ∂θg∂u

∂∂θ

+ ∂zg∂u

∂∂z

= 1√1+u2

∂∂z

g∗∂∂v

= g∗∂∂v

(θ) ∂∂θ

+ g∗∂∂v

(z) ∂∂z

= ∂θg∂v

∂∂θ

+ ∂zg∂v

∂∂z

= ∂∂θ

Ahora facilmente se comprueba que〈g∗ ∂∂u , g∗

∂∂u〉 = 〈 1√

1+u2

∂∂z, 1√

1+u2

∂∂z〉 = 1

1+u2 〈 ∂∂z ,∂∂z〉

= 11+u2 (1 + senh2 z) = 1

〈g∗ ∂∂v , g∗∂∂v〉 = 〈 ∂

∂θ, ∂∂θ〉 = cosh2 z = 1 + u2

En los demas casos los coeficientes metricos son iguales a cero.

3. Longitudes de curvas y volumenes

Definicion 3.1. Sea σ : R → M una curva en una variedad rie-manniana. Sea un intervalo [a, b] ⊂ Domσ . La longitud de la curvaentre a y b se define como la integral

lba =

∫ b

a

〈σ, σ〉12dt.

Sea V un espacio vectorial con un producto interno. Si (e1, . . . , em)es una base ortonormal de V , entonces el volumen del paralelepıpe-do expandido por v1 =

∑a1jej, . . . , vm =

∑amjej viene dado por

vol(v1, . . . , vm) = det(aij) . Notemos que

gij = 〈vi, vj〉 = 〈∑k

aikek,∑l

ajlel〉 =∑

aikajlδkl =∑k

aikajk

De donde deducimos que vol(v1, . . . , vm) =√

det(gij) donde el signo delvolumen se puede tomar positivo si det(aij) > 0 o negativo si det(aij) <0 .

Recordemos tambien que si respecto una base v1, . . . , vm tenemoslos coeficientes gij = 〈vi, vj〉 y respecto a otra base w1, . . . , wm losnuevos coeficientes hij = 〈wi, wj〉 . Si ws =

∑l cslvl entonces hij =

〈∑

k cikvk,∑

l cjlvl〉 =∑

kl cikgklcjl .Sea R una“region” contenida en el dominio de una carta x donde

los coeficientes metricos son gij = gijx−1 , entonces el volumen de esta

region se define como

vol(R) =

∫xR

√det(gij(r)) dr1 . . . drm .

Page 142: Apuntes Geom Diferencial

136 7. VARIEDADES Y CONEXIONES RIEMANNIANAS

El volumen es independiente de la carta elegida supongamos que Resta contenido en el dominio de una carta y con coeficientes metricoshij = hijy

−1 y con misma orientacion que x ; es decir, det(∂xi∂yj

) > 0 .

Al realizar el cambio r = xy−1(s) se tiene que∫xR

√det(gkl(r)) dr1 . . . drm

=

∫yR

|det(∂ri∂sj

)|√

det(gkl(xy−1(s))) ds1 . . . dsm

=

∫yR

√det

(∂xi∂yk

)y−1(s)

det(gkl(y−1(s)))det

(∂xl∂yj

)y−1(s)

ds1 . . . dsm

=

∫yR

√det(hij(s)) ds1 . . . dsm .

Problemas

3.1. Calcular el area de la 2-esfera de radio r > 0 .

3.2. Calcular el area del plano proyectivo obtenido a partir de la2-esfera de radio r > 0 .

4. Conexiones riemannianas

Definicion 4.1. Se dice que una conexion lineal O sobre una va-riedad M es simetrica si para X, Y campos tangentes a M se verificaque

OXY − OYX = [X, Y ].

Se dice que una conexion lineal O sobre una variedad riemanniana(M, 〈−,−〉) es compatible con la metrica si para X, Y, Z campos tan-gentes a M se verifica que

X〈Y, Z〉 = 〈OXY, Z〉+ 〈X,OXZ〉.

En una variedad riemanniana una conexion riemanniana es una cone-xion simetrica y compatible con la metrica.

Proposicion 4.1. Una conexion O es simetrica si y solo si paracada carta los sımbolos de Christoffel satisfacen que Γkij = Γkji .

Demostracion. Supongamos que

O ∂∂xi

∂xj=∑k

Γkij∂

∂xk

O ∂∂xj

∂xi=∑k

Γkji∂

∂xk

Page 143: Apuntes Geom Diferencial

4. CONEXIONES RIEMANNIANAS 137

Entonces por ser la conexion simetrica

O ∂∂xi

∂xj− O ∂

∂xj

∂xi=

[∂

∂xi,∂

∂xj

]= 0

Se donde se concluye que Γkji = Γkij . El recıproco se prueba sin dificul-tad.

Proposicion 4.2. La metrica de una variedad riemanniana induceun isomorfismo canonico entre el fibrado tangente y el cotangente.

Demostracion. Supongamos que u ∈ TpM Entonces considere-mos la aplicacion lineal θ(u) : T ∗pM → R definida por θ(u)(v) = 〈u, v〉p .Por ser el producto interno no singular, se tiene que θ : TM → T ∗Mes una biyeccion que preserva fibras y es lineal en cada una de ellas.Ademas para x carta de M se tiene que gij = 〈 ∂

∂xi, ∂∂xj〉 son funcio-

nes diferenciables. Entonces el cambio de cartas ∗xθx−1((r1, · · · , rm),(s1, · · · .sm)) = ((r1, · · · , rm), (

∑sigi1(r), · · · .

∑sigim(r))) es un difeo-

morfismo. Por lo tanto θ es un difeomorfismo global y sobreyectivo.

Como consecuencia del resultado anterior en una variedad rieman-niana una 1-forma determina un unico campo y recıprocamente.

Teorema 4.1. (Levi-Civita) Dada una variedad riemanniana Mentonces existe una unica conexion simetrica compatible con su metricadeterminada por la siguiente expresion:

〈Z,OYX〉 = 12(X〈Y, Z〉+ Y 〈Z,X〉 − Z〈X, Y 〉−〈[X,Z], Y 〉 − 〈[Y, Z], X〉 − 〈[X, Y ], Z〉)

Demostracion. Una conexion compatible con la metrica paracampos X, Y, Z debe verificar las formulas:

X〈Y, Z〉 = 〈OXY, Z〉+ 〈Y,OXZ〉(4.1)

Y 〈Z,X〉 = 〈OYZ,X〉+ 〈Z,OYX〉Z〈X, Y 〉 = 〈OZX, Y 〉+ 〈X,OZY 〉

De estas, aplicando que O es simetrica, se infiere queX〈Y, Z〉+ Y 〈Z,X〉 − Z〈X, Y 〉 = 〈[X,Z], Y 〉+ 〈[Y, Z], X〉+ 〈[X, Y ], Z〉

+2〈Z,OYX〉de donde se obtiene que

〈Z,OYX〉 =1

2(X〈Y, Z〉+ Y 〈Z,X〉 − Z〈X, Y 〉(4.2)

− 〈[X,Z], Y 〉 − 〈[Y, Z], X〉 − 〈[X, Y ], Z〉)Entonces cualquier conexion riemanniana debe verificar la formula an-terior. Es importante observar que para cada par de campos X, Y eloperador

ω(Z) = 12(X〈Y, Z〉+ Y 〈Z,X〉 − Z〈X, Y 〉−〈[X,Z], Y 〉 − 〈[Y, Z], X〉 − 〈[X, Y ], Z〉)

Page 144: Apuntes Geom Diferencial

138 7. VARIEDADES Y CONEXIONES RIEMANNIANAS

es lineal respecto funciones diferenciables con valores reales, en efecto:

ω(fZ) =1

2(X〈Y, fZ〉+ Y 〈fZ,X〉 − fZ〈X, Y 〉

− 〈[X, fZ], Y 〉 − 〈[Y, fZ], X〉 − 〈[X, Y ], fZ〉)

=1

2(fX〈Y, Z〉+ fY 〈Z,X〉 − fZ〈X, Y 〉+Xf〈Y, Z〉+ Y f〈Z,X〉

− f〈[X,Z], Y 〉 − f〈[Y, Z], X〉 − f〈[X, Y ], Z〉− 〈XfZ, Y 〉 − 〈Y fZ,X〉)= fω(Z)

donde hemos aplicado la formula dada en el ejercicio 2.4 del capıtulo6. Tambien se comprueba facilmente que ω(Z + Z ′) = ω(Z) + ω(Z ′) .

Si suponemos que O,O′ son conexiones riemannianas, para X, Ycampos tangentes se tiene que 〈−,OYX〉 = ω = 〈−,O′YX〉 . De lasproposiciones 2.3 (capıtulo 6) y 4.2 , se sigue que OYX = O′YX .

Para probar la existencia, notemos que cada dos campos X, Y de-terminan una 1-forma ω . Teniendo en cuenta la proposicion 2.3 delcapıtulo 6 y la anterior proposicion 4.2 , se obtiene que los camposX, Y determinan un unico nuevo campo OYX verificando la formula4.2 .

Es inmediato comprobar que se verifican la propiedades linealesCL1, CL2 . Para probar CL3 consideremos

〈Z,OY fX〉 =1

2(fX〈Y, Z〉+ Y 〈Z, fX〉 − Z〈fX, Y 〉

− 〈[fX,Z], Y 〉 − 〈[Y, Z], fX〉 − 〈[fX, Y ], Z〉)Teniendo en cuenta que

Y 〈Z, fX〉 = Y f〈Z,X〉+ fY 〈Z,X〉−Z〈fX, Y 〉 = −Zf〈X, Y 〉 − fZ〈X, Y 〉−〈[fX,Z], Y 〉 = Zf〈X, Y 〉+ f〈[Z,X], Y 〉−〈[fX, Y ], Z〉 = Y f〈X,Z〉+ f〈[Y,X], Z〉

se tiene que

〈Z,OY fX〉 = f〈Z,OYX〉+1

2(2Y f〈Z,X〉)

= 〈Z, fOYX + Y fX〉Por lo tanto OY fX = fOYX + Y fX y se verifica la propiedad CL3 .

La comprobacion de que esta conexion es simetrica se obtiene res-tando terminos en las siguientes formulas:

〈Z,OYX〉 = 12(X〈Y, Z〉+ Y 〈Z,X〉 − Z〈X, Y 〉−〈[X,Z], Y 〉 − 〈[Y, Z], X〉 − 〈[X, Y ], Z〉)

〈Z,OXY 〉 = 12(Y 〈X,Z〉+X〈Z, Y 〉 − Z〈Y,X〉−〈[Y, Z], X〉 − 〈[X,Z], Y 〉 − 〈[Y,X], Z〉)

Page 145: Apuntes Geom Diferencial

CONEXIONES RIEMANNIANAS 139

lo que implica que

〈Z,OYX − OXY 〉 = 〈[Y,X], Z〉 = 〈Z, [Y,X]〉para todo Z . Por lo tanto OYX − OXY = [Y,X] .

Ahora sumando

〈OXY, Z〉 = 12(Y 〈X,Z〉+X〈Z, Y 〉 − Z〈Y,X〉−〈[Y, Z], X〉 − 〈[X,Z], Y 〉 − 〈[Y,X], Z〉)

〈Z,OXY 〉 = 12(Z〈X, Y 〉+X〈Y, Z〉 − Y 〈Z,X〉−〈[Z, Y ], X〉 − 〈[X, Y ], Z〉 − 〈[Z,X], Y 〉)

se sigue que la conexion es riemanniana

〈OXY, Z〉+ 〈Z,OXY 〉 = X〈Y, Z〉

Si en la expresion anterior tomamos Z = ∂∂xl

, Y = ∂∂xj

y X = ∂∂xi

se obtiene que ∑k

Γkjiglk =1

2

(∂gjl∂xi

+∂gli∂xj− ∂gij∂xl

)Si denotamos por (gln) la matriz inversa de la matriz (gln) se obtieneque ∑

k,l

Γljigklgln =

1

2

∑l

gln(∂gjl∂xi

+∂gli∂xj− ∂gij∂xl

)y por lo tanto:

Γnji =1

2

∑l

gln(∂gjl∂xi

+∂gli∂xj− ∂gij∂xl

)Problemas

4.1. Considerar en R2 \ (0, 0) la carta inclusion que tiene comocoordenadas x, y . Supongamos que la metrica viene dada por

(gij) =

( 1x2+y2

0

0 1x2+y2

)a) Probar que los coeficientes de Christoffel vienen dados por

Γ111 = − x

x2 + y2

Γ112 = − y

x2 + y2= Γ1

21

Γ122 =

x

x2 + y2

Γ211 =

y

x2 + y2

Γ212 = − x

x2 + y2= Γ2

21

Page 146: Apuntes Geom Diferencial

140 7. VARIEDADES Y CONEXIONES RIEMANNIANAS

Γ222 = − y

x2 + y2

y que el sistema de ecuaciones diferenciales de las de las geodesicas esel siguiente:

− x x′2

x2 + y2− 2 y x′ y′

x2 + y2+

x y′2

x2 + y2+ x′′ = 0

y x′2

x2 + y2− 2x x′ y′

x2 + y2− y y′2

x2 + y2+ y′′ = 0

donde x′ = dxdt

, x′′ = d2x(dt)2

y analogamente para y .

b) Estudiar si la curvas α : R→ R2\(0, 0) dada por α(t) = et(cos t, sen t) ,β : R → R2 \ (0, 0) dada por β(t) = (cos t, sen t) y γ(t) = (et, 0) songeodesicas de la variedad.

Solucion: a) La matrız inversa (gij) tiene como coeficientes

g11 = x2 + y2 = g22

g12 = 0 = g21

Notemos que se tiene que

∂g11

∂x= − 2x

(x2 + y2)2=∂g22

∂x∂g11

∂y= − 2y

(x2 + y2)2=∂g22

∂yaplicando las formulas se obtienen

Γ111 =

1

2g11∂g11

∂x= − x

x2 + y2

Γ112 =

1

2g11∂g11

∂y= − y

x2 + y2= Γ1

21

Γ122 = −1

2g11∂g22

∂x=

x

x2 + y2

Γ211 = −1

2g22∂g11

∂y=

y

x2 + y2

Γ212 =

1

2g22∂g22

∂x= − x

x2 + y2= Γ2

21

Γ222 =

1

2g22∂g22

∂y= − y

x2 + y2

Llamando x e y a las coordenadas de los puntos de la curva seobtiene

a =d2x

dt2+

(dx

dt

)2(− x

x2 + y2

)+2

dx

dt

dy

dt

(− y

x2 + y2

)+

(dy

dt

)2(x

x2 + y2

)= 0

Page 147: Apuntes Geom Diferencial

CONEXIONES RIEMANNIANAS 141

b =d2y

dt2+

(dx

dt

)2(y

x2 + y2

)+2

dx

dt

dy

dt

(− x

x2 + y2

)+

(dy

dt

)2(− y

x2 + y2

)= 0

b)Para la curva α tenemosx = et cos t y = et sen tx′ = et cos t− et sen t y′ = et sen t+ et cos tx′′ = −2et sen t y′′ = 2et cos t(x′)2 = e2t(cos t− sen t)2 (y′)2 = e2t(sen t+ cos t)2

x2 + y2 = e2t x′y′ = e2t(cos2 t− sen2 t)De donde se tiene quea = 2et sen t− et cos t(cos2 t+ sen2 t− 2 sen t cos t)−2et sen t(cos2 t− sen2 t) + et cos t(cos2 t+ sen2 t+ 2 sen t cos t)= −2et sen t+ 2et sen3 t+ 2et sen t cos2 t= 2et sen t(−1 + cos2 t+ sen2 t) = 0

b = 2et cos t+ et sen t(cos2 t+ sen2 t− 2 sen t cos t)−2et cos t(cos2 t− sen2 t) + et sen t(cos2 t+ sen2 t+ 2 sen t cos t)= 2et cos t− 2et cos3 t− 2et sen2 t cos t= 2et cos t(1− cos2 t− sen2 t) = 0

Por lo tanto α es una geodesica.Para la curva β tenemosx = cos t y = sen tx′ = − sen t y′ = cos tx′′ = − cos t y′′ = − sen t(x′)2 = sen2 t (y′)2 = cos2 tx2 + y2 = 1 x′y′ = − sen t cos t

De donde se tiene quea = − cos t− cos t sen2 t+ 2 sen2 t cos t+ cos3 t

= cos t(−1 + cos2 t+ sen2t) = 0b = − sen t+ sen3 t+ 2 sen t cos2 t− sen t cos2 t

= sen t(−1 + cos2 t+ sen2 t) = 0Por lo tanto β es tambien una geodesica.Finalmente y de modo casi obvio se comprueba que γ es una geodesi-

ca.

Cartas de Clairaut de una superficie. Sea x una carta deuna superficie con una metrica riemannianna que sera denotada porE = g1 1x

−1 , F = g1 2x−1 = g2 1x

−1 , G = g2 2x−1 . Utilizaremos las

notaciones Γkij = Γkijx−1 para los sımbolos de Christoffel. Denotaremos

las coordenadas por u = pr1x , v = pr2x. Una carta se dira que es deClairaut si se tiene que Eu = Gu = F = 0 .

4.2. Comprobar que para una superficie las geodesicas estan deter-minadas por las dos ecuaciones diferenciales siguientes:

u′′ + Γ111u′2 + 2Γ1

12u′v′ + Γ1

22v′2 = 0

Page 148: Apuntes Geom Diferencial

142 7. VARIEDADES Y CONEXIONES RIEMANNIANAS

v′′ + Γ111u′2 + 2Γ2

12u′v′ + Γ2

22v′2 = 0

4.3. Demostrar que para una carta de Clairaut los sımbolos de ch-ristoffel verifican:

Γ111 = 0, Γ2

11 =−Ev2G

,

Γ112 =

Ev2E

, Γ212 = 0,

Γ122 = 0, Γ2

22 =Gv

2G.

Demostracion: Recordemos que los sımbolos de Christoffel vienendados por

Γmij =1

2

∑k

(∂gjk∂xi

+∂gki∂xj− ∂gij∂xk

)gkm

En este caso se tiene que

g11 =1

Eg12 = g21 = 0 g22 =

1

G

de donde se deduce que:

Γ111 =

1

2

(∂g11

∂u

)g11 =

1

2Eug

11 = 0

Γ211 =

1

2

(− ∂g11

∂v

)g22 =

−Ev2G

Γ112 =

1

2

(∂g11

∂v

)g11 =

Ev2G

Γ212 =

1

2

(∂g22

∂u

)g22 =

1

2Gug

22 = 0

Γ122 =

1

2

(− ∂g22

∂u

)g11 = −1

2Gug

22 = 0

Γ222 =

1

2

(∂g22

∂v

)g22 =

Gv

2G

4.4. Demostrar que para una carta de Clairaut las ecuaciones geodesi-cas se reducen a

u′′ +EvEu′v′ = 0

v′′ − Ev2G

u′2

+Gv

2Gv′

2= 0

Solucion: Basta sustituir los valores encontrados de los sımbolos deChristoffel en la ecuacion de la geodesicas.a

Page 149: Apuntes Geom Diferencial

CONEXIONES RIEMANNIANAS 143

4.5. Sea σ : R→ S una geodesica en una superficie S y supongamosque tenemos una carta de Clairaut, xσ(t) = (u(t), v(t)), para un valordel parametro t supongamos que en σ(t) el angulo que forma el vectortangente y ∂

∂ues θ. Probar que a lo largo de la geodesica se verifica que√

E cos θ = Eu′ =constante.Solucion: Notemos que

√E cos θ = 〈 ∂

∂u, u′

∂u+ v′

∂v〉 = u′E

Por otra parte

d(Eu′)

dt= (Euu

′ + Evv′)u′ + Eu′′ = Eu′′ + Evu

′v′ = 0

4.6. Sea σ : R→ S una curva en una superficie S y supongamos quetenemos una carta de Clairaut, xσ(t) = (u(t), v(t) . Entonces (u(t), v(t)representa una geodesica si y solo si existe una constante c tal que severifican las ecuaciones de primer orden:

u′ =c

E

v′ =

√E − c2

√EG

.

Solucion: La primera se ha obtenido en el ejecicio anterior. Por otrolado sabemos que

1 = 〈σ, σ〉 = E(u′)2 +G(v′)2 =c2

E+G(v′)2

de aquı se obtiene que

v′ =

√E − c2

√EG

Recıprocamente si suponemos que se satisfacen las ecuaciones anterio-res derivando se obtienen las ecuaciones de las geodesicas.

4.7. Sea σ : R → S una curva en una superficie S y supongamosque tenemos una carta de Clairaut, xσ(u) = (u, v(u)) . Entonces, salvoparametrizacion correcta, la curva v = v(u) representa una geodesicasi y solo si existe una constante c para la cual se verifiva la ecuacion:

dv

du=

√E√E − c2

c√G

4.8. Probar que en el semiplano superior con la metrica de PoincareE = G = 1

v2, salvo parametrizaciones, la ecuacion de las geodesicas no

verticales es

dv

du=

√1v2− c2

cCalcular las geodesicas del plano hiperbolico.

Page 150: Apuntes Geom Diferencial

144 7. VARIEDADES Y CONEXIONES RIEMANNIANAS

4.9. Dada una superficie de revolucion

(φ(v) cosu, φ(v) senu, ψ(v))|u, v ∈ RProbar que la metrica viene determinada por E = φ(v)2 , F = 0 ,

G = φ′(v)2 +ψ′(v)2 y que la ecuacion de las geodesicas viene dada por

dv

du=φ√φ2 − c2

c√φ′2 + ψ′2

.

5. Curvatura

Definicion 5.1. El tensor curvatura R de una variedad riemannia-na M es una correspondencia que a cada par de campos X, Y le asociaun operador R(X, Y ) que transforma cada campo Z en un campo de-notado por R(X, Y )Z definido por la expresion:

R(X, Y )Z = OYOXZ − OXOYZ + O[X,Y ]Z

donde O es la conexion riemanniana de M .

Proposicion 5.1. Sean X, Y, Z,W campos tangentes y f, g fun-ciones diferenciables con valores reales, entonces

R(fX + gY, Z) = fR(X,Z) + gR(Y, Z)

R(X, fY + gZ) = fR(X, Y ) + gR(X,Z)

R(X, Y )(fZ + gW ) = fR(X, Y )Z + gR(X, Y )W

Demostracion. Es facil ver que son lineales respecto escalaresreales. Para funciones se tiene:

R(fX, Y )Z = OYOfXZ − OfXOYZ + O[fX,Y ]Z

= OY fOXZ − OfXOYZ + Of [X,Y ]−Y fXZ

= Y fOXZ + fOYOXZ − fOXOYZ + Of [X,Y ]Z + O−Y fXZ

= fOYOXZ − fOYOXZ + fO[X,Y ]Z

= fR(X, Y )Z

Para la variable Y se sigue facilmente R(X, fY ) = −R(fY,X) =−fR(Y,X) = fR(X, Y ) . Para la variable Z se tiene:

R(X, Y )fZ = OYOXfZ − OXOY fZ + O[X,Y ]fZ

Los terminos de la derecha verifican

OYOXfZ = OY (XfZ + fOXZ)

= Y XfZ +XfOYZ + Y fOXZ + fOYOXZ

−OXOY fZ = −OX(Y fZ + fOYZ)

= −XY fZ − Y fOXZ −XfOYZ − fOXOYZO[X,Y ]fZ = [X, Y ]fZ + fO[X,Y ]Z

Sumando se tiene que R(X, Y )fZ = fR(X, Y )Z .

Page 151: Apuntes Geom Diferencial

5. CURVATURA 145

Lema 5.1. Si uno de los campos X, Y, Z se anula en un punto p ,entonces R(X, Y )Z tambien se anula en p .

Demostracion. Como en algunas demostraciones anteriores, delhecho de que R(X, Y )Z sea lineal en cada variable se sigue como esusual que si U es un abierto de la variedad entonces (R(X, Y )Z)|U =R(X|U , Y )Z = R(X, Y |U)Z = R(X, Y )(Z|U) . En el dominio U de una

carta x se tiene que si por ejemplo X|U =∑

i fi

(∂∂xi

), entonces

(R(X, Y )Z)|U =R(X|U , Y )Z = R(∑i

fi

(∂

∂xi

), Y )Z

=∑i

fiR(

(∂

∂xi

), Y )Z .

Si p ∈ U ∩DomX ∩DomY ∩DomZ y Xp = 0 , se tiene que fi(p) = 0para i ∈ 1, · · · ,m . Por lo tanto (R(X, Y )Z)p = ((R(X, Y )Z)|U)p =

(R(X|U , Y )Z)p =∑

i fi(p)(

(R((

∂∂xi

), Y )Z

)p

= 0 .

Como consecuencia del lema anterior si u, v, w son vectores tangen-tes en el punto p , podemos definir el vector tangente R(u, v)w toman-do campos U, V,W tales que Up = u , Vp = v , Wp = w y definiendoR(u, v)w = (R(U, V )W )p .

El tensor curvatura verifica las propiedades siguientes:

Proposicion 5.2. (Identidad de Bianchi) Sean X, Y, Z campostangentes, entonces

R(X, Y )Z +R(Y, Z)X +R(Z,X)Y = 0.

Demostracion. De la definicion del tensor curvatura se tiene que

R(X, Y )Z = OYOXZ − OXOYZ + O[X,Y ]Z

R(Y, Z)X = OZOYX − OYOZX + O[Y,Z]X

R(Z,X)Y = OXOZY − OZOXY + O[Z,X]Y

Sumando se obtiene

R(X, Y )Z +R(Y, Z)X +R(Z,X)Y = OY [X,Z] + OX [Z, Y ] + OZ [Y,X]

− O[Y,X]Z − O[Z,Y ]X − O[X,Z]Y

= [Y [X,Z]] + [X, [Z, Y ]] + [Z[Y,X]]

= 0

Proposicion 5.3. Sean X, Y, Z, T campos tangentes, entonces(i) 〈R(X, Y )Z, T 〉+ 〈R(Y, Z)X,T 〉+ 〈R(Z,X)Y, T 〉 = 0 ,(ii) 〈R(X, Y )Z, T 〉 = −〈R(Y,X)Z, T 〉 ,(iii) 〈R(X, Y )Z, T 〉 = −〈R(X, Y )T, Z〉 ,(iv) 〈R(X, Y )Z, T 〉 = 〈R(Z, T )X, Y 〉 .

Page 152: Apuntes Geom Diferencial

146 7. VARIEDADES Y CONEXIONES RIEMANNIANAS

Demostracion. La propiedad (i) se sigue de la identidad de Bian-chi y la (ii) es consecuencia inmediata de la definicion. La propiedad(iii) es equivalente a ver que 〈R(X, Y )Z,Z〉 = 0 . Entonces

〈R(X, Y )Z,Z〉 = 〈OYOXZ − OXOYZ + O[X,Y ]Z,Z〉= 〈OYOXZ,Z〉 − 〈OXOYZ,Z〉+ 〈O[X,Y ]Z,Z〉= Y 〈OXZ,Z〉 − 〈OXZ,OYZ〉 −X〈OYZ,Z〉

+ 〈OYZ,OXZ〉+1

2[X, Y ]〈Z,Z〉

=1

2Y (X〈Z,Z〉)− 1

2X(Y 〈Z,Z〉) +

1

2[X, Y ]〈Z,Z〉

= −1

2[X, Y ]〈Z,Z〉+

1

2[X, Y ]〈Z,Z〉

= 0

Para probar (iv), tenemos que de la (i) se sigue que

〈R(X, Y )Z, T 〉+ 〈R(Y, Z)X,T 〉+ 〈R(Z,X)Y, T 〉 = 0

〈R(Y, Z)T,X〉+ 〈R(Z, T )Y,X〉+ 〈R(T, Y )Z,X〉 = 0

〈R(Z, T )X, Y 〉+ 〈R(T,X)Z, Y 〉+ 〈R(X,Z)T, Y 〉 = 0

〈R(T,X)Y, Z〉+ 〈R(X, Y )T, Z〉+ 〈R(Y, T )X,Z〉 = 0

Sumando se obtiene que 2〈R(Z,X)Y, T 〉+ 2〈R(Y, T )X,Z〉 = 0 . Por lotanto 〈R(Z,X)Y, T 〉 = 〈R(Y, T )Z,X〉 . De donde se deduce la propie-dad (iv) .

Denotemos por |u|2 = 〈u, u〉 y por |u ∧ v| =√|u|2|v|2 − 〈u, v〉2 .

Proposicion 5.4. Sea σ un subespacio bidimensional real, u, vuna base de σ y supongamos que tenemos un producto interno y unoperador R satisfaciendo las propiedades anteriores, entonces

〈R(u, v)u, v〉|u ∧ v|2

es independiente de la base elegida.

Demostracion. Realicemos un cambio de base u′ = au + bv ,v′ = cu+dv . De los 16 terminos que se generan al aplicar que 〈R(au+bv, cu+ dv)au+ bv, cu+ dv〉 si se tienen en cuenta las propiedades deltensor quedan los cuatro terminos siguientes:

〈R(u′, v′)u′, v′〉 =adad〈R(u, v)u, v〉+ adbc〈R(u, v)v, u〉+ bcad〈R(v, u)u, v〉+ bcbc〈R(v, u)v, u〉

=〈R(u, v)u, v〉(a2d2 − 2adbc+ b2c2)

=(ab− cd)2〈R(v, u)u, v〉

Page 153: Apuntes Geom Diferencial

5. CURVATURA 147

Por otra parte |(au + bv) ∧ (cu + dc)|2 = (ad − bc)2|u ∧ v|2 . Deaquı teniendo en cuenta que en el cambio de base se verifica que ad−bc 6= 0 se tiene que

(5.1)〈R(u′, v′)u′, v′〉|u′ ∧ v′|2

=(ad− bc)2〈R(u, v)u, v〉

(ad− bc)2|u ∧ v|2=〈R(u, v)u, v〉|u ∧ v|2

Definicion 5.2. Sea M una variedad riemanniana y σ un subespa-cio bidimensional de TpM , llamaremos curvatura seccional o curvaturade Riemann de σ a numero real

K(σ) =〈R(u, v)u, v〉|u ∧ v|2

donde u, v es una base del subsepacio σ de TpM .

Dada una carta x de una variedad riemanniana, utilizaremos lassiguientes funciones que determinan el tensor curvatura

R

(∂

∂xi,∂

∂xj

)∂

∂xk=∑l

Rlijk

∂xl

Rijks = 〈R(∂

∂xi,∂

∂xj

)∂

∂xk,∂

∂xs〉

No es difıcil comprobar que

Rlijk =

∑s

ΓsikΓljs −

∑s

ΓsjkΓlis +

∂Γlik∂xj−∂Γljk∂xi

.

Rijks =∑l

Rlijkgls .

Senalaremos tambien que en el caso de 2-variedades riemannianas,en el dominio de una carta x la curvatura seccional se puede calcularcon la formula

K =R1212

g11g22 − g212

.

Es frecuente utilizar ciertas combinaciones de la curvatura seccio-nal. Por ejemplo, si u es un vector unitario u consideramos una baseortonormal u1, · · ·um−1 del hiperplano ortogonal de u en TpM si seconsideran los promedios

Ric(u) =1

m− 1

m−1∑i=1

〈R(u, ui, )u, ui)〉

KE =1

m

m∑j=1

Ric(uj) =1

m(m− 1)

∑i j

R(ui, uj, )ui, uj)〉

Page 154: Apuntes Geom Diferencial

148 7. VARIEDADES Y CONEXIONES RIEMANNIANAS

se obtiene la curvatura de Ricci en la direccion u en el punto p y la lacurvatura escalar en el punto p . Notemos que en principio las curva-turas anteriores no estan bien definidas ya que dependen de como secomplete la base ortonormal. La siguiente proposicion da una definicionintrınseca que prueba que estan bien definidas.

Si V es un espacio vectorial de dimension finita. Una aplicacion li-neal h : V → V tiene asociada el invariante denomiado traza y que de-notaremos por Tr(h) que se define mediante la formula Tr(h) =

∑mi=1 aii

si (aij) es la matriz de h respecto una base v1, · · · , vm del espacio vec-torial.

Sean ahora u, v ∈ TpM de una variedad riemanniana M . Con-sideremos la aplicacion lineal R(u,−)v : TpM → TpM definida porR(u,−)v(w) = R(u,w)v donde R es el tensor curvatura.

Lema 5.2. Sea M una variedad riemanniana con tensor de curva-tura R .

(i) La aplicacion Q : TpM × TpM → R dada por

Q(u, v) = Tr(R(u−)v)

es bilineal y simetrica.(ii) Para u ∈ TpM de norma uno, se tiene que

Ric(u) =1

(m− 1)Q(u, u) .

(iii) Sea E es la aplicacion lineal autoadjunta de la forma bilinealQ ; esto es 〈E(u), v〉 = Q(u, v) . Entonces la curvatura escalarviene dada por

KE = Tr(E) .

Demostracion. El hecho que Q sea bilineal es una comprobacionrutinaria. Para ver que es simetrica sea u1, · · · , un una base ortonormalde TpM , entonces

Q(u, v) =∑i

〈R(u, ui)v, ui〉 =∑i

〈R(v, ui)u, ui〉 = Q(v, u) .

Notemos ademas si u = um se tiene que Q(u, u) = (m− 1) Ric(u) .Por otra parte se tiene que

Tr(E) =∑

i〈E(ui), ui〉 =∑

iQ(ui, ui)

= (m− 1)∑

i Ric(ui) = m(m− 1)KE

Nosotros llamaremos tensor de Ricci a la forma bilineal simetrica Q

y los diremos que las funciones Qi k = Q(

∂∂xi, ∂∂xj

)son los coeficientes

del tensor de Ricci .

Page 155: Apuntes Geom Diferencial

CURVATURA 149

Una carta x de la variedad riemanniana M induce los campos coor-denados usuales. La forma bilineal Q queda deteminada por los valores

Qi k = Q

(∂

∂xi,∂

∂xj

)=∑j

Rji j k

Sea u un vector unitario tangente en el punto p; es decir si u =∑i ai

(∂∂xi

)y∑

ij aigijaj = 1 , entonces la curvatura de Ricci viene

dada por

Ric(u) =1

m− 1

∑i k

aiQi kak

Observemos que puesto que la aplicacion bilineal autoadjunta E : TpM →TpM verifica que Q(u, v) = 〈E(u), v〉 . De donde se obtiene que la cur-vatura escalar viene dada por

KE =1

m(m− 1)

∑i k

Qi kgi k

Problemas

5.1. Considerar en R2 \ (0, 0) la carta inclusion que tiene comocoordenadas x, y . La metrica viene dada por

(gij) =

( 1x2+y2

0

0 1x2+y2

)Los coeficientes de Christoffel vienen dados por

Γ111 = − x

x2 + y2

Γ112 = − y

x2 + y2= Γ1

21

Γ122 =

x

x2 + y2

Γ211 =

y

x2 + y2

Γ212 = − x

x2 + y2= Γ2

21

Γ222 = − y

x2 + y2

Probar que la curvatura seccional es nula.

Solucion:La curvatura seccional en dimension dos viene dada por la formula

K =R1212

g11g22 − g212

=R1212

g11g22

Page 156: Apuntes Geom Diferencial

150 7. VARIEDADES Y CONEXIONES RIEMANNIANAS

Ademas el coeficiente del numerador viene dado por

R1212 = R1121g12 +R2

121g22 = R2121g22

Para calcular la curvatura necesitamos el coeficiente R2121 .

R2121 = Γ1

11Γ221 + Γ2

11Γ222 − (Γ1

21Γ211 + Γ2

21Γ212) +

∂Γ211

∂y− ∂Γ2

21

∂x

Teniendo en cuenta que

Γ111 = Γ2

21 = Γ212,

Γ211 = −Γ2

22 = −Γ112 = Γ2

11 ,

∂Γ211

∂y=∂Γ2

21

∂x

se obtiene R2121 = 0 . De aqui se sigue que K = 0 .

5.2. Sea U = (x, y)|x2 + y2 < 1 el disco unidad y considerarla metrica cuyos coeficientes son g11 = 4

(1−(x2+y2))2, g12 = g21 = 0 y

g22 = 4(1−(x2+y2))2

.

i) Calcular los coeficientes de Christoffel.ii) Encontrar la ecuacion diferencial de las geodesicas.

iii) Calcular la curvatura seccional.

5.3. Sea M el paraboloide de ecuacion z = x2 +y2 y sea j : M → R3

la inclusion canonica.

i) Calcular los coeficientes de la metrica inducida por j en el lavariedad M .

ii) Calcular los coeficientes de Christoffel.iii) Encontrar la ecuacion diferencial de las geodesicas.iv) Calcular la curvatura seccional.

6. Miscelanea

6.1. Considerar la subvariedad E de R2 de ecuacion (xa)2 + (y

a)2 +

( zc)2 = 1 Sea la carta (φ, θ) tal que su dominio es el elipsoide E

menos el subconjunto (x, y, z) ∈ E|y = 0 x ≤ 0 y es tal quex = a cos θ cosφ y = a cos θ senφ z = c sen θ y su codominio es(−π, π)× (−π

2, π

2) .

a) Probar que los coeficientes metricos en la carta anterior son:g11 = a2 cos2 θ, g12 = 0 = g21, g22 = a2 cos2 θ + c2 cos2 θ .

b) Probar que los coeficientes de Christoffel vienen dados por

Γ111 = 0 Γ1

12 = − tan θ = Γ121 Γ1

22 = 0

Γ211 =

a2 sen 2θ

2(a2 sen2 θ + c2 cos2 θ)

Page 157: Apuntes Geom Diferencial

6. MISCELaNEA 151

Γ212 = 0 = Γ2

21

Γ222 =

(a2 − c2) sen 2θ

2(c2 cos2 θ + a2 sen2 θ)

c) Encontrar la ecuacion diferencial de las geodesicas.d) Probar que la curvatura seccional en los puntos de la carta vienedada por

K =c2

(c2 cos2 θ + a2 sen2 θ)2

6.2. Considerar la subvariedad H de R2 de ecuacion x2 + y2− z2 =1 Sea la carta (φ, θ) tal que su dominio es H menos el subconjunto(x, y, z) ∈ H|y = 0 x ≤ 0 y es tal que x = cosh θ cosφ y =cosh θ senφ z = senh θ y su codominio es (−π, π)× (1,+∞) .

a) Probar que los coeficientes metricos en la carta anterior son:g11 = cosh2 θ, g12 = 0 = g21, g22 = cosh2 θ + senh2 θ .

b) Probar que los coeficientes de Christoffel vienen dados por

Γ111 = 0

Γ112 = tanh θ = Γ1

21

Γ122 = 0

Γ211 = −1

2tanh 2θ

Γ212 = 0 = Γ2

21

Γ222 = tanh 2θ

c) Encontrar la ecuacion diferencial de las geodesicas.d) Calcular la curvatura seccional en los puntos de la carta.

Solucion: Se trata de calculos rutinarios. En los apartados a) y b)ya se ha incluido la solucion.

c) Las ecuaciones de las geodesicas es la siguiente

d2φ

dt2+ 2

dt

dttanh θ = 0

d2θ

dt2− 1

2

(dφ

dt

)2

tanh 2θ +

(dθ

dt

)2

tanh 2θ = 0

d)Se obtiene que

R2121 = −1

2(tanh 2θ)2 +

1

2tanh(θ) tanh(2θ)− 1

(cosh 2θ)2= − (cosh θ)2

(cosh 2θ)2

K =R1212

g11g22 − g212

=R2

121g22

g11g22

=R2

121

g11

= − 1

(cosh 2θ)2

Page 158: Apuntes Geom Diferencial

152 7. VARIEDADES Y CONEXIONES RIEMANNIANAS

6.3. Sea la superficie de R3 que tiene ecuacion a ez cos(a x)−cos(a y) =0 . Consideremos la carta (u, v) : M → R2 tal que

Dom(u, v) = (x, y, z)|a ez cos(a x)− cos(a y) = 0, cos a x 6= 0

y tal que u = x, v = ya) Probar que los coeficientes metricos en la carta anterior son lossiguientes:

g11 = 1 + a2 tan2(a u), g12 = −a2(tan a u)(tan a v) = g21, g22 =1 + a2 tan2(a v) .b) Probar que los coeficientes de Christoffel vienen dados por

Γ111 =

a3 sec2(a u) tan(a u)

1 + a2 tan2(a u) + a2 tan2(a v)

Γ112 = 0 = Γ1

21

Γ122 = −

(a3 sec2(a v) tan(a u)

1 + a2 tan2(a u) + a2 tan2(a v)

)Γ2

11 = −(

a3 sec2(a u) tan(a v)

1 + a2 tan2(a u) + a2 tan2(a v)

)Γ2

12 = 0 = Γ221

Γ222 =

a3 sec2(a v) tan(a v)

1 + a2 tan2(a u) + a2 tan2(a v)

c) Encontrar la ecuacion diferencial de las geodesicas.d) Probar que la curvatura seccional en los puntos de la carta vienedada por

K = −

(a4 sec2(a u) sec2(a v)

(1 + a2 tan2(a u) + a2 tan2(a v))2

)Solucion: a) Ya hemos visto en el ejercio anterior que puesto que

x in = u ,y in = v y z in = log cos(a v)a cos(a u)

se tiene que

in∗p

(∂

∂u

)p

=

(∂

∂x

)p

+ (a tan au)p

(∂

∂z

)p

in∗p

(∂

∂v

)p

=

(∂

∂y

)p

− (a tan av)p

(∂

∂z

)p

de donde se obtiene queg11 = 1 + a2 tan2(a u) g12 = g21 = −a2(tan a u)(tan a v) g22 =

1 + a2 tan2(a v)b) La matriz inversa tiene como coeficientes

Page 159: Apuntes Geom Diferencial

6. MISCELaNEA 153

g11 =1 + a2 tan2(a v)

1 + a2 tan2(a u) + a2 tan2(a v)

g12 =a2 tan(a u) tan(a v)

1 + a2 tan2(a u) + a2 tan2(a v)= g21

g22 =1 + a2 tan2(a u)

1 + a2 tan2(a u) + a2 tan2(a v)

Notemos que se tiene que

∂g11

∂u= 2 a3 sec2(a u) tan(a u)

∂g22

∂v= 2 a3 sec2(a u) tan(a v)

∂g11

∂v= 0 =

∂g22

∂u

∂g12

∂u= −

(a3 sec2(a u) tan(a v)

)=∂g21

∂u

∂g12

∂v= −

(a3 sec2(a v) tan(a u)

)=∂g21

∂vaplicando las formulas se obtienen

Γ111 =

a3 sec2(a u) tan(a u)

1 + a2 tan2(a u) + a2 tan2(a v)

Γ112 = 0 = Γ1

21

Γ122 = −

(a3 sec2(a v) tan(a u)

1 + a2 tan2(a u) + a2 tan2(a v)

)Γ2

11 = −(

a3 sec2(a u) tan(a v)

1 + a2 tan2(a u) + a2 tan2(a v)

)Γ2

12 = 0 = Γ221

Γ222 =

a3 sec2(a v) tan(a v)

1 + a2 tan2(a u) + a2 tan2(a v)

c) Llamando u y v a las coordenadas de los puntos de la curva seobtiene

∂2u

∂t2+

((∂u

∂t

)2

sec2(a u)−(∂v

∂t

)2

sec2(a v)

)a3 tan(a u)

1 + a2 tan2(a u) + a2 tan2(a v)= 0

∂2v

∂t2−

((∂u

∂t

)2

sec2(a u) +

(∂v

∂t

)2

sec2(a v)

)a3 tan(a u)

1 + a2 tan2(a u) + a2 tan2(a v)= 0

Page 160: Apuntes Geom Diferencial

154 7. VARIEDADES Y CONEXIONES RIEMANNIANAS

d) Para calcular la curvatura necesitamos los coeficientes R1121 y

R2121 para su computo realizamos previamente los siguientes calculos:

R1121 = Γ2

11Γ122 +

∂Γ111

∂v

R2121 = Γ2

11Γ222 +

∂Γ211

∂v

∂Γ111

∂v=−2 a6 sec2(a u) sec2(a v) tan(a u) tan(a v)

(1 + a2 tan2(a u) + a2 tan2(a v))2

∂Γ211

∂v=

2 a6 sec2(a u) sec2(a v) tan2(a v)

(1 + a2 tan2(a u) + a2 tan2(a v))2−

a4 sec2(a u) sec2(a v)

1 + a2 tan2(a u) + a2 tan2(a v)

de donde se obtiene

R1121 = −

(a6 sec2(a u) sec2(a v) tan(a u) tan(a v)

(1 + a2 tan2(a u) + a2 tan2(a v))2

)

R2121 = −

(a4 sec2(a u) sec2(a v) (1 + a2 tan2(a u))

(1 + a2 tan2(a u) + a2 tan2(a v))2

)De aqui se sigue que

R1212 = −(

a4 sec2(a u) sec2(a v)

1 + a2 tan2(a u) + a2 tan2(a v)

)Finalmente

K = −

(a4 sec2(a u) sec2(a v)

(1 + a2 tan2(a u) + a2 tan2(a v))2

)

Page 161: Apuntes Geom Diferencial

CAPıTULO 8

EL PAQUETE RIEMANNIAN GEOMETRY

1. El paquete RiemannianGeometry

1.1. Introduccion. En este paquete se definen una familia depequenos programas que denominaremos funciones que calculan di-versos coeficientes pseudo-metricos en un abierto coordenado de unavariedad pseudo-riemanniana. La construccion de las funciones y laestructura de las funciones esta basada en el libro “Introducion a laGeometrıa Diferencial”que esta disponible en la pagina web:

http://www.unirioja.es/cu/luhernan/lnes.html

Versiones comprimidas del paquete junto con documentacion y ejem-plos se hallan tambien disponibles en la misma pagina web.

Para instalar el paquete seguir las instrucciones que estan en docu-mento leame.

Este paquete se carga mediante el comando:<<RiemannianGeometry2007`Podemos ver las funciones que contiene, ordenadas alfabeticamente,

mediante:?RiemannianGeometry`*

RiemannianGeometry`Christoffel HiperbolicBallCurvatureTensor InducedPseudoMetricEuclideanMetric InmersionGeometricCoefficients2DGeodesicLines JacobianMGeometricCoefficients MinkowskiPseudometricGeometricCoefficients2D SectionalCurvature2D

Para ver la informacion que existe sobre cada funcion del paquete,se teclea el interrogante de cierre y el nombre de la funcion:?Christoffel

Christoffel[pseudometrica,variables] Calcula los sımbolos deChristoffel de una variedad pseudo-riemanniana a partir de los coe-ficientes de la metrica. Tiene como entrada la matriz m × m de unapseudo-metrica de un abierto coordenado de una variedad M cuyos ele-mentos son funciones que dependen de las variables que aparecen en lasegunda entrada, estas variables denotan las coordenadas de una cartade la variedad M . La salida es un tensor de dimensiones m×m×m ,formado por las funciones denominadas como sımbolos de Christoffel,cada una de estas depende de las mismas variables de las que dependıan

155

Page 162: Apuntes Geom Diferencial

156 8. EL PAQUETE RIEMANNIAN GEOMETRY

los coeficientes pseudo-metricos. Los sımbolos de Chistofell determinanla conexion pseudo-riemannniana en el abierto coordenado, se utilizanpara calcular las geodesicas y la curvatura seccional”

1.2. Algunas pseudometricas mas frecuentes. Sea x unauna carta de una variedad M . Respecto dicha carta la pseudometricaviene determinada por una matriz cuyos elementos son funciones deM en R . Estas funciones se suelen expresar de modo que dependande las coordenadas de la carta elegida x . Veamos algunas de las masfrecuentes:

EuclideanMetric[n] Construye la matriz identidad. Se puede uti-lizar para dar la metrica euclıdia en el espacio usual Rn .

MinkowskiPseudometric[n] Construye la matriz diagonal contodo unos salvo el lugar (n, n) que es menos uno. En particular tenemosla pseudo-metrica de Minkowski en Rn .

HiperbolicBall[n, variables] Calcula la metrica riemanniana hi-perbolica en la bola unidad. Tiene como entrada la dimension del es-pacio y las variables que se deseen utilizar (exactamente n variables),la salida es la metrica hiperbolica en la bola unidad. En la esfera uni-dad no hay definada ninguna matriz, fuera del disco unidad apareceinducida una pseudo-metrica.EuclideanMetric[3]1, 0, 0, 0, 1, 0, 0, 0, 1

MinkowskiPseudometric[4]1, 0, 0, 0, 0, 1, 0, 0, 0, 0, 1, 0, 0, 0, 0,−1

HiperbolicBall[2, u,v]4

(1−u2−v2)2, 0,

0, 4(1−u2−v2)2

1.3. Pseudo-metrica inducida por una inmersion. Dada una

inmersion f : M → N , en la que se supone que disponemos de una car-ta x en la variedad M y tambien de una carta y en la variedad N .Supondremos que la variedad N es pseudo-riemanniana y que cono-cemos la matriz de la pseudo-metrica respecto de la carta y, cuyoscoeficientes dependen de las coordenadas de la carta y. En una propo-sicion del capıtulo “Variedades y Conexiones Riemannianas”se pruebaque bajo estas condiciones queda inducida una nueva pseudo-metri-ca en la variedad M . Con los siguientes miniprogramas se calcula lapseudo-metrica inducida en la variedad M . Se puede utlilizar para es-tudiar las pseudo-metricas inducidas en subvariedades de los espacioseuclıdeos, hiperbolicos o de Minkowski. Tambien puede ser utilizadocon homeomorfismos locales y aplicaciones recubridoras.

JacobianM[componentes,variables] Obtiene la matriz jacobia-na de una funcion dependiente de varias variables y que tiene valoresvectoriales. Tiene como entrada componentes que es una lista de fun-ciones que dependen de las variables que estan en la segunda entrada.

Page 163: Apuntes Geom Diferencial

1. EL PAQUETE RIEMANNIANGEOMETRY 157

La salida es la matriz jacobiana obtenida al calcular las derivadas par-ciales de las componentes repecto de las variables. Las filas contienenlas derivadas parciales respecto las diferentes variables.

InducedPseudoMetric[componentes, pseudometrica, variables1,variables2] Calcula la pseudo-metrica inducida por una inmer-sion. Dada una funcion de una variedad M en otra N y supongamosque respecto dos cartas de M y N la funcion viene dada por las funcio-nes que forman la lista componentes que es la primera entrada, estasfunciones dependen de las variables1 que forman la tercera entrada yson las coordenadas de la carta que tenemos en la variedad M . La en-trada pseudometrica es la pseudo-metrica de la variedad N en terminosde la carta considerada en la variedad N , cada coeficiente es una expre-sion que depende de variables2 que es la ultima entrada. Tiene comosalida la pseudo-metrica inducida en la variedad M que dependera delas coordenadas de M contenidas en la lista variables1.

paraboloide= u, v, u2 + v2 ;ParametricPlot3D[paraboloide,u,-1,1,v,-1,1]

-1-0.5 00.51

-1-0.5

00.51

00.51

1.52

-1-0.5 00.51

-1-0.5

00.51

JacobianM[paraboloide, u,v]1, 0, 0, 1, 2u, 2 vInducedPseudoMetric[paraboloide, EuclideanMetric[3],u,v,x,y,z]

1 + 4u2, 4u v,

4u v, 1 + 4 v2

1.4. Sımbolos de Christoffel y ecuaciones de las geodesi-cas. Dada una variedad M y una carta x las dos funciones siguientescalculan los sımbolos de Christoffel que dependen de las coordenadasde la carta. A partir de los sımbolos de Christoffel tambien se puedecalcular el sistema de ecuaciones diferenciales de las geodesicas de lavariedad M . El programa no integra estas ecuaciones por lo que nose calculan explıcitamente las geodesicas. No obstante se pueden uti-lizar las funciones basicas de Mathematica para intentar encontrar lassoluciones de dicho sistema de ecuaciones diferenciales.

Christoffel[pseudometrica,variables] Calcula los sımbolos deChristoffel de una variedad pseudo-riemanniana a partir de los coe-ficientes de la pseudo-metrica. Tiene como entrada la matriz m×m deuna pseudo-metrica de un abierto coordenado de una variedad M cuyoselementos son funciones que dependen de las variables que aparecen en

Page 164: Apuntes Geom Diferencial

158 8. EL PAQUETE RIEMANNIAN GEOMETRY

la segunda entrada, estas variables denotan las coordenadas de una car-ta de la variedad M . La salida es un tensor de dimensiones m×m×m ,formado por las funciones denominadas como sımbolos de Christoffel,cada una de estas depende de las mismas variables de las que dependıanlos coeficientes pseudo-metricos. Los sımbolos de Chistofell determinanla conexion pseudo-riemannniana en el abierto coordenado, se utilizanpara calcular las geodesicas y la curvatura seccional.

GeodesicLines[chris,variables,nombretiempo] Obtiene el sis-tema de ecuaciones diferenciales de las geodesicas de una variedadpseudo-riemanniana. Tiene como entrada los sımbolos de Christoffel,las variables de las que dependen y nombretiempo que es el nombre dela variable ‘tiempo’. La salida es una lista que contiene el sistema deecuaciones diferenciales de las geodesicas de la variedad en la carta.paraboloidepsm = 1 + 4u2, 4uv, 4uv, 1 + 4v2 ;Christoffel[paraboloidepsm,u,v]

4 u1+4 u2+4 v2 , 0

,

0, 4 u1+4 u2+4 v2

,

4 v1+4 u2+4 v2 , 0

,

0, 4 v1+4 u2+4 v2

paraboloidechris=

4 u1+4 u2+4 v2 , 0

,

0, 4 u1+4 u2+4 v2

,

41+4 u2+4 v2 , 0

,

0, 4 v1+4 u2+4 v2

;

GeodesicLines[paraboloidechris,u,v,t]4 u u′[t]2

1+4 u2+4 v2 + 4 u v′[t]2

1+4 u2+4 v2 + u′′[t], 4 v u′[t]2

1+4 u2+4 v2 + 4 v v′[t]2

1+4 u2+4 v2 + v′′[t]

1.5. Tensor de curvatura y Curvatura Seccional. Dada unavariedad M y una carta x las dos funciones siguientes calculan loscoeficientes del tensor de curvatura que dependen de las coordenadasde la carta y para el caso de variedades de dimension dos obtiene lacurvatura seccional.

CurvatureTensor[simboloschristoffel, pseudometrica, variables] Calcula todos los coeficientes del tensor de curvatura de unavariedad riemanniana a partir de los sımbolos de Christoffel y de loscoeficientes de la pseudo-metrica. Tiene como como primera entradael tensor de los sımbolos de Christoffel que seguramente se habra cal-culado con la funcion del paquete Christoffel, estos sımbolos dependende las variables que aparecen en la tercera entrada, la segunda entradaes la matriz m ×m de una pseudo-metrica de un abierto coordenadode una variedad M cuyos elementos tambien son funciones que depen-den de las variables que aparecen en la tercera y ultima entrada, estasvariables denotan las coordenadas de una carta de la variedad M . Lasalida es un tensor de dimensiones m×m×m×m , formado por lasfunciones del tensor de curvatura, cada una de estas depende de lasmismas variables de las que dependıan los coeficientes pseudometricos.El tensor de curvatura se utiliza para calcular la curvatura seccional yotras curvaturas.

SectionalCurvature2D[tensordecurvatura, pseudometrica]Calcula la curvatura seccional, que coincide con la de Gauss, a par-tir de los coeficientes del tensor de curvatura y de la metrica. Tiene

Page 165: Apuntes Geom Diferencial

1. EL PAQUETE RIEMANNIANGEOMETRY 159

como como primera entrada el tensor de curvatura 2x2x2x2 que segu-ramente se habra calculado con la funcion CurvatureTensor del paqueteRiemannianGeometry, como segunda entrada la matriz 2x2 de la pseu-dometrica de un abierto coordenado de la superficie M . La salida esun una funcion que da la curvatura seccional de la superfice que es unavariedad de dimension dos.

CurvatureTensor[paraboloidechris, paraboloidepsm,u,v]0, 0, 0, 0,

0, 4

1+4u2+4 v2

,− 4

1+4u2+4 v2, 0

,0,− 4

1+4u2+4 v2

,

41+4u2+4 v2

, 0

, 0, 0, 0, 0

paraboloidecur=0, 0, 0, 0,

0, 4

1+4u2+4 v2

,− 4

1+4u2+4 v2, 0

,0,− 4

1+4u2+4 v2

,

41+4u2+4 v2

, 0

, 0, 0, 0, 0

;

SectionalCurvature2D[paraboloidecur, paraboloidepsm]4

(1+4u2+4 v2)2

1.6. Coeficientes geometricos inducidos por una inmer-sion. El miniprograma siguiente calcula la pseudo-metrica, los sımbo-los de Christoffel, las ecuaciones diferenciales de las geodesicas, los coe-ficientes del tensor de curvatura y la curvatura seccional de la geometrıainducida en una superficie por una inmersion de esta en una varie-dad pseudo-riemanniana. El siguiente miniprograma para una superfi-cie pseudo-riemanniana calcula los sımbolos de Christoffel, el sistemade ecuaciones diferenciales de las geodesicas, los coeficientes del tensorde curvatura y la curvatura seccional. El tercer programa para un va-riedad pseudo-riemanniana calcula todos los elementos anteriores salvola curvatura seccional.

InmersionGeometricCoefficients2D[componentes, inductora, variables1, variables2, nombretiempo] Dada una inmersion deuna variedad M en una variedad N que tiene una pseudo-metrica cono-cida esta funcion calcula la pseudo-metrica inducida en la variedad My tambien obtiene los sımbolos de Christoffel, el sistema de ecuacionesdiferenciales de las geodesicas, los coeficientes del tensor de curvaturay la curvatura seccional la variedad M . Tiene como primera entradacomponentes que son n funciones con valores reales que dependen delas m variables que componen, variables1, que son precisamente lasexplicitadas en la tercera entrada. Estas funciones se supone que sehan obtenido al considerar una inmersion f : M → N entre varieda-des y tomar sistemas coordenados x e y respectivamente, se tiene quecomponentes= yfx−1 . La segunda entrada es la matriz de la pseudo-metrica de la variedad N respecto la carta y , los coeficientes pseudo-metricos dependen de la cuarta entrada, variables2. Las mas usual enlos ejemplos que acompanan este paquete es EuclideanMetric[3], que

Page 166: Apuntes Geom Diferencial

160 8. EL PAQUETE RIEMANNIAN GEOMETRY

es la matriz identidad de dimension 3x3. La variable nombretiempo,que es el nombre que asignamos al ‘tiempo’ en el sistema de ecuacionesdiferenciales que verifican las geodesicas. La salida consiste en variaslıneas que van explicando los resultados que se van obteniendo. Paraobtener una lista de los resultados anteriores basta introducir comoimput %. Tomando elementos de esta se pueden extraer cada resultadoque interese para poderlo ulilizar de nuevo.

GeometricCoefficients2D[pseudometrica,variables,nombretiempo] Obtiene los sımbolos de Christoffel, el sistema de ecuacionesdiferenciales de las geodesicas, los coeficientes del tensor de curvaturade una pseudo- metrica riemanniana y la curvatura seccional de unapseudo-metrica riemanniana. Tiene como primera entrada pseudome-trica, que es la matriz de la pseudo-metrica de la variedad M respectola carta x, los coeficientes pseudo-metricos dependen de la segunda en-trada, variables. La tercera entrada, nombretiempo, que es el nombreque asignamos al ‘tiempo’ en el sistema de ecuaciones diferenciales queverifican las geodesicas. La salida consiste en varias lıneas que van ex-plicando los resultados que se van obteniendo. Para obtener una listade los resultados anteriores basta introducir como imput %. Tomandoelementos de esta se pueden extraer cada resultado que interese parapoderlo ulilizar de nuevo.

GeometricCoefficients[pseudometrica,variables,nombretiempo]Obtiene los sımbolos de Christoffel, el sistema de ecuaciones dife-renciales de las geodesicas y los coeficientes del tensor de curvaturade una pseudo-metrica riemanniana. Tiene como primera entrada pe-sudometrica, que es la matriz de la pseudo-metrica de la variedad Mrespecto la carta x, los coeficientes pseudo-metricos dependen de la en-trada variables. La tercera entrada, nombretiempo, que es el nombreque asignamos al ‘tiempo’ en el sistema de ecuaciones diferenciales queverifican las geodesicas. La salida consiste en varias lıneas que van ex-plicando los resultados que se van obteniendo. Para obtener una listade los resultados anteriores basta introducir como imput %. Tomandoelementos de esta se pueden extraer cada resultado que interese parapoderlo ulilizar de nuevo. A diferencia de la funcion GeometricCoeffi-cients2D funciona en cuarquier dimension, pero no calcula la curvaturaseccional.

InmersionGeometricCoefficients2D[paraboloide,Euclidean-Metric[3],u,v,x,y,z,t]

La matriz de la pseudometrica es la siguiente:= 1 + 4u2, 4uv, 4uv, 1 + 4v2 ;Los sımbolos de Christofell son los siguientes:

4 u1+4 u2+4 v2 , 0

,

0, 4 u1+4 u2+4 v2

,

4 v1+4 u2+4 v2 , 0

,

0, 4 v1+4 u2+4 v2

Las ecuaciones diferenciales de las geodesicas son:

Page 167: Apuntes Geom Diferencial

1. EL PAQUETE RIEMANNIANGEOMETRY 1614 u u′[t]2

1+4 u2+4 v2 + 4 u v′[t]2

1+4 u2+4 v2 + u′′[t], 4 v u′[t]2

1+4 u2+4 v2 + 4 v v′[t]2

1+4 u2+4 v2 + v′′[t]

Los coeficientes del tensor de curvatura son:0, 0, 0, 0,

0, 4

1+4u2+4 v2

,− 4

1+4u2+4 v2, 0

,0,− 4

1+4u2+4 v2

,

41+4u2+4 v2

, 0

, 0, 0, 0, 0

La curvatura seccional viene dada por4

(1+4u2+4 v2)2

Para obtener los resultados anteriores introduce despues de la eje-cucion ‘ %’ como un input

1.7. Ejemplos. La superficies inmersas en el 3-espacio euclıdioheredan una metrica riemanniana. Si disponemos de una carta de la su-perficie y las funciones que nos dan las coordenadas cartesianas euclıdiasen funcion de las de la superficie, entonces podemos calcular los diversoscoeficientes geometricos.hiperboloide =Cosh[θ ] Cos[ϕ], Cosh[θ] Sin[ϕ], Sinh[θ] ;ParametricPlot3D[hiperboloide,ϕ,-4,4,θ,-1,1]

-10

1

-101

-1-0.5

00.51

-10

1

-101

InmersionGeometricCoefficients2D[hiperboloide, EuclideanMetric[3],ϕ,θ,x,y, z,t]

La matriz de la pseudometrica es la siguiente:cosh(θ)2, 0, 0, cosh(2 θ)Los sımbolos de Christofell son los siguientes:

0, tanh(θ), tanh(θ), 0, − tanh(2 θ)2

, 0, 0, tanh(2 θ)Las ecuaciones diferenciales de las geodesicas son:

2 tanh(θ) θ′(t)ϕ′(t) + ϕ′′(t), tanh(2 θ) θ′(t)2 − tanh(2 θ)ϕ′(t)2

2+ θ′′(t)

Los coeficientes del tensor de curvatura son:0, 0, 0, 0, 0,−

(cosh(θ)2 Sech(2 θ)

), cosh(θ)2 Sech(2 θ), 0,

0, cosh(θ)2 Sech(2 θ), −(cosh(θ)2 Sech(2 θ)

), 0, 0, 0, 0, 0

La curvatura seccional viene dada por−Sech(2 θ)2

Para obtener los resultados anteriores introduce despues de la eje-cucion ‘ %’ como un imput

elipsoide=a Cos[θ] Cos[ϕ], a Cos[θ] Sin[ϕ], c Sin[θ] ;eli=elipsoide//.a->1,c->2 ;ParametricPlot3D[eli,ϕ,-2 Pi,2 Pi,θ,-Pi/2,Pi/2]

Page 168: Apuntes Geom Diferencial

162 8. EL PAQUETE RIEMANNIAN GEOMETRY

-1-0.5

00.5

1

-1

-0.500.5

1

-2

-1

0

1

2-1

-0.500.5

InmersionGeometricCoefficients2D[elipsoide, EuclideanMetric[3],ϕ,θ,x,y,z,t]

La matriz de la pseudometrica es la siguiente:

a2+c2+(−a2+c2) cos(2 θ)

2, 0, 0, a2 cos(θ)2

Los sımbolos de Christofell son los siguientes:

−(

(−a2+c2) sin(2 θ)

a2+c2+(−a2+c2) cos(2 θ)

), 0, 0, 2 a2 cos(θ) sin(θ)

a2+c2+(−a2+c2) cos(2 θ),

0,− tan(θ), − tan(θ), 0Las ecuaciones diferenciales de las geodesicas son:

−(

(−a2+c2) sin(2 θ) θ′(t)2

a2+c2+(−a2+c2) cos(2 θ)

)+ 2 a2 cos(θ) sin(θ)ϕ′(t)2

a2+c2+(−a2+c2) cos(2 θ)+ θ′′(t),

− 2 tan(θ) θ′(t)ϕ′(t) + ϕ′′(t)Los coeficientes del tensor de curvatura son:0, 0, 0, 0, 0, 2 a2 c2 cos(θ)2

a2+c2+(−a2+c2) cos(2 θ),

−2 a2 c2 cos(θ)2

a2+c2+(−a2+c2) cos(2 θ), 0, 0, −2 a2 c2 cos(θ)2

a2+c2+(−a2+c2) cos(2 θ),

2 a2 c2 cos(θ)2

a2+c2+(−a2+c2) cos(2 θ), 0, 0, 0, 0, 0

La curvatura seccional viene dada por4 c2

(a2+c2+(−a2+c2) cos(2 θ))2

Para obtener los resultados anteriores introduce despues de la eje-cucion ‘ %’ como un imput

Page 169: Apuntes Geom Diferencial

1. EL PAQUETE RIEMANNIANGEOMETRY 163

Una variedad pseudo-riemanniana viene determinada en cada cartapor una matriz simetrica que depende de las coordenadas. A partirde sus coeficientes se pueden calcular, los sımbolos de Chrsitoffel, lasgeodesicas, el tensor de curvatura y en el caso de dimension dos lacurvatura seccional. Estudiamos a continuacion las bolas hiperbolicasde dimensiones tres y dos.GeometricCoefficients[HiperbolicBall[3,u,v,w],u,v,w,t]

La matriz de la pseudometrica es la siguiente: 9

(1−u2−v2−w2)2, 0, 0, 0, 9

(1−u2−v2−w2)2, 0, 0, 0, 9

(1−u2−v2−w2)2

Los sımbolos de Christofell son los siguientes: −2u

−1+u2+v2+w2 ,−2 v

−1+u2+v2+w2 ,−2w

−1+u2+v2+w2, −2 v−1+u2+v2+w2 ,

2u−1+u2+v2+w2 , 0,

−2w−1+u2+v2+w2 , 0,

2u−1+u2+v2+w2,

2 v−1+u2+v2+w2 ,

−2u−1+u2+v2+w2 , 0,

−2u−1+u2+v2+w2 ,

−2 v−1+u2+v2+w2 ,

−2w−1+u2+v2+w2,

0, −2w−1+u2+v2+w2 ,

2 v−1+u2+v2+w2,

2w−1+u2+v2+w2 , 0,

−2u−1+u2+v2+w2,

0, 2w−1+u2+v2+w2 ,

−2 v−1+u2+v2+w2,

−2u−1+u2+v2+w2 ,

−2 v−1+u2+v2+w2 ,

−2w−1+u2+v2+w2

Las ecuaciones diferenciales de las geodesicas son:

−2uu′(t)2

−1+u2+v2+w2− 4 v u′(t) v′(t)−1+u2+v2+w2 + 2u v′(t)2

−1+u2+v2+w2− 4wu′(t)w′(t)−1+u2+v2+w2 + 2uw′(t)2

−1+u2+v2+w2 +u′′(t),

2 v u′(t)2

−1+u2+v2+w2− 4uu′(t) v′(t)−1+u2+v2+w2− 2 v v′(t)2

−1+u2+v2+w2− 4w v′(t)w′(t)−1+u2+v2+w2 + 2 v w′(t)2

−1+u2+v2+w2 +v′′(t),

2wu′(t)2

−1+u2+v2+w2 + 2w v′(t)2

−1+u2+v2+w2− 4uu′(t)w′(t)−1+u2+v2+w2− 4 v v′(t)w′(t)

−1+u2+v2+w2− 2ww′(t)2

−1+u2+v2+w2 +w′′(t)

Los coeficientes del tensor de curvatura son:0, 0, 0, 0, 0, 0, 0, 0, 0, 0, −36

(−1+u2+v2+w2)4, 0,

36(−1+u2+v2+w2)4

, 0, 0, 0, 0, 0, 0, 0, −36(−1+u2+v2+w2)4

, 0, 0, 0, 36

(−1+u2+v2+w2)4, 0, 0,

0, 36(−1+u2+v2+w2)4

, 0, −36(−1+u2+v2+w2)4

, 0, 0, 0, 0, 0,0, 0, 0, 0, 0, 0, 0, 0, 0,0, 0, 0, 0, 0, −36

(−1+u2+v2+w2)4, 0, 36

(−1+u2+v2+w2)4, 0,

0, 0, 36(−1+u2+v2+w2)4

, 0, 0, 0, −36(−1+u2+v2+w2)4

, 0, 0,0, 0, 0, 0, 0, 36

(−1+u2+v2+w2)4, 0, −36

(−1+u2+v2+w2)4, 0,

0, 0, 0, 0, 0, 0, 0, 0, 0Para obtener los resultados anteriores introduce despues de la eje-

cucion ‘ %’ como un imput

Page 170: Apuntes Geom Diferencial

164 8. EL PAQUETE RIEMANNIAN GEOMETRY

2. Integracion numerica de las geodesicas

El paquete dispone de dos funciones denominadas NGeodesicLines e InmersionNGeodesicLines3D. La primera de ellas se puedeutilizar para cualquier inmersion y la salida es una lista de pares quecontienen las coordenadas de los puntos y de las velocidades de unageodesica. La segunda se utiliza solamente para inmersiones en R3 ysu salida es de tipo grafico; dibuja una geodesica en una superficiedando su punto y velocidad iniciales. Describimos a continuacion estasfunciones y despues daremos algunos ejemplos.

NGeodesicLines[chris,variables,puntoinicial,velocidadinicial, incremento,iteraciones] tiene como entrada los coeficientes deChristoffel, las variables de las que dependen, las coordenadas del puntoinicial, las coordenadas de la velocidad inicial, el incremento de tiempoque se va a utilizar para saltar de un punto al siguiente de la geodesicay el numero de iteraciones que vamos a realizar para encontrar maso menos coordenadas de puntos de la geodesica. La salida consiste enun lista de pares, los primeros elementos del par son coordenadas depuntos de la geodesica y los segundos elementos las coordenadas delvector velocidad en ese punto.

InmersionNGeodesicLines3D[componentes,inductora,variables1, variables2,nombretiempo,puntoinicial,velocidadinicial,incremento, iteraciones,rango1,rango2] tiene como primera en-trada componentes que son n funciones con valores reales que dependende las m variables que componen, variables1, que son precisamente lasexplicitadas en la tercera entrada. Estas funciones se supone que sehan obtenido al considerar una inmersion f : M → N entre varieda-des y tomar sistemas coordenados x e y respectivamente, se tiene quecomponentes = yfx−1. La segunda entrada es la matriz de la pseudo-metrica de la variedad N respecto la carta y, los coeficientes pseudo-metricos dependen de la cuarta entrada variables2. La mas usual enlos ejemplos que acompanan este paquete es EuclideanMetric[3], quees la matriz identidad de dimension 3x3. La variable nombretiempo esel nombre que asignamos al ‘tiempo’ en el sistema de ecuaciones dife-renciales que verifican las geodesicas. Las siguientes variables son lascoordenadas del punto inicial, las coordenadas de la velocidad inicial,el incremento de tiempo que se va a utilizar para saltar de un punto alsiguiente de la geodesica y el numero de iteraciones que vamos a reali-zar para encontrar mas o menos coordenadas de puntos de la geodesi-ca. La variables, rango1, rango2, son de la forma variables1[[1]],a,b,variables1[[2]],c,d y determinan los rangos para dibujar en parametri-cas la superficie dada por la inmersion definida en la variable com-ponentes. En variables1[[1]] se repite la primera de las variables quecomponen variables1 y analogamente para variables1[[2]] .

Page 171: Apuntes Geom Diferencial

2. INTEGRACIoN NUMeRICA DE LAS GEODeSICAS 165

El el siguiente ejemplo tenemos una geodesica en el paraboloide. Seobserva que que al menos se corta a si misma una vez.

InmersionNGeodesicLines3D@paraboloide, EuclideanMetric@3D,8u, v<, 8x, y, z<, t, 83, 3<, 8-1, -2<, 0.05, 400, 8u, -3, 3<, 8v, -3, 3<D

Las ecuaciones diferenciales de las geodésicas son:

:4 u u¢@tD2

1 + 4 u2 + 4 v2+

4 u v¢@tD2

1 + 4 u2 + 4 v2+ u¢¢@tD,

4 v u¢@tD2

1 + 4 u2 + 4 v2+

4 v v¢@tD2

1 + 4 u2 + 4 v2+ v¢¢@tD>

Geodésica con el punto y la velocidad inicial introducidos:

-2

0

2

-2

0

2

0

5

10

15

Page 172: Apuntes Geom Diferencial

166 8. EL PAQUETE RIEMANNIAN GEOMETRY

En este caso se considera una geodesica en el hiperboloide.

hiperboloide = 8Cosh@ΘD Cos@jD, Cosh@ΘD Sin@jD, Sinh@ΘD<8Cos@jD Cosh@ΘD, Cosh@ΘD Sin@jD,Sinh@ΘD<InmersionNGeodesicLines3D@hiperboloide, EuclideanMetric@3D,8j, Θ<, 8x, y, z<, t, 81,3<, 80.1, -0.1<, 0.2, 1000, 8j, 0, 2 Pi<, 8Θ, -3, 3<D

8Cos@jD Cosh@ΘD, Cosh@ΘD Sin@jD, Sinh@ΘD<

8Cos@jD Cosh@ΘD, Cosh@ΘD Sin@jD, Sinh@ΘD<

Las ecuaciones diferenciales de las geodésicas son:

:2 Tanh@ΘD Θ¢@tD j

¢@tD + j¢¢@tD, Tanh@2 ΘD Θ

¢@tD2-1

2Tanh@2 ΘD j

¢@tD2+ Θ

¢¢@tD>

Geodésica con el punto y la velocidad inicial introducidos:

-10

-5

0

5

10

-10

-5

0

5

10

-10

-5

0

5

10

3. Algunas aplicaciones informaticas para la geometrıa

Algunas de las aplicaciones de los siguientes enlaces se adaptan bien a loscontenidos de este curso. Tengase en cuenta que los muchos enlaces funcionandurante un tiempo limitado. No obstante en el caso que no funcionen realizandoalguna busqueda en la red se suelen encontrar los nuevos lugares donde han sidoalojadas las paginas o aplicaciones que se mencionan a continuacion.

Page 173: Apuntes Geom Diferencial

4. OTROS ENLACES INTERESANTES 167

Enlace 3.1. Dibuja superficies en implıcitas y parametricas. Enellas se puede considerar la metrica inducida por el ambiente que pue-de ser euclıdio o minkowskiano. Tambien trabaja con metricas pseu-doriemannianas en un abierto del plano. Calcula la curvatura en cadapunto, dibuja geodesicas, realiza transporte paralelo. En mi opinion esuna herramiente docente motivadora e interesante.

http://www.uv.es/~montesin/

En el momento de escribir estas notas el programa anterior no seha actualizado para que pueda funcionar con el sistema X (10.x) delMacOS.

Enlace 3.2. Aquı se puede localizar la aplicacion 3D-XplorMath(solo para Macintosh) que es una herramienta para la visulizacion deobjetos matematicos, que incluyes superficies, curvas y poliedros. Tam-bien contiene otra version de la aplicacion 3D-XplorMath-J que requie-re Java y que funciona en todos los sistemas.

http://3d-xplormath.org/index.html

Enlace 3.3. Contiene informacion sobre un libro de A. Gray queademas geometrıa diferencial ensena a utilizar Mathematica para re-presentar graficamente curvas y superficies, calcular curvaturas, y otroscalculos de interes en geometrıa diferencial.

http://alpha01.dm.unito.it/personalpages/abbena/gray/

Material adicional se puede encontrar en la pagina web dedicada ala memoria de Alfred Gray

http://www.math.umd.edu/research/bianchi/

Enlace 3.4. Visualization ToolKit (VTK) es un codigo fuenteabierto, se trata de software libre para graficas de ordenador en 3D,procesamiento de imagen y visualizacion, utilizado por miles de inves-tigadores en el mundo. VTK consiste en una librerıa de clase C++ y va-rios modos de interfaces que incluyen Tcl/Tk, Java, and Python. VTKdispone de una amplia variedad de algoritmos que incluyen metodosescalares, vectoriales, tensoriales y volumetricos. Tiene tecnicas avan-zadas y numerosos algoritmos para mezclar imagenes 2D y 3D. Ha sidoinstalado y comprobado en plataformas basadas en Unix, Windows 98y Mac OSX.

http://public.kitware.com/VTK/

4. Otros enlaces interesantes

Enlace 4.1. La pagina web personal del autor de estas notas con-tiene materiales didacticos relacionados con geometrıa diferencial y elgrupo fundamental, ası como otros trabajos de divulgacion sobre po-liedros, nudos, ...

http://www.unirioja.es/cu/luhernan/

Page 174: Apuntes Geom Diferencial

168 8. EL PAQUETE RIEMANNIAN GEOMETRY

Enlace 4.2. Tiene enlaces a varias paginas web relacionadas conGeometrıa Diferencial Elemental

http://www.math.wayne.edu/~drucker/diffgeomrefsF07.html

Enlace 4.3. Se trata de un estudio de visualizacion de sistemasdinamicos realizado en la tesis de Helwig Loffelmann “Visualizing Lo-cal Properties and Characteristic Structures of Dynamical Systems”

http://www.cg.tuwien.ac.at/~helwig/diss/diss.htm

Enlace 4.4. Contiene publicaciones on line, videos, imagenes desuperficies, etc

http://www-sfb288.math.tu-berlin.de/

Page 175: Apuntes Geom Diferencial

Indice alfabetico

i-esima coordenada de la carta, 231-forma, 101

abiertos, 10accion de un grupo, 83accion discontinua, 83accion eficiente, 83accion libre, 83algebra de Lie, 103aplicacion exponencial, 128aplicacion lineal, 52atlas compatibles, 23atlas diferenciable m-dimensional, 23

base de entornos, 11base de un espacio vectorial, 15base de una topologıa, 11

campo completo, 115campo de vectores, 101campo paralelo, 127campo tangente a una variedad a lo

largo de una curva, 124campos linealmente independientes,

109carta m-dimensional, 23cerrado, 11clausura, 11cociente, 13codominio de una funcion, 9coeficientes de la metrica, 131coeficientes del tensor de Ricci, 148cohomologıa de DE Rham, 106comosicion de funciones, 9conexion compatible con la metrica,

136conexion lineal en un punto, 123conexion lineal en una variedad, 123conexion riemmanniana, 136conexion simetrica, 136conexo, 12

conexo por caminos, 12conjunto final, 9conjunto imagen, 9conjunto inicial, 9conjunto subyacente de la variedad,

24continua, 10coordenadas, 15corchete de Lie, 103curva integral, 114curva integral completa, 115curvatura de Ricci, 148curvatura escalar, 148curvatura seccional o de Riemann,

147

derivable en r0 con respecto a lai-esima variable, 19

derivable en r0 en la direccion de u,18

derivable en un punto, 17derivable, 17derivacion o vector tangente en un

punto, 52derivada covariante, 125derivada de una funcion, 17derivada direccional de f en la

direccion u en el punto r0, 18derivada parcial de f con respecto la

i-esima coordenada, 51difeomorfa, 25difeomorfismo, 25difeomorfismo local, 57difeomorfismo local en el punto, 57diferencial de una forma, 105dominio, 9dominio fundamental, 85dominio fundamental normal, 85

encaje, 61encaje regular, 61

169

Page 176: Apuntes Geom Diferencial

170 INDICE ALFABETICO

entorno de un punto, 11epimorfismo, 16espacio de Minkovski, 131espacio hiperbolico, 131espacio metrico, 14espacio tangente de la variedad en el

punto, 53espacio topologico, 10espacio vectorial real, 15estructura diferenciable, 24

fibra de una funcion, 9fibrado cotangente, 100fibrado tangente, 99forma cerra, 106forma exacta, 106funcon de clase Ck, 20funcion analıtica en un punto, 20funcion analıtica real, 20funcion de clase Ck en un punto, 19funcion de clase C∞, 20funcion de clase C∞ en un punto, 20funcion diferenciable, 25funcion diferenciable en un punto, 25funcion inyectiva, 9funcion sobre, 10funcion suprayectiva, 9funcion, 9funcion continua, 10

geodesica, 128germen, 52grupo de Lie, 93

homeomorfos, 10

imagen de una funcion, 9imagen inversa de un subconjunto, 9inmersion, 61inmersion en un punto, 61inmersion glogal, 61inmersion isometrica, 130interior, 11isometrica, 130isometrıa local, 130

la aplicacion tangente en el punto, 55localmente conexo, 12localmente conexo por caminos, 12

metrica pseudoriemanniana, 130metrica riemanniana, 129matriz jacobiana, 49

matriz jacobiana de φ en el punto p,55

metrizable, 14monomorfismo, 16morfismo de grupos de Lie, 95metrica, 13metrica cartesinana, 14metrica euclidiana, 14

operador derivacion, 101operador lineal, 104orbita, 84orientacion de un espacio vectorial

real, 110orientacion de una variedad, 110

paralelizable, 109primero contable, 11producto, 12producto interno, 129

r-forma, 105rango de una aplicacion

diferenciable, 56rango de una aplicacion lineal, 56restricion, 9

sımbolos de Christoffel, 126segundo contable, 11sistema dinamico diferenciable, 89submersion, 68submersion en un punto, 68submersion global, 68subvariedad, 61subvariedad regular, 61suma disjunta, 12

tensor curvatura, 144tensor de Ricci, 148topologıa, 10topologıa inducida por la estructrura

diferenciable, 30transformacion, 83

variedad cociente, 68variedad cociente vıa f , 68variedad diferenciable

m-dimensional, 24variedad orientable, 110variedad orientada, 110variedad pseudoriemanniana, 130variedad riemanniana, 130vector cotangente, 100vectores, 15

Page 177: Apuntes Geom Diferencial

INDICE ALFABETICO 171

velocidad, 124

Page 178: Apuntes Geom Diferencial
Page 179: Apuntes Geom Diferencial

Bibliografıa

1. Apostol, T.M. Analisis Matematico, 2a edicion, ed. Reverte, 1976.2. Bishop, R.L. Geometry of Manifolds, Academic Press, New York, 1964.3. Bredon, G.E. An Introduction to Transformation Groups, Academic Press,

New York, 1979.4. Boothby, W.M. Introduction to Differentiable Manifolds and Riemannian

Geometry, Academic Press, New York, 1975.5. Brickell, F., Clark, R.S. Differentiable manifolds. An Introduction, Van Nos-

trand Reinhold Company, London, 1970.6. Chevalley, C. Theory of Lie Groups, Pinceton Univ. Press, Princeton, N.J.

, 1946.7. Conner, P.E. Differentiable Periodic Maps (second edition) , Springer, 1979.8. Cordero, L. A., Fernandez, M. and Gray, A. Geometrıa diferencial de curvas

y superficies (con Mathematica), Addison-Wesley Iberoamericana, 1995.9. Cheeger, J. and Ebin, D.G. Comparison Theorems in Riemannian Geo-

metry, North Holland/ American Elsevier, 1975.10. Carmo, M. P. do. Riemannian Geometry, Birkhauser, Boston, 1993.11. Carmo, M. P. do. Geometrıa diferencial de curvas y superficies, Alianza

Universidad Textos, 1994.12. Brocker, T. and Janich, K. Introduccion a la Topologıa Diferencial, Edito-

rial AC, 1977.13. Dubrovin, B. A., Fomenko, A.T. and Novikov S. P. Modern Geometry-

Methods. Part II. The Geometry and Topology of Manifolds, Springer-Verlag, New York, 1990.

14. Fleming, W. Functions of Several Variables, (2nd edition), Springer Verlag,1977, New York.

15. Guillemin , V. and Pollack, A. Differential Topology, Prentice Hall, 1974.16. Gray, A. , Abbena, E. and Salamon, S. Modern Differential Geometry of

Curves and Surfaces with Mathematica (3e), CRC Press, 2006.17. Hicks, N. J. Notas sobre Geometrıa Diferencial, Editorial Hispano Europea,

1974.18. Hirsch, M.W. Differential Topology, Springer-Verlag, 1976.19. Kobayashi, S. and Nomizu, K. Foundations of Differential Geometry. Vo-

lume I and II, John Wiley and Sons, Inc., 1963.20. Lang, S. Differential and Riemannian Manifolds, Springer (GTM 160),

1995.21. Madsen, I. and Tornehave, J. From Calculus to Cohomology, Cambridge

Univ. Press, 1997.22. Margalef J. y Outerelo, E. Topologıa Diferencial, C.S.I.C, 1988.23. Matsushima, Y. Differentiable Manifold, Marcel Decker, New York, 1972.24. Milnor, J. W. On manifods homeomorphic to the 7-sphere, Ann. Math. 64

(1956), 399-405.25. Milnor, J. W. Morse Theory, Princeton University Press (Study 51), 1963.

173

Page 180: Apuntes Geom Diferencial

174 BIBLIOGRAFIA

26. Milnor, J. and Stasheff, J.D. Characteristic classes, Princeton UniversityPress (Study 51), 1974.

27. Mishchenko, A. and Fomenko, A. A Course of Differential Geometry andTopology, Mir Publishers Moscow, 1988.

28. Montesinos, J. M. Classical Tessellations and Threee-Manifolds, Springer,1985.

29. Okubo, T. Differential Geometry, Marcel Dekker, Inc. New York and Basel,1987.

30. Olver, P. J. Applications of Lie Groups to Differential Equations, Springer-Verlag New York, 1993.

31. O’Neill, B. Semi-Riemannian Geometry. With Applications to Relativity,Academic Press,1983.

32. Outerelo, E. y Ruiz J. M. Topologıa Diferencial. Variedades con borde.Transversalidad. Aproximacion, Addison-Weley, 1998.

33. Poor, W. L. Differential Geometric Structure, McGraw-Hill, New York,1981.

34. Sharpe, R. W. Differential Geometry. Cartan’s Generalization of Klein’sErlangen Program, Springer (GTM 166), 1997.

35. Singer, I.M. and Thorpe, J.A. Lecture notes on elementary topology andgeometry, Springer-Verlag, 1967.

36. Thorpe, J.A. Elementary Topics in Differential Geometry, Springer-Verlag,1979.

37. Thurston, W.P. Three-Dimensional Geometry and Topology. Volume 1,Princeton Univ. Press, New Jersey, 1997.

38. Vinberg, E. B. (Ed) Geometry II. Spaces of constant Curvature, Encyclo-paedia of Mathematical Sicences, vol. 29, Springer-Verlag, Berlin, 1993.

39. Warner, F.W. Foundations of Differentiable Manifolds and Lie Groups,Scoot, Foresman and Company, 1971.

40. Willmore, T.J. Riemannian Geometry, Clarendon Press, Oxford, 1993.41. Whitney, H. Differentiable Manifolds, Ann. Maths. 37, no. 3, (1936) 645-

680.42. Whitney, H. Self intersections of a smooth n-manifold in 2n-space, Ann.

Maths. 45 (1944) 220-246.43. Eves, H. Estudio de las Geometrıas, Tomos I y II, UTEHA, Mejico, 1969.44. Rıbnikov, K. Historia de las Matematicas, Editorial Mir, Moscu, 1987.45. Bonola, R. Non-euclidean Geometry, Dover Publications, Inc., New York,

1955.